You are on page 1of 189

Funtional Analysis

Lecture notes for 18.102

Richard Melrose
Department of Mathematics, Massachusetts Institute of Technol-
ogy
E-mail address: rbm@math.mit.edu
Version 0.8D; Revised: 4-5-2010; Run: April 14, 2011 .
Contents

Preface 5
Introduction 6
Chapter 1. Normed and Banach spaces 9
1. Vector spaces 9
2. Normed spaces 10
3. Banach spaces 13
4. Operators and functionals 15
5. Quotients 17
6. Completion 19
7. Examples 22
8. Baire’s theorem 23
9. Uniform boundedness 24
10. Open mapping theorem 25
11. Closed graph theorem 27
12. Hahn-Banach theorem 27
13. Double dual 31
14. Problems 31
15. Solutions to problems 33
Chapter 2. The Lebesgue integral 37
1. Continuous functions of compact support (2011) 38
2. Step functions 44
3. Lebesgue integrable functions 46
4. Covering lemmas 47
5. Density of step functions (2011) 48
6. Sets of measure zero 49
7. Monotonicity lemmas 51
8. Linearity of L1 (R) 53
9. The integral is well-defined
R 54
10. The seminorm |f | 57
11. Null functions 59
12. The space L1 (R) 61
13. The three integration theorems 62
14. Notions of convergence (2011) 65
15. The space L2 (R) (2011) 66
16. The spaces Lp (R) 71
17. Lebesgue measure 74
18. Measures on the line 75
19. Higher dimensions (2011) 76
3
4 CONTENTS

20. Problems 78
21. Solutions to problems 83
Chapter 3. Hilbert spaces 85
1. pre-Hilbert spaces 85
2. Hilbert spaces 86
3. Orthonormal sets 87
4. Gram-Schmidt procedure 87
5. Complete orthonormal bases 88
6. Isomorphism to l2 89
7. Parallelogram law 89
8. Convex sets and length minimizer 90
9. Orthocomplements and projections 91
10. Riesz’ theorem 92
11. Adjoints of bounded operators 93
12. Compactness and equi-small tails 94
13. Finite rank operators 96
14. Compact operators 98
15. Weak convergence 99
16. The algebra B(H) 102
17. Spectrum of an operator 104
18. Spectral theorem for compact self-adjoint operators 105
19. Functional Calculus 108
20. Compact perturbations of the identity 109
21. Fredholm operators 112
22. Problems 113
23. Solutions to problems 125
Chapter 4. Applications 127
1. Fourier series and L2 (0, 2π). 127
2. Dirichlet problem on an interval 130
3. Harmonic oscillator 136
4. Completeness of Hermite basis 138
5. Fourier transform 142
6. Dirichlet problem 145

Appendix: Exam Preparation Problems 147


Solutions to problems 151
Bibliography 189
PREFACE 5

Preface
These are notes for the course ‘Introduction to Functional Analysis’ – or in the
MIT style, 18.102 for Spring 2011 and represent only a mild revision of the notes
from earlier years.
6 CONTENTS

Thanks to many people, for comments and corrections to the notes, including
most recently Greg Hersh.

Introduction
This course is intended for ‘well-prepared undergraduates’ meaning specifically
that a rigourous background in analysis at roughly the level of the first half of
Rudin’s book [2] is assumed – at MIT this is 18.100B. In particular the basic
theory of metric spaces is used freely. Some familiarity with linear algebra is also
assumed, but not at a very sophisticated level.
The main aim of the course in a mathematical sense is the presentation of
the standard constructions of linear functional analysis, centred on Hilbert space
and its most significant analytic realization as the Lebesgue space L2 (R). In a one-
semester course at MIT it is possible to include one or two applications such as
the existence of eigenfunctions for ‘Hill’s operator’ or the Dirichlet problem on an
interval, [0, 1] ⊂ R.
Let V : [0, 1] −→ R be a real-valued function. We are interested in ‘oscillating
modes’ on the interval; something like this arises in quantum mechanics for instance.
Namely we want to know about functions u(x) – twice continuously differentiable
on [0, 1] so that things make sense – which satisfy the differential equation
d2 u
− (x) + V (x)u(x) = λu(x) and the
(1) dx2
boundary conditions u(0) = u(1) = 0.
Here the eigenvalue, λ is an ‘unknown’ constant. More precisely we wish to to know
which such λ’s can occur. In fact all λ’s can occur with u ≡ 0 but this is the ‘trivial
solution’ which will always be there for such an equation. What other solutions are
there? The main result is that there is an infinite sequence of λ’s for which there
is a non-trivial solution of (1) λj ∈ R – they are all real, no non-real complex λ’s
can occur. For each of these there is at least one (and maybe more) ‘independent’
solution uj . We can say a lot more about everything here but one main aim of this
course is to get at least to this point.
So the journey to a discussion of the Dirichlet problem is rather extended and
apparently wayward. The relevance of Hilbert space and the Lebesgue integral
is not immediately apparent, but I hope will become clear as we proceed. The
basic idea is that we consider a space of all ‘potential’ solutions to the problem at
hand. In this case one might take the space of all twice continuously differentiable
functions on [0, 1] – we will consider such spaces at least briefly below. In any case
one of the significant properties of the equation (1) is that it is ‘linear’. So we
start with a brief discussion of linear spaces. What we are dealing with here can be
thought of as the an eigenvalue problem for an ‘infinite matrix’. This in fact is not a
very good way of looking at things (there was such a matrix approach to quantum
mechanics in the early days but it was replaced by the sort of ‘operator’ theory
on Hilbert space that we will use here.) One of the crucial distinctions between
the treatment of finite dimensional matrices and an infinite dimensional setting is
that in the latter topology is encountered. This is enshrined here in the notion of a
normed linear space which is the first important topic treated.
After a brief treatment of normed and Banach spaces, the course proceeds to
the construction of the Lebesgue integral on the line, leading to the definition of
INTRODUCTION 7

the space L1 (R). I follow here the idea of Mikusiński that one can simply define
integrable functions as the almost everywhere limits of absolutely summable series
of step functions and more significantly the basic properties can be deduced this
way. This approach is followed in the book of Debnaith and Mikusiński; why did I
not just follow their book? I did try it.
After a three-week stint of measure and integration the course proceeds to the
more gentle ground of Hilbert spaces. Here I am influenced by the book of Simmons.
We proceed to a short discussion of operators and the spectral theorem for compact
self-adjoint operators. Then in the last third or so of the semester this theory is
applied to the treatment of the Dirichlet eigenvalue problem and also the eigenvalue
problem for ‘Hill’s equation’, essentially the same question with periodic boundary
conditions. Finally various loose end are brought together, or at least that is my
hope.
CHAPTER 1

Normed and Banach spaces

There are many good references for this material and it is always a good idea
to get at least a couple of different views. I suggest the following on-line sources
(1) Wilde [4]
(2) Chen [1]
(3) Ward [3]

1. Vector spaces
So, you should have some familiarity with linear, or I will usuall say ‘vector’,
spaces. Should I break out the axioms? I think not. It is a space V in which
we can add elements and multiply by scalars with rules very similar to the basic
examples of Rn or Cn . Note that for us the ‘scalars’ are either the real numbers of
the complex numbers – usually the latter. Let’s be neutral and denote by K either
R or C but of course consistently. Then our set V – the set of vectors with which
we will deal, comes with two ‘laws’. These are maps
(1.1) + : V × V −→ V, · : K × V −→ V.
which we denote not by +(v, w) and ·(s, v) but by v + w and sv. Then we impose
the axioms of a vector space – look them up! These are commutative group axioms
for + axioms for the action of K and the distributive law.
The basic examples:
The field K which is either R or C is a vector space over itself.
The vector spaces Kn consisting of order n-tuples of elements of K. Addition is by
components and the action of K is by multiplication on all components. You should
be reasonably familiar with these spaces and other finite dimensional vector spaces.
Seriously non-trivial examples such as C([0, 1]) the space of continuous functions
on [0, 1] (say with complex values).
In these and many other examples we will encounter below the ‘component
addition’ corresponds to the addition of functions.
Lemma 1. If X is a set then the spaces of all functions
(1.2) F(X; R) = {u : X −→ R}, F(X; C) = {u : X −→ C}
are vector spaces over R and C respectively.
Non-Proof. Since I have not written out the axioms of a vector space it is
hard to check this – and I leave it to you as the first of many important exercises.
In fact, better do it more generally as in Problem 1.1 – then you can say ‘if V is
a linear space then F(X; V ) inherits a linear structure’. The main point to make
sure you understand is that, because we do know how to add and multiply in either
9
10 1. NORMED AND BANACH SPACES

R and C, we can add functions and multiply them (by constants or indeed other
functions):-
(1.3) (c1 f1 + c2 f2 )(x) = c1 f1 (c) + c2 f2 (x)
defines the function c1 f1 + c2 f2 if c1 , c2 ∈ K and f1 , f2 ∈ F(X; K). 

Here and below we sometime write K to stand for R or C consitently in an


argument when either will work.
Most of the linear spaces we will meet are either subspaces of these function-
type spaces, or quotients – see Problems 1.2 and 1.3.
Although you are probably most comfortable with finite dimensional vector
spaces. This is based on the notion of linear independence of a subset of a vector
space. Thus a subset E ⊂ V is said to be linearly independent if for any finite
collection of elements vi ∈ E, i = 1, . . . , N, and any collection of ‘constants’ ai ∈ K,
i = 1, . . . , N the identity
N
X
(1.4) ai vi = 0 =⇒ ai = 0 ∀ i.
i=1

That is, it is a set in which there are ‘no non-trivial finite linear dependence relations
between the elements’. A vector space if finite-dimensional if the is a finite and
maximal linearly independent subset – a basis – where maximal means that if any
new element is added to the set E then it is no longer linearly independent. A
basic result is that any two such ‘bases’ in a finite dimensional vector space have
the same number of elements.
Still it is time to leave this secure domain behind since we most interested in
the other case, namely infinite-dimensional vector spaces.
Definition 1. A vector space is infinite-dimensional if it is not finite dimen-
sional, i.e. for any N ∈ N there exist N elements with no, non-trivial, linear
dependence relation between them.
So, finite-dimensional vector spaces have finite bases, infinite-dimensional vec-
tor spaces do not. The notion of a basis in an infinite-dimensional vector spaces
needs to be modified to be useful analytically. Convince yourself that the vector
space in Lemma 1 is infinite dimensional if and only if X is infinite.

2. Normed spaces
In order to deal with infinite-dimensional vector spaces we need the control
given by a metric (or more generally a non-metric topology, but we will not quite
get that far). A norm on a vector space leads to a metric which is ‘compatible’
with the linear structure.
Definition 2. A norm on a vector space is a function, traditionally denoted
(1.5) k · k : V −→ [0, ∞)
with the following properties
(Definiteness)
(1.6) v ∈ V, kvk = 0 =⇒ v = 0.
2. NORMED SPACES 11

(Absolute homogeneity) For any λ ∈ K and v ∈ V,


(1.7) kλvk = |λ|kvk.
(Triangle Inequality) The triangle inequality holds, in the sense that for any two
elements v, w ∈ V
(1.8) kv + wk ≤ kvk + kwk.
Note that (1.7) implies that k0k = 0. Thus (1.6) means that kvk = 0 is equiv-
alent to v = 0. This definition is based on the same properties holding for the
standard norm(s), |z|, on R and C. You should make sure you understand that
(
x if x ≥ 0
R 3 x −→ |x| = ∈ [0, ∞) is a norm as is
(1.9) −x if x ≤ 0
1
C 3 z = x + iy −→ |z| = (x2 + y 2 ) 2 .
Situations do arise in which we do not have (1.6):-
Definition 3. A function (1.5) which satisfes (1.7) and (1.8) but possibly not
(1.6) is called a seminorm.
A metric, or distance function, on a set is a map
(1.10) d : X × X −→ [0, ∞)
satisfying three standard conditions
(1.11) d(x, y) = 0 ⇐⇒ x = y,
(1.12) d(x, y) = d(y, x) ∀ x, y ∈ X and
(1.13) d(x, y) ≤ d(x, z) + d(z, y) ∀ x, y, z ∈ X.
If you do not know about metric spaces, then you are in trouble. I suggest that
you take the appropriate course now and come back next year. You could read the
first few chapters of Rudin’s book [2] before trying to proceed much further but it
will be a struggle to say the least. The point of course is
Proposition 1. If k · k is a norm on V then
(1.14) d(v, w) = kv − wk
is a distance on V turning it into a metric space.
Proof. Clearly (1.11) corresponds to (1.6), (1.12) arises from the special case
λ = −1 of (1.7) and (1.12) arises from (1.8). 
We will not use any special notation for the metric, nor usually mention it
explicitly – we just subsume all of metric space theory from now on. So kv − wk is
the distance between two points in a normed space.
Now, we need to talk about examples. The most important ones are the usual
finite-dimensional spaces Rn and Cn with their Euclidean norms
! 12
X
(1.15) |x| = |xi |2
i
where it is at first confusing that we just use single bars for the norm, just as for
R and C, but you just need to get used to that.
12 1. NORMED AND BANACH SPACES

There are other norms on Cn (I will mostly talk about the complex case, but
the real case is essentially the same). The two most obvious ones are
|x|∞ = max |xi |, x = (x1 , . . . , xn ) ∈ Cn ,
(1.16)
X
|x|1 = |xi |
i

but as you will see (if you do the problems) there are also the norms
X 1
(1.17) |x|p = ( |xi |p ) p , 1 ≤ p < ∞.
i

In fact, for p = 1, (1.17) reduces to the second norm in (1.16) and in a certain sense
the case p = ∞ is consistent with the first norm there.
I usually do not discuss the notion of equivalence of norms straight away. How-
ever, two norms on the one vector space – which we can denote k · k(1) and k · k(2)
are equivalent if there exist constants C1 and C2 such that
(1.18) kvk(1) ≤ C1 kvk(2) , kvk(2) ≤ C2 kvk(1) ∀ v ∈ V.
The equivalence of the norms implies that the metrics define the same open sets –
the topologies induced are the same. You might like to check that the reverse is also
true, if two norms induced the same topologies (just meaning the same collection
of open sets) through their assoicated metrics, then they are equivalent in the sense
of (1.18).
See Problem 1.5 which shows why we are not so interested in the finite-dimensional
case – namely any two norms on a finite dimensional vector space are equivalent
and so in that case a choice of norm does not tell us much, although it certainly
has its uses.
One important class of normed spaces consists of the spaces of bounded con-
tinuous functions on a metric space X :
0 0
(1.19) C∞ (X) = C∞ (X; C) = {u : X −→ C, continuous and bounded} .
That this is a linear space follows from the (obvious) result that a linear combi-
nation of bounded functions is bounded and the (less obvious) result that a linear
combination of continuous functions is continuous; this we know. The norm is the
best bound
(1.20) kuk∞ = sup |u(x)|.
x∈X

That this a norm is easy to check. Absolute homogeneity is clear, kλuk∞ = |λ|kuk∞
and kuk∞ = 0 means that u(x) = 0 for all x ∈ X which is exactly what it means
for a function to vanish. The triangle inequality ‘is inherited from C’ since for any
two functions and any point,
(1.21) |(u + v)(x)| ≤ |u(x)| + |v(x)| ≤ kuk∞ + kvk∞
by the definition of the norms, and taking the sup of the left gives ku + vk∞ ≤
kuk∞ + kvk∞ .
Of course the norm (1.20) is defined even for bounded, not necessarily contin-
0
uous functions on X. Note that convergence of a sequence un ∈ C∞ (X) (remember
this means with respect to the distance induced by the norm) is just uniform con-
vergence
(1.22) kun − vk∞ → 0 ⇐⇒ un (x) → v(x) uniformly on X.
3. BANACH SPACES 13

Other examples of infinite dimensional normed spaces are the space lp in the
problems below. Of these l2 is the most important for us. It is in fact one form of
Hilbert space, with which we are primarily concerned:-
X
(1.23) l2 = {a : N −→ C; |a(j)|2 < ∞}.
j

It is not immediately obvious that this is a linear space, nor that


  21
X
(1.24) kak2 =  |a(j)|2 
j

is a norm. It is. From now on we will generally use sequential notation and think
of a map from N to C as a sequence, so setting a(j) = aj . Thus the ‘Hilbert space’
l2 consists of the square summable sequences.

3. Banach spaces
You are supposed to remember from metric space theory that there are three
crucial properties, completeness, compactness and connectedness. It turns out that
normed spaces are always connected, so that is not very interesting, and they are
never compact (unless you consider the trivial case V = {0}) so that is not very
interesting either – altough we will ultimately be very interested in compact subsets
– so that leaves completeness. That is so important that we give it a special name.
Definition 4. A normed space which is complete with respect to the induced
metric is a Banach space.
0
Lemma 2. The space C∞ (X), for any metric space X, is a Banach space.
Proof. This is a standard result from metric space theory – basically that the
uniform limit of a sequence of (bounded) continuous functions on a metric space is
continuous. See Problem ?? where a proper proof is outlined. 
There is a space of sequences which is really an example of this Lemma. Con-
sider the space c0 consisting of all the sequence {aj } (valued in C) such that
limj→∞ aj = 0. As remarked above, sequences are just functions N −→ C. If
we make {aj } into a function on α : D = {1, 1/2, 1/3, . . . } −→ C by setting
α(1/j) = aj then we get a function on the metric space D. Add 0 to D to get
D = D ∪ {0} ⊂ [0, 1] ⊂ R; clearly 0 is a limit point of D and D is, as the nota-
tion dangerously indicates, the closure of D in R. Now, you will easily check (it is
really the definition) that α : D −→ C corresponding to a sequence, extends to a
continuous function on D vanishing at 0 if and only if limj→∞ aj = 0, which is to
say, {aj } ∈ c0 . Thus it follows, with a little thought which you should give it, that
c0 is a Banach space with the norm
(1.25) kak∞ = sup kaj k.
j

What is an example of a non-complete normed space, a normed space which is


not a Banach space? These are legion of course. The simplest way to get one is to
‘put the wrong norm’ on a space, one which does not correspond to the definition.
Consider for instance the linear space T of sequences N −→ C which ‘terminate’,
i.e. each element {aj } ∈ T has aj = 0 for j > J, where of course the J may depend
14 1. NORMED AND BANACH SPACES

on the particular sequence. The T ⊂ c0 , the norm on c0 defines a norm on T but


it cannot be complete, since the closure of T is easily seen to be all of c0 – so there
are Cauchy sequences in T without limit in T .
One result we will exploit below, and I give it now just as preparation, concerns
absolutely summable series. Recall that a series is just a sequence where we ‘think’
about adding the terms. So a sequence {vn } is said to be an absolutely summable
series if
X
(1.26) kvn k < ∞.
n

Proposition 2. The sequence of partial sums of any absolutely summable se-


ries in a normed space is Cauchy and a normed space is complete if and only if
every absolutely summable series in it converges, meaning that the sequence of par-
tial sums converges.
Proof. The sequence of partial sums is
n
X
(1.27) wn = vj .
j=1

Thus, if m ≥ n then
m
X
(1.28) wm − wn = vj .
j=n+1

It follows from the triangle inequality that


m
X
(1.29) kwn − wm kV ≤ kvj kV .
j=n+1

P ∞
P
So if the series is absolutely summable then kvj kV < ∞ and limn→∞ kvj kV =
j=1 j=n+1
0. Thus {wn } is Cauchy if {vj } is absolutely summable. Hence if V is complete
then every absolutely summable series is summable, i.e. the sequence of partial
sums converges.
Conversely, suppose that every absolutely summable series converges in this
sense. Then we need to show that every Cauchy sequence in V converges. Let
un be a Cauchy sequence. It suffices to show that this has a subsequence which
converges, since a Cauchy sequence with a convergent subsequence is convergent.
To do so we just proceed inductively. Using the Cauchy condition we can for every
k find an integer Nk such that
(1.30) n, m > Nk =⇒ kun − um k < 2−k .
Now choose an increasing sequence nk where nk > Nk and nk > nk−1 to make it
increasing. It follows that
(1.31) kunk − unk−1 k ≤ 2−k+1 .
Denoting this subsequence as u0k = unk it follows from (1.31) and the triangle
inequality that

X
(1.32) ku0n − u0n+1 k ≤ 4
n=1
4. OPERATORS AND FUNCTIONALS 15

so the sequence v1 = u01 , vk = u0k − u0k−1 is absolutely summable. Its sequence of


partial sums is wj = u0j so the assumption is that this converges, hence the original
Cauchy sequence converges and V is complete. 
Notice the idea here, of ‘speeding up the convergence’ of the Cauchy sequence
by dropping a lot of terms. We will use this idea of absolutely summable series
heavily in the discussion of Lebesgue integration.

4. Operators and functionals


I suggest that you read this somewhere else (as well) for instance Wilde, [4],
Chapter 2 to 2.7, Chen, [1], the first part of Chapter 6 and of Chapter 7 and/or
Ward, [3], Chapter 3, first 2 sections.
The vector space we are most intereted in are, as already remarked, spaces
of functions (or something a little more general). The elements of these are the
objects of primary interest but they are related by linear maps. A map between
two vector spaces (over the same field, for us either R or C) is linear if it takes
linear combinations to linear combinations:-
(1.33) T : V −→ W, T (a1 v1 +a2 v2 ) = a1 T (v1 )+a2 T (v2 ), ∀ v1 , v2 ∈ V, a1 , a2 ∈ K.
The sort of examples we have in mind are differential, or more especially, integral
operators. For instance if u ∈ C 0 ([0, 1]) then its Riemann integral
Z x
(1.34) (T u)(x) = u(s)ds
0
is continuous in x ∈ [0, 1] and so defines a map
(1.35) T : C 0 ([0, 1]) −→ C 0 ([0, 1]).
This is a linear map, with linearity being one of the standard properties of the
Riemann integral.
In the finite dimensional case linearity is enough to allow maps to be studied.
However in the case of infinite-dimensional normed spaces we need to impose con-
tinuity. Of course it makes perfectly good sense to say, demand or conclude, that
a map as in (1.33) is continuous if V and W are normed spaces since they are then
metric spaces. Recall that for metric spaces there are several different equivalent
conditions that ensure a map is continuous:
(1.36) vn → v in V =⇒ T vn → T v in W
(1.37) O ⊂ W open =⇒ T −1 (O) ⊂ V open
(1.38) C ⊂ W closed =⇒ T −1 (C) ⊂ V closed.
For a linear map between normed spaces there is a simpler characterization of
continuity in terms of the norm.
Proposition 3. A linear map (1.33) between normed spaces is continuous if
and only if it is bounded in the sense that there exists a constant C such that
(1.39) kT vkW ≤ CkvkV ∀ v ∈ V.
Of course bounded for a function on a metric space already has a meaning and this
is not it! The usual sense would be kT vk ≤ C but this would imply kT (av)k =
|a|kT vk ≤ C so T v = 0. Hence it is not so dangerous to use the term ‘bounded’ for
16 1. NORMED AND BANACH SPACES

(1.39) – it is really ‘relatively bounded’. From now on, bounded for a linear map
means (1.39).
Proof. If (1.39) holds then if vn → v in V it follows that kT v − T vn k =
kT (v − vn )k ≤ Ckv − vn k → 0 as n → ∞ so T vn → T v and continuity follows.
For the reverse implication we use second characterization of continuity above.
Thus if T is continuous then the inverse image T −1 (B(0, 1)) of the open unit ball
around the origin contains the origin in V and so, being open, must contain some
B(0, ) in V. This means that
(1.40) T (B(0, )) ⊂ B(0, 1) so kvk <  =⇒ kT vk ≤ 1.
Now scaling if v ∈ V then kv 0 k <  where v 0 = v/2kvk which means that kT v 0 k ≤ 1
but this implies (1.39) with C = 2/ – it is trivially true if v = 0. 
So, if T : V −→ W is continous and linear between normed spaces, or from
now on ‘bounded’, then
(1.41) kT k = sup kT vk < ∞.
kvk=1

Lemma 3. The bounded linear maps between normed spaces V and W form a
linear space B(V, W ) on which kT k defined by (1.40) or equivalently
(1.42) kT k = inf{C; (1.39) holds}
is a norm.
Proof. First check that (1.41) is equivalent to (1.42). Define kT k by (1.41).
Then for any v ∈ V, v 6= 0,
v kT vk
(1.43) kT k ≥ kT ( k= =⇒ kT vk ≤ kT kkvk
kvk kvk
since as always this is trivially true for v = 0. Thus kT k is a constant for which
(1.39) holds.
Conversely, from the definition of kT k, if  > 0 then there exists v ∈ V with
kvk = 1 such that kT k −  < kvk ≤ C for any C for which (1.39) holds. Since  > 0
is arbitrary, kT k ≤ C and hence kT k is given by (1.42).
From the definition of kT k, kT k = 0 implies T v = 0 for all v ∈ V and hence
T = 0 as a map. Similarly, if λ 6= 0, λ ∈ K then
(1.44) kλT k = sup kλT vk = |λ|kT k
kvk

and this is also obvious for λ = 0. This only leaves the triangle inequality to check
and for any T, S ∈ B(V, W ), and v ∈ V with kvk = 1
(1.45) k(T + S)vkW = kT v + SvkW ≤ kT vk + W + kSvkW ≤ kT k + kSk
so taking the supremum, kT + Sk ≤ kT k + kSk. 
Thus we see that the space of bounded linear maps between two normed spaces
is a normed space, with the norm being the best constant in the estimate (1.39).
Such bounded linear maps between normed spaces are often called ‘operators’ be-
cause we are thinking of the normed spaces as being like function spaces.
You might like to check boundedness for the example of a linear operator in
(1.35), namely that in terms of the supremum norm on C 0 ([0, 1]), kT k ≤ 1.
5. QUOTIENTS 17

One particularly important case is when W = K is the field, for us usually C.


Then a simpler notation is handy and one sets V 0 = B(V, C) – this is called the
dual space of V.
Proposition 4. If W is a Banach space then B(V, W ) with the norm (1.41)
is a Banach space.
Proof. We simply need to show that if W is a Banach space then every Cauchy
sequence in B(V, W ) is convergent. The first thing to do is to find the limit. To
says that Tn ∈ B(V, W ) is Cauchy, is just to say that given  > 0 there exists N
such that n, m > N implies kTn − Tm k < . By the definition of the norm, if v ∈ V
then kTn v − Tm vkW ≤ kTn − Tm kkvkV so Tn v is Cauchy in W for each v ∈ V. By
assumption, W is complete, so
(1.46) Tn v −→ w in W.
However, the limit can only depend on v so we can define a map T : V −→ W by
T v = w in (1.46).
This map defined from the limits is linear, since Tn (λv) = λTn v −→ λT v and
Tn (v1 + v2 ) = Tn v1 + Tn v2 −→ T v2 + T v2 = T (v1 + v2 ). Moreover, |kTn k − kTn k| ≤
kTn − Tm k so kTn k is Cauchy in [0, ∞) and hence converges, with limit S, and
(1.47) kT vk = lim kTn vk ≤ Skvk
n→∞

so kT k ≤ S shows that T is bounded.


Returning to the Cauchy condition above and passing to the limit in kTn −
Tm vk ≤ kvk as m → ∞ shows that kTn − T k ≤  if n > M and hence Tn → T in
B(V, W ) which is therefore complete. 
One simple consequence of this is:-
Corollary 1. The dual space of a normed space is always a Banach space.
However you should be a little suspicious here since we have not shown that
the dual space V 0 is non-trivial, meaning we have not eliminated to possibility that
V 0 = {0} even when V 6= {0}. The Hahn-Banach Theorem, discussed below, takes
care of this.

5. Quotients
The notion of a linear subspace of a vector space is natural enough, and you
are likely quite familiar with it. Namely W ⊂ V where V is a vector space is
a (linear) subspace if any linear combinations λ1 w1 + λ2 w2 ∈ W if λ1 , λ2 ∈ K
and w1 , w2 ∈ W. Thus W ‘inherits’ its linear structure from V. There is a second
very important way to construct new linear spaces from old. Namely we want to
make a linear space out of ‘the rest’ of V, given that W is a linear subspace. In
finite dimensions one way to do this is to give V an inner product and then take
the subspace orthogonal to W. One problem with this is that the result depends,
although not in an essential way, on the inner product. Instead we adopt the usual
‘myopia’ approach and take an equivalance relation on V which identifies points
which differ by an element of W. The equivalence classes are then ‘planes parallel
to W ’. I am going through this construction quickly here under the assumption
that it is familiar to most of you, if not you should think about it carefully since
we need to do it several times later.
18 1. NORMED AND BANACH SPACES

So, if W ⊂ V is a linear subspace of V we define a relation on V – remember


this is just a subset of V × V with certain properties – by
(1.48) v ∼W v 0 ⇐⇒ v − v 0 ∈ W ⇐⇒ ∃ w ∈ W s.t. v = v 0 + w.
This satisfies the three conditions for an equivalence relation:
(1) v ∼W v
(2) v ∼W v 0 ⇐⇒ v 0 ∼W v
(3) v ∼W v 0 , v 0 ∼W w00 =⇒ v ∼W v 00
which means that we can regard it as a ‘coarser notion of equality.’
Then V /W is the set of equivalence classes with respect to ∼W . You can think
of the elements of V /W as being of the form v + W – a particular element of V
plus an arbitrary element of W. Then of course v 0 ∈ v + W if and only if v 0 − v ∈ W
meaning v ∼W v 0 .
The crucial point here is that
(1.49) V /W is a vector space.
You should check the details – see Problem ??. Note that the ‘is’ in (1.49) should
really be expanded to ‘is in a natural way’ since as usual the linear structure is
inherited from V :
(1.50) λ(v + W ) = λv + W, (v1 + W ) + (v2 + W ) = (v1 + v2 ) + W.
The subspace W appears as the origin in V /W.
Now, two immediate cases of this are of interest to us, the second less so.
Proposition 5. If k · k is a seminorm on V then
(1.51) E = {v ∈ V ; kvk = 0} ⊂ V
is a linear subspace and
(1.52) kv + EkV /E = kvk
defines a norm on V /E.
Proof. That E is linear follows from the properties of a seminorm, since
kλvk = |λ|kvk shows that λv ∈ E if v ∈ E and λ ∈ K. Similarly the triangle
inequalty shows that v1 + v2 ∈ E if v1 , v2 ∈ E.
To check that (1.52) defines a norm, first we need to check that it makes sense
as a function k · kV /E −→ [0, ∞). This amounts to the statement that kv 0 k is the
same for all elements v 0 = v + e ∈ v + E for a fixed v. This however follows from
the triangle inequality applied twice:
(1.53) kv 0 k ≤ kvk + kek = kvk ≤ kv 0 k + k − ek = kv 0 k.
Now, I leave you the exercise of checking that k·kV /E is a norm, see Problem ?? 
The second application is more serious, but in fact we will not use it for some
time so I usually do not do this in lectures at this stage.
Proposition 6. If W ⊂ V is a closed subspace of a normed space then
(1.54) kv + W kV /W = inf kv + wkV
w∈W

defines a norm on V /W ; if V is a Banach space then so is V /W.


For the proof see Problems ?? and ??.
6. COMPLETION 19

6. Completion
A normed space not being complete, not being a Banach space, is considered
to be a defect which we might, indeed will, wish to rectify.
Let V be a normed space with norm k · kV . A completion of V is a Banach
space B with the following properties:-
(1) There is an injective (i.e. 1-1) linear map I : V −→ B
(2) The norms satisfy
(1.55) kI(v)kB = kvkV ∀ v ∈ V.
(3) The range I(V ) ⊂ B is dense in B.
Notice that if V is itself a Banach space then we can take B = V with I the
identity map.
So, the main result is:
Theorem 1. Each normed space has a completion.
There are several ways to prove this, we will come across a more sophisticated one
(using the Hahn-Banach Theorem) later. However, we want a ‘hands-on’ proof
which motivates (I hope) the construction of the Lebesgue integral – which is in
our near future.

‘Proof’ (the last bit is left to you). Let V be a normed space. First
we introduce the rather large space

( )
X

(1.56) V = {uk }k=1 ; uk ∈ V and
e kuk k < ∞
k=1

the elements of which, if you recall, are said to be absoutely summable. Notice that
the elements of Ve are sequences, valued in V so two sequences are equal, are the
same, only when each entry in one is equal to the corresponding entry in the other
– no shifting around or anything is permitted as far as equality is concerned. We
think of these as series (remember this means nothing except changing the name, a
series is a sequence and a sequence is a series), the only difference is that we ‘think’
of taking the limit of sequence but we ‘think’ of summing the elements of a series,
whether we can do so or not being a different matter.
Now, each element of Ve is a Cauchy sequence – meaning the corresponding
PN
sequence of partial sums vN = uk is Cauchy if {uk } is absolutely summable.
k=1
This is simply because if M ≥ N then
M
X M
X X
(1.57) kvM − vN k = k uj k ≤ kuj k ≤ kuj k
j=N +1 j=N +1 j≥N +1
P
gets small with N by the assumption that kuj k < ∞ (time to polish up your
j
understanding of the convergence of series of positive real numbers?)
Moreover, Ve is a linear space, where we add sequences, and multiply by con-
stants, by doing the operations on each component:-
(1.58) t1 {uk } + t2 {u0k } = {t1 uk + t2 u0k }.
20 1. NORMED AND BANACH SPACES

This always gives an absolutely summable series by the triangle inequality:


X X X
(1.59) kt1 uk + t2 u0k k ≤ |t1 | kuk k + |t2 | ku0k k.
k k k

Within Ve consider the linear subspace


( )
X X
(1.60) S = {uk }; kuk k < ∞, uk = 0
k k

of those which converge to 0. As discussed above, we can form the quotient


(1.61) B = Ve /S

the elements of which are the ‘cosets’ of the form {uk } + S ⊂ Ve where {uk } ∈ Ve .
We proceed to check the following properties of this B.
(1) A norm on B is defined by
n
X
(1.62) kbkB = lim k uk k, b = {uk } + S ∈ B.
n→∞
k=1

(2) The original space V is imbedded in B by


(1.63) V 3 v 7−→ I(v) = {uk } + S, u1 = v, uk = 0 ∀ k > 1
and the norm satisfies (1.55).
(3) I(V ) ⊂ B is dense.
(4) B is a Banach space with the norm (1.62).
So, first that (1.62) is a norm. The limit on the right does exist since the limit
of the norm of a Cauchy sequence always exists – namely the sequence of norms
is itself Cauchy but now in R. Moreover, adding an element of S to {uk } does not
change the norm of the sequence of partial sums, since the addtional term tends
to zero in norm. Thus kbkB is well-defined for each element b ∈ B and kbkB = 0
means exactly that the sequence {uk } used to define it tends to 0 in norm, hence is
in S hence b = 0 in B. The other two properties of norm are reasonably clear, since
if b, b0 ∈ B are represented by {uk }, {u0k } in Ve then tb and b + b0 are reprented by
{tuk } and {uk + u0k } and
n
X n
X
lim k tuk k = |t| lim k uk k,
n→∞ n→∞
k=1 k=1
n
X
lim k (uk + u0k )k = A =⇒
n→∞
k=1
n
X
(1.64) for  > 0 ∃ N s.t. ∀ n ≥ N, A −  ≤ k (uk + u0k )k =⇒
k=1
n
X n
X
A−≤k uk k + k u0k )k ∀ n ≥ N =⇒
k=1 k=1
A −  ≤ kbkB + kb0 kB ∀  > 0 =⇒
kb + b0 kB ≤ kbkB + kb0 kB .
6. COMPLETION 21

Now the norm of the element I(v) = v, 0, 0, · · · , is the limit of the norms of the
sequence of partial sums and hence is kvkV so kI(v)kB = kvkV and I(v) = 0
therefore implies v = 0 and hence I is also injective.
We need to check that B is complete, and also that I(V ) is dense. Here is
an extended discussion of the difficulty – of course maybe you can see it directly
yourself (or have a better scheme). Note that I ask you to write out your own
version of it carefully in one of the problems.
Okay, what does it mean for B to be a Banach space, well as we above it means
that every absolutely summable series in B is convergent. Such a series {bn } is
(n) (n)
given by bn = {uk } + S where {uk } ∈ Ve and the summability condition is that
N
(n)
X X X
(1.65) ∞> kbn kB = lim k uk k V .
N →∞
n n k=1
P
So, we want to show that bn = b converges, and to do so we need to find the
n
limit b. It is supposed to be given by an absolutely summable series. The ‘problem’
P P (n)
is that this series should look like uk in some sense – because it is supposed
n k
to represent the sum of the bn ’s. Now, it would be very nice if we had the estimate
X X (n)
(1.66) kuk kV < ∞
n k

since this should allow us to break up the double sum in some nice way so as to get
an absolutely summable series out of the whole thing. The trouble is that (1.66)
need not hold. We know that each of the sums over k – for given n – converges,
but not the sum of the sums. All we know here is that the sum of the ‘limits of the
norms’ in (1.65) converges.
So, that is the problem! One way to see the solution is to note that we do not
(n)
have to choose the original {uk } to ‘represent’ bn – we can add to it any element
(n)
of S. One idea is to rearrange the uk – I am thinking here of fixed n – so that it
‘converges even faster.’ Given  > 0 we can choose p1 so that for all p ≥ p1 ,
X (n) X (n)
(1.67) |k uk kV − kbn kB | ≤ , kuk kV ≤ .
k≤p k≥p

Then in fact we can choose successive pj > pj−1 (remember that little n is fixed
here) so that
X (n) X (n)
(1.68) |k uk kV − kbn kB | ≤ 2−j , kuk kV ≤ 2−j  ∀ j.
k≤pj k≥pj

p1 pj
(n) P (n) (n) P (n)
Now, ‘resum the series’ defining instead v1 = uk , vj = uk and
k=1 k=pj−1 +1
do this setting  = 2−n for the nth series. Check that now
X X (n)
(1.69) kvk kV < ∞.
n k

(n)
Of course, you should also check that bn = {vk } + S so that these new summable
series work just as well as the old ones.
22 1. NORMED AND BANACH SPACES

After this fiddling you can now try to find a limit for the sequence as
(p)
X
(1.70) b = {wk } + S, wk = vl ∈ V.
l+p=k+1

So, you need to check that this {wk } is absolutely summable in V and that bn → b
as n → ∞.
Finally then there is the question of showing that I(V ) is dense in B. You can
do this using the same idea as above – in fact it might be better to do it first. Given
an element b ∈ B we need to find elements in V, vk such that kI(vk ) − bkB → 0 as
Nj
P
k → ∞. Take an absolutely summable series uk representing b and take vj = uk
k=1
where the pj ’s are constructed as above and check that I(vj ) → b by computing
X X
(1.71) kI(vj ) − bkB = lim k uk kV ≤ kuk kV .
→∞
k>pj k>pj

7. Examples
Let me collect some examples of normed and Banach spaces. Those mentioned
above and in the problems include:
• c0 the space of convergent sequences in C with supremum norm, a Banach
space.
• lp one space for each real number 1 ≤ p < ∞; the space of p-summable
series with corresponding norm; all Banach spaces. The most important
of these for us is the case p = 2, which is (a) Hilbert space.
• l∞ the space of bounded sequences with supremumb norm, a Banach space
with c0 ⊂ l∞ a closed subspace with the same norm.
• C 0 ([a, b]) or more generally C 0 (M ) for any compact metric space M – the
Banach space of continuous functions with supremum norm.
0 0
• C∞ (R), or more generally C∞ (M ) for any metric space M – the Banach
space of bounded continuous functions with supremum norm.
• C00 (R), or more generally C00 (M ) for any metric space M – the Banach
space of continuous functions which ‘vanish at infinity’ (see Problem ??
0
with supremum norm. A closed subspace, with the same norm, in C∞ (M ).
k
• C ([a, b]) the space of k times continuously differentiable (so k ∈ N) func-
tions on [a, b] with norm the sum of the supremum norms on the function
and its derivatives. Each is a Banach space – see Problem ??.
• The space C 0 ([0, 1]) with norm
Z b
(1.72) kukL1 = |u|dx
a
given by the Riemann integral of the absolute value. A normed space, but
not a Banach space. We will construct the concrete completion, L1 ([0, 1])
of Lebesgue integrable ‘functions’.
• The space R([a, b]) of Riemann integrable functions on [a, b] with kuk
defined by (1.72). This is only a seminorm, since there are Riemann
integrable functions (note that u Riemann integrable does imply that |u|
is Riemann integrable) with vanishing Riemann integral but which are
8. BAIRE’S THEOREM 23

not identically zero. This cannot happen for continuous functions. So the
quotient is a normed space, but it is not complete.
• The same spaces – either of continuous or of Riemann integrable functions
but with the (semi- in the second case) norm
! p1
Z b
(1.73) kukLp = |u|p .
a

Not complete in either case even after passing to the quotient to get a
norm for Riemann integrable functions.
Here is an example to see that the space of continuous functions on [0, 1] with
norm (1.72) is not complete; things are even worse than this example indicates! It
is a bit harder to show that the quotient of the Riemann integrable functions is not
complete, feel free to give it a try!
Take a simple non-negative continuous function on R for instance
(
1 − |x| if |x| ≤ 1
(1.74) f (x) =
0 if |x| > 1.
R1
Then −1 f (x) = 1. Now scale it up and in by setting

(1.75) fN (x) = N f (N 3 x) = 0 if |x| > N −3 .


R1
So it vanishes outside [−N −3 , N −3 ] and has −1 fN (x)dx = N −2 . It follows that the
sequence {fN } is absolutely summable
P with respect to the integral norm in (1.72)
on [−1, 1]. The pointwise series fN (x) converges everywhere except at x = 0 –
N
since at each point x 6= 0, fN (x) = 0 if N 3 |x| > 1. The resulting function, even if we
ignore the problem at x = 0, is not Riemann integrable because it is not bounded.
You might respond that the sum of the series is ‘improperly Riemann inte-
grable’. This is true but does not help much.
It is at this point that I start doing Lebesgue integration in the lectures. The
following material is from later in the course but fits here quite reasonably.

8. Baire’s theorem
At least once I wrote a version of the following material on the blackboard
during the first mid-term test, in an an attempt to distract people. It did not work
very well – its seems that MIT students have already been toughened up by this
stage. Baire’s theorem will be used later (it is also known as ‘Baire category theory’
although it has nothing to do with categories in the modern sense).
This is a theorem about complete metric spaces – it could be included in the
earlier course ‘Real Analysis’ but the main applications are in Functional Analysis.
Theorem 2 (Baire). If M is a non-empty complete metric space and Cn ⊂ M,
n ∈ N, are closed subsets such that
[
(1.76) M= Cn
n

then at least one of the Cn ’s has an interior point.


24 1. NORMED AND BANACH SPACES

Proof. We can assume that the first set C1 6= ∅ since they cannot all be
empty and dropping some empty sets does no harm. Let’s assume the contrary of
the desired conclusion, namely that each of the Cn ’s has empty interior, hoping to
arrive at a contradiction to (1.76) using the other properties. This means that an
open ball B(p, ) around a point of M (so it isn’t empty) cannot be contained in
any one of the Cn .
So, choose p ∈ C1 . Now, there must be a point p1 ∈ B(p, 1/3) which is not
in C1 . Since C1 is closed there exists 1 > 0, and we can take 1 < 1/3, such that
B(p1 , 1 ) ∩ C1 = ∅. Continue in this way, choose p2 ∈ B(p1 , 1 /3) which is not in
C2 and 2 > 0, 2 < 1 /3 such that B(p2 , 2 ) ∩ C2 = ∅. Here we use both the fact
that C2 has empty interior and the fact that it is closed. So, inductively there is a
sequence pi , i = 1, . . . , k and positive numbers 0 < k < k−1 /3 < k−2 /32 < · · · <
1 /3k−1 < 3−k such that pj ∈ B(pj−1 , j−1 /3) and B(pj , j ) ∩ Cj = ∅. Then we can
add another pk+1 by using the properties of Ck – it has non-empty interior so there is
some point in B(pk , k /3) which is not in Ck+1 and then B(pk+1 , k+1 ) ∩ Ck+1 = ∅
where k+1 > 0 but k+1 < k /3. Thus, we have a sequence {pk } in M. Since
d(pk+1 , pk ) < k /3 this is a Cauchy sequence, in fact
(1.77) d(pk , pk+l ) < k /3 + · · · + k+l−1 /3 < 3−k .
Since M is complete the sequence converges to a limit, q ∈ M. Notice however that
pl ∈ B(pk , 2k /3) for all k > l so d(pk , q) ≤ 2k /3 which implies that q ∈
/ Ck for
any k. This is the desired contradiction to (1.76).
Thus, at least one of the Cn must have non-empty interior. 

In applications one might get a complete mentric space written as a countable


union of subsets
[
(1.78) M= En , En ⊂ M
n

where the En are not necessarily closed. We can still apply Baire’s theorem however,
just take Cn = En to be the closures – then of course (1.76) holds since En ⊂ Cn .
The conclusion of course is then that
(1.79) For at least one n the closure of En has non-empty interior.

9. Uniform boundedness
One application of this is often called the uniform boundedness principle or
Banach-Steinhaus Theorem.
Theorem 3 (Uniform boundedness). Let B be a Banach space and suppose
that Tn is a sequence of bounded (i.e. continuous) linear operators Tn : B −→ V
where V is a normed space. Suppose that for each b ∈ B the set {Tn (b)} ⊂ V is
bounded (in norm of course) then supn kTn k < ∞.
Proof. This follows from a pretty direct application of Baire’s theorem to B.
Consider the sets
(1.80) Sp = {b ∈ B, kbk ≤ 1, kTn bkV ≤ p ∀ n}, p ∈ N.
Each Sp is closed because Tn is continuous, so if bk → b is a convergent sequence
then kbk ≤ 1 and kTn (p)k ≤ p. The union of the Sp is the whole of the closed ball
10. OPEN MAPPING THEOREM 25

of radius one around the origin in B :


[
(1.81) {b ∈ B; d(b, 0) ≤ 1} = Sp
p

because of the assumption of ‘pointwise boundedness’ – each b with kbk ≤ 1 must


be in one of the Sp ’s.
So, by Baire’s theorem one of the sets Sp has non-empty interior, it therefore
contains a closed ball of positive radius around some point. Thus for some p, some
v ∈ Sp , and some δ > 0,
(1.82) w ∈ B, kwkB ≤ δ =⇒ kTn (v + w)kV ≤ p ∀ n.
Since v ∈ Sp is fixed it follows that kTn wk ≤ kTn vk+p ≤ 2p for all w with kwk] ≤ δ.
Moving v to (1 − δ/2)v and halving δ as necessary it follows that this ball
B(v, δ) is contained in the open ball around the origin of radius 1. Thus, using the
triangle inequality, and the fact that kTn (v)kV ≤ p this implies
(1.83) w ∈ B, kwkB ≤ δ =⇒ kTn (w)kV ≤ 2p =⇒ kTn k ≤ 2p/δ.
1
The norm of the operator is sup{kT wkV ; kwkB = 1} = δ sup{kT wkV ; kwkB = δ}
so the norms are uniformly bounded:
(1.84) kTn k ≤ 2p/δ
as claimed. 

10. Open mapping theorem


The second major application of Baire’s theorem is to
Theorem 4 (Open Mapping). If T : B1 −→ B2 is a bounded and surjective
linear map between two Banach spaces then T is open:
(1.85) T (O) ⊂ B2 is open if O ⊂ B1 is open.
This is ‘wrong way continuity’ and as such can be used to prove the continuity of
inverse maps as we shall see. The proof uses Baire’s theorem pretty directly, but
then another similar sort of argument is needed.
Proof. What we will try to show is that the image under T of the unit open
ball around the origin, B(0, 1) ⊂ B1 contains an open ball around the origin in B2 .
The first part, of the proof, using Baire’s theorem shows that the closure of the
image, so in B2 , has 0 as an interior point – i.e. it contains an open ball around
the origin in B2 :
(1.86) T (B(0, 1) ⊃ B(0, δ), δ > 0.
To see this we apply Baire’s theorem to the sets
(1.87) Cp = clB2 T (B(0, p))
the closure of the image of the ball in B1 of radius p. We know that
[
(1.88) B2 = T (B(0, p))
p

since that is what surjectivity means – every point is the image of something. Thus
one of the closed sets Cp has an interior point, v. Since T is surjective, v = T u for
some u ∈ B1 . The sets Cp increase with p so we can take a larger p and v is still
26 1. NORMED AND BANACH SPACES

an interior point, from which it follows that 0 = v − T u is an interior point as well.


Thus indeed
(1.89) Cp ⊃ B(0, δ)
for some δ > 0. Rescaling by p, using the linearity of T, it follows that with δ
replaced by δ/p, we get (1.86).
Having applied Baire’s thereom, consider now what (1.86) means. It follows
that each v ∈ B2 with kvk = δ is the limit of a sequence T un where kun k ≤ 1.
What we want to find is such a sequence which converges. To do so we need to
choose the sequence more carefully. Certainly we can stop somewhere along the
way and see that
δ 1
(1.90) v ∈ B2 , kvk = δ =⇒ ∃ u ∈ B1 , kuk ≤ 1, kv − T uk ≤ = kvk
2 2
where of course we could replace 2δ by any positive constant but the point is the
last inequality relative to the norm of v. Now, again scaling if we take any v 6= 0 in
B2 and apply (1.90) to v/kvk we conclude that (for C = p/δ a fixed constant)
1
(1.91) v ∈ B2 =⇒ ∃ u ∈ B1 , kuk ≤ Ckvk, kv − T uk ≤ kvk
2
where the size of u only depends on the size of v; of course this is also true for v = 0
by taking u = 0.
Using this we construct the desired better approximating sequence. Given
w ∈ B1 , choose u1 = u according to (1.91) for v = w = w1 . Thus ku1 k ≤ C,
and w2 = w1 − T u1 satisfies kw2 k ≤ 12 kwk. Now proceed by induction, supposing
that we have constructed a sequence uj , j < n, in B1 with kuj k ≤ C2−j+1 p and
kwj k ≤ 2−j+1 kwk for j ≤ n, where wj = wj−1 − T uj−1 – which we have for
n = 1. Then we can choose un , using (1.91), so kun k ≤ Ckwn k ≤ C2−n+1 kwk
and such that wn+1 = wn − T un has kwn+1 k ≤ 12 kwn k ≤ 2−n kwk to extend the
induction.
P Thus we get a sequence un which is absolutely summable in B1 , since
kun k ≤ 2Ckwk, and hence converges by the assumed completeness of B1 this
n
time. Moreover
n
X n
X
(1.92) w − T( uj ) = w1 − (wj − wj+1 ) = wn+1
j=1 j=1

so T u = w and kuk ≤ 2Ckwk.


Thus finally we have shown that each w ∈ B(0, 1) in B2 is the image of some
u ∈ B1 with kuk ≤ 2C. Thus T (B(0, 3C)) ⊃ B(0, 1). By scaling it follows that the
image of any open ball around the origin contains an open ball around the origin.
Now, the linearity of T shows that the image T (O) of any open set is open,
since if w ∈ T (O) then w = T u for some u ∈ O and hence u + B(0, ) ⊂ O for  > 0
and then w + B(0, δ) ⊂ T (O) for δ > 0 sufficiently small. 

One important corollary of this is something that seems like it should be obvi-
ous, but definitely needs the completeness to be true.
Corollary 2. If T : B1 −→ B2 is a bounded linear map between Banach
spaces which is 1-1 and onto, i.e. is a bijection, then it is a homeomorphism –
meaning its inverse, which is necessarily linear, is also bounded.
12. HAHN-BANACH THEOREM 27

Proof. The only confusing thing is the notation. Note that T −1 is used to
denote the inverse maps on sets. The inverse of T, let’s call it S : B2 −→ B1 is
certainly linear. If O ⊂ B1 is open then S −1 (O) = T (O), since to say v ∈ S −1 (O)
means S(v) ∈ O which is just v ∈ T (O), is open by the Open Mapping theorem, so
S is continuous. 

11. Closed graph theorem


For the next application you should recall that the product of two Banach
spaces, B1 × B2 , – which is just the linear space of all pairs (u, v), u ∈ B1 and
v ∈ B2 , is a Banach space with respect to the sum of the norms
(1.93) k(u, v)k = kuk1 + kvk2 .
Theorem 5 (Closed Graph). If T : B1 −→ B2 is a linear map between Banach
spaces then it is bounded if and only if its graph
(1.94) Gr(T ) = {(u, v) ∈ B1 × B2 ; v = T u}
is a closed subset of the Banach space B1 × B2 .
Proof. Suppose first that T is bounded, i.e. continuous. A sequence (un , vn ) ∈
B1 × B2 is in Gr(T ) if and only if vn = T un . So, if it converges, then un → u and
vn = T un → T v by the continuity of T, so the limit is in Gr(T ) which is therefore
closed.
Conversely, suppose the graph is closed. This means that viewed as a normed
space in its own right it is complete. Given the graph we can reconstruct the map
it comes from (whether linear or not) in a little diagram. From B1 × B2 consider
the two projections, π1 (u, v) = u and π2 (u, v) = v. Both of them are continuous
since the norm of either u or v is less than the norm in (1.93). Restricting them to
Gr(T ) ⊂ B1 × B2 gives
(1.95) Gr(T )
x FF
π1 x x FFπ2
xx FF
x FF
|xx #
B1
T / B2 .

This little diagram commutes. Indeed there are two ways to map a point (u, v) ∈
Gr(T ) to B2 , either directly, sending it to v or first sending it u ∈ B1 and then to
T u. Since v = T u these are the same.
Now, as already noted, Gr(T ) ⊂ B1 × B2 is a closed subspace, so it too is a
Banach space and π1 and π2 remain continuous when restricted to it. The map π1
is 1-1 and onto, because each u occurs as the first element of precisely one pair,
namely (u, T u) ∈ Gr(T ). Thus the Corollary above applies to π1 to show that its
inverse, S is continuous. But then T = π2 ◦ S, from the commutativity, is also
continuous proving the theorem. 

12. Hahn-Banach theorem


Now, there is always a little pressure to state and prove the Hahn-Banach
Theorem. This is about extension of functionals. Stately starkly, the basic question
is: Does a normed space have any non-trivial continuous linear functionals on it?
That is, is the dual space always non-trivial (of course there is always the zero linear
functional but that is not very amusing). We do not really encounter this problem
28 1. NORMED AND BANACH SPACES

since for a Hilbert space, or even a pre-Hilbert space, there is always the space
itself, giving continuous linear functionals through the pairing – Riesz’ Theorem
says that in the case of a Hilbert space that is all there is. I could have used the
Hahn-Banach Theorem to show that any normed space has a completion, but I
gave a more direct argument for this, which was in any case much more relevant
for the cases of L1 (R) and L2 (R) for which we wanted concrete completions.
Theorem 6 (Hahn-Banach). If M ⊂ V is a linear subspace of a normed space
and u : M −→ C is a linear map such that
(1.96) |u(t)| ≤ CktkV ∀ t ∈ M

there exists a bounded linear functional U : V −→ C with kU k ≤ C and


then
U M = u.
First, by computation, we show that we can extend any continuous linear func-
tional ‘a little bit’ without increasing the norm.
Lemma 4. Suppose M ⊂ V is a subspace of a normed linear space, x ∈/ M
and u : M −→ C is a bounded linear functional as in (1.96) then there exists
u0 : M 0 = {t0 ∈ V ; t0 = t + ax}, a ∈ C such that
u0 M = u, |u0 (t + ax)| ≤ Ckt + axkV , ∀ t ∈ M, a ∈ C.

(1.97)
Proof. Note that the decompositon t0 = t + ax of a point in M 0 is unique,
since t + ax = t̃ + ãx implies (a − ã)x ∈ M so a = ã, since x ∈
/ M and hence t = t̃
as well. Thus
(1.98) u0 (t + ax) = u0 (t) + au(x) = u(t) + λa, λ = u0 (x)
and all we have at our disposal is the choice of λ. Any choice will give a linear
functional extending u, the problem of course is to arrange the continuity estimate
without increasing the constant. In fact if C = 0 then u = 0 and we can take
the zero extension. So we might as well assume that C = 1 since dividing u by C
arranges this and if u0 extends u/C then Cu0 extends u and the norm estimate in
(1.97) follows. So we are assuming that
(1.99) |u(t)| ≤ ktkV ∀ t ∈ M.
We want to choose λ so that
(1.100) |u(t) + aλ| ≤ kt + axkV ∀ t ∈ M, a ∈ C.
Certainly when a = 0 this represents no restriction on λ. For a 6= 0 we can divide
through by a and (1.100) becomes
t t
(1.101) |a||u( ) − λ| = |u(t) + aλ| ≤ kt + axkV = |a|k − xkV
a a
and since t/a ∈ M we only need to arrange that
(1.102) |u(t) − λ| ≤ kt − xkV ∀ u ∈ M
and the general case follows.
So, we will choose λ to be real. A complex linear functional such as u can be
recovered from its real part, so set
(1.103) w(t) = Re(u(t)) ∀ t ∈ M
12. HAHN-BANACH THEOREM 29

and just try to extend w to a real functional – it is not linear over the complex
numbers of course, just over the reals, but what we want is the anaogue of (1.102):
(1.104) |w(t) − λ| ≤ kt − xkV ∀ t ∈ M
which does not involve linearity. What we know about w is the norm estimate
(1.99) which implies
(1.105) |w(t1 ) − w(t2 )| ≤ |u(t1 ) − u(t2 )| ≤ kt1 − t2 k ≤ kt1 − xkV + kt2 − xkV .
Writing this out usual the reality we find
w(t1 ) − w(t2 ) ≤ kt1 − xkV + kt2 − xkV =⇒
(1.106)
w(t1 ) − kt1 − xk ≤ w(t2 ) + kt2 − xkV ∀ t1 , t2 ∈ M.
We can then take the sup on the right and the inf on the left and choose λ in
between – namely we have shown that there exists λ ∈ R with
(1.107) w(t) − kt − xkV ≤ sup (w(t1 ) − kt1 − xk) ≤ λ
t2 ∈M
≤ inf (w(t1 ) + kt1 − xk) ≤ w(t) + kt − xkV ∀ t ∈ M.
t2 ∈M

This in turn implies that


(1.108) −kt − xkV ≤ −w(t) + λ ≤ kt − xkV =⇒ |w(t)λ| ≤ −kt − xkV ∀ t ∈ M.
This is what we wanted – we have extended the real part of u to
(1.109) w0 (t + ax) = w(t) − (Re a)λ and |w0 (t + ax)| ≤ kt + axkV .
Now, finally we get the extension of u itself by ‘complexifying’ – defining
(1.110) u0 (t + ax) = w0 (t + ax) − iw0 (i(t + ax)).
This is linear over the complex numbers since
(1.111) u0 (z(t + ax)) = w0 (z(t + ax)) − iw0 (iz(t + ax)
= w0 (Re z(t + ax) + i Im z(t + ax)) − iw0 (i Re z(t + ax)) + iw0 (Im z(t + ax))
= (Re z + i Im z)w0 (t + ax) − i(Re z + i Im z)(w0 (i(t + ax)) = zu0 (t + ax).
It certainly extends u from M – since the same identity gives u in terms of its real
part w.
Finally then, to see the norm estimate note that (as we did long ago) there
exists a uniqe θ ∈ [0, 2π) such that
|u0 (t + ax)| = Re eiθ u0 (t + ax) = Re u0 (eiθ t + eiθ ax)
(1.112)
= w0 (eiθ u + eiθ ax) ≤ keiθ (t + ax)kV = kt + axkV .
This completes the proof of the Lemma. 
Of Hahn-Banach. This is an application of Zorn’s Lemma. I am not going
to get into the derivation of Zorn’s Lemma from the Axiom of Choice, but if you
believe the latter – and you are advised to do so, at least before lunchtime – you
should believe the former.
So, Zorn’s Lemma is a statement about partially ordered sets. A partial order
on a set E is a subset of E × E, so a relation, where the condition that (e, f ) be in
the relation is written e ≺ f and it must satisfy
(1.113) e ≺ e, e ≺ f and f ≺ e =⇒ e = f, e ≺ f and f ≺ g =⇒ e ≺ g.
30 1. NORMED AND BANACH SPACES

So, the missing ingredient between this and an order is that two elements need not
be related at all, either way.
A subsets of a partially ordered set inherits the partial order and such a subset
is said to be a chain if each pair of its elements is related one way or the other.
An upper bound on a subset D ⊂ E is an element e ∈ E such that d ≺ e for all
d ∈ D. A maximal element of E is one which is not majorized, that is e ≺ f, f ∈ E,
implies e = f.
Lemma 5 (Zorn). If every chain in a (non-empty) partially ordered set has an
upper bound then the set contains at least one maximal element.
Now, we are given a functional u : M −→ C defined on some linear subspace
M ⊂ V of a normed space where u is bounded with respect to the induced norm on
M. We apply this to the set E consisting of all extensions
(v, N ) of u with the same
norm. That is, V ⊃ N ⊃ M must contain M, v M = u and kvkN = kukM . This is
certainly non-empty since it contains (u, M ) and has the natural partial order that
(v1 , N1 ) ≺ (v2 , N2 ) if N1 ⊂ N2 and v2 N = v1 . You can check that this is a partial
1
order.
Let C be a chain in this set of extensions. Thus for any two elements (vi , N1 ) ∈
C, either (v1 , N1 ) ≺ (v2 , N2 ) or the other way around. This means that
[
(1.114) Ñ = {N ; (v, N ) ∈ C for some v} ⊂ V
is a linear space. Note that this union need not be countable, or anything like that,
but any two elements of Ñ are each in one of the N ’s and one of these must be
contained in the other by the chain condition. Thus each pair of elements of Ñ is
actually in a common N and hence so is their linear span. Similarly we can define
an extension
(1.115) ṽ : Ñ −→ C, ṽ(x) = v(x) if x ∈ N, (v, N ) ∈ C.
There may be many pairs (v, N ) satisfying x ∈ N for a given x but the chain
condition implies that v(x) is the same for all of them. Thus ṽ is well defined, and
is clearly also linear, extends u and satisfies the norm condition |ṽ(x)| ≤ kukM kvkV .
Thus (ṽ, Ñ ) is an upper bound for the chain C.
So, the set of all extension E, with the norm condition, satisfies the hypothesis
of Zorn’s Lemma, so must – at least in the mornings – have an maximal element
(ũ, M̃ ). If M̃ = V then we are done. However, in the contary case there exists
x ∈ V \ M̃ . This means we can apply our little lemma and construct an extension
(u0 , M̃ 0 ) of (ũ, M̃ ) which is therefore also an element of E and satisfies (ũ, M̃ ) ≺
(u0 , M̃ 0 ). This however contradicts the condition that (ũ, M̃ ) be maximal, so is
forbidden by Zorn. 

There are many applications of the Hahn-Banach Theorem, one significant one
is that the dual space of a non-trivial normed space is itself non-trivial.
Proposition 7. For any normed space V and element 0 6= v ∈ V there is a
continuous linear functional f : V −→ C with f (v) = 1 and kf k = 1/kvkV .
Proof. Start with the one-dimensional space, M, spanned by x and define
u(zx) = z. This has norm 1/kxkV . Extend it using the Hahn-Banach Theorem and
you will get a continuous functional f as desired. 
14. PROBLEMS 31

13. Double dual


Let me give another application of the Hahn-Banach theorem, although I have
never covered this in lectures. If V is a normed space, we know its dual space, V 0 ,
to be a Banach space. Let V 00 = (V 0 )0 be the dual of the dual.
Proposition 8. If v ∈ V then the linear map on V 0 :
(1.116) Tv : V 0 −→ C, Tv (v 0 ) = v 0 (v)
is continuous, and so defines an isometric linear injection V ,→ V 00 , kTv k = kvk.
Proof. The definition of Tv is ‘tautologous’, meaning it is almost the definition
of V 0 . First check Tv in (1.116) is linear. Indeed, if v10 , v20 ∈ V 0 and λ1 , λ2 ∈ C then
Tv (λ1 v10 + λ2 v20 ) = (λ1 v10 + λ2 v20 )(v) = λ1 v10 (v) + λ2 v20 (v) = λ1 Tv (v10 ) + λ2 Tv (v20 ).
That Tv ∈ V 00 , i.e. is bounded, follows too since |Tv (v 0 )| = |v 0 (v)| ≤ kv 0 kV 0 kvkV ;
this also shows that kTv kV 00 ≤ kvk. On the other hand, by Proposition 7 above,
if kvk = 1 then there exists v 0 ∈ V 0 such that v 0 (v) = 1 and kv 0 kV 0 = 1. Then
Tv (v 0 ) = v 0 (v) = 1 shows that kTv k = 1 so in general kTv k = kvk. It also needs
to be checked that V 3 v 7−→ Tv ∈ V 00 is a linear map – this is clear from the
definition. It is necessarily 1-1 since kTv k = kvk. 
Now, it is definitely not the case in general that V 00 = V in the sense that this
injection is also a surjection. Since V 00 is always a Banach space, one necessary
condition is that V itself should be a Banach space. In fact the closure of the image
of V in V 00 is a completion of V. If the map to V 00 is a bijection then V is said
to be reflexive. It is pretty easy to find examples of non-reflexive Banach spaces,
the most familiar is c0 – the space of infinite sequences converging to 0. Its dual
can be identified with l1 , the space of summable sequences. Its dual in turn, the
bidual of c0 , is the space l∞ of bounded sequences, into which the embedding is the
obvious one, so c0 is not reflexive. In fact l1 is not reflexive either. There are useful
characterizations of reflexive Banach spaces. You may be interested enough to look
up James’ Theorem:- A Banach space is reflexive if and only if every continuous
linear functional on it attains its supremum on the unit ball.

14. Problems
Problem 1.1. Show that if V is a vector space (over R or C) then for any set
X the space
(1.117) F(X; V ) = {u : X −→ V }
is a linear space over the same field, with ‘pointwise operations’.
Problem 1.2. If V is a vector space and S ⊂ V is a subset which is closed
under addition and scalar multiplication:
(1.118) v1 , v2 ∈ S, λ ∈ K =⇒ v1 + v2 ∈ S and λv1 ∈ S
then S is a vector space as well (called of course a subspace).
Problem 1.3. If S ⊂ V be a linear subspace of a vector space show that the
relation on V
(1.119) v1 ∼ v2 ⇐⇒ v1 − v2 ∈ S
is an equivalence relation and that the set of equivalence classes, denoted usually
V /S, is a vector space in a natural way.
32 1. NORMED AND BANACH SPACES

Problem 1.4. In case you do not know it, go through the basic theory of
finite-dimensional vector spaces. Define a vector space V to be finite-dimensional
if there is an integer N such that any N elements of V are linearly dependent – if
vi ∈ V for i = 1, . . . N, then there exist ai ∈ K, not all zero, such that
N
X
(1.120) ai vi = 0 in V.
i=1

Call the smallest such integer the dimension of V and show that a finite dimensional
vector space always has a basis, ei ∈ V, i = 1, . . . , dim V which are not linearly
dependent and such that any element of V can be written as a linear combination
dim
XV
(1.121) v= bi ei , bi ∈ K.
i=1

Problem 1.5. Show that any two norms on a finite dimensional vector space
are equivalent.
Problem 1.6. Show that if two norms on a vector space are equivalent then
the topologies induced are the same – the sets open with respect to the distance
from one are open with respect to the distance coming from the other. The converse
is also true, you can use another result from this section to prove it.
Problem 1.7. Write out a proof (you can steal it from one of many places but
at least write it out in your own hand) either for p = 2 or for each p with 1 ≤ p < ∞
that
X∞
lp = {a : N −→ C; |aj |p < ∞, aj = a(j)}
j=1
is a normed space with the norm
  p1

X
kakp =  |aj |p  .
j=1

This means writing out the proof that this is a linear space and that the three
conditions required of a norm hold.
Problem 1.8. The ‘tricky’ part in Problem 1.1 is the triangle inequality. Sup-
pose you knew – meaning I tell you – that for each N
  p1
XN
 |aj |p  is a norm on CN
j=1

would that help?


Problem 1.9. Prove directly that each lp as defined in Problem 1.1 is complete,
i.e. it is a Banach space. At the risk of offending some, let me say that this means
showing that each Cauchy sequence converges. The problem here is to find the limit
of a given Cauchy sequence. Show that for each N the sequence in CN obtained by
truncating each of the elements at point N is Cauchy with respect to the norm in
Problem 1.2 on CN . Show that this is the same as being Cauchy in CN in the usual
sense (if you are doing p = 2 it is already the usual sense) and hence, this cut-off
15. SOLUTIONS TO PROBLEMS 33

sequence converges. Use this to find a putative limit of the Cauchy sequence and
then check that it works.
Problem 1.10. The space l∞ consists of the bounded sequences
(1.122) l∞ = {a : N −→ C; sup |an | < ∞}, kak∞ = sup |an |.
n n
Show that it is a Banach space.
Problem 1.11. Another closely related space consists of the sequences con-
verging to 0 :
(1.123) c0 = {a : N −→ C; lim an = 0}, kak∞ = sup |an |.
n→∞ n

Check that this is a Banach space and that it is a closed subspace of l∞ (perhaps
in the opposite order).
Problem 1.12. Consider the ‘unit sphere’ in lp . This is the set of vectors of
length 1 :
S = {a ∈ lp ; kakp = 1}.
(1) Show that S is closed.
(2) Recall the sequential (so not the open covering definition) characterization
of compactness of a set in a metric space (e .g . by checking in Rudin).
(3) Show that S is not compact by considering the sequence in lp with kth
element the sequence which is all zeros except for a 1 in the kth slot. Note
that the main problem is not to get yourself confused about sequences of
sequences!
Problem 1.13. Show that the norm on any normed space is continuous.
Problem 1.14. Finish the proof of the completeness of the space B constructed
in DD.

15. Solutions to problems


Solution 1.1 (1.1). If V is a vector space (over K which is R or C) then for
any set X consider
(1.124) F(X; V ) = {u : X −→ V }.
Addition and scalar multiplication are defined ‘pointwise’:
(1.125) (u + v)(x) = u(x) + v(x), (cu)(x) = cu(x), u, v ∈ F(X; V ), c ∈ K.
These are well-defined functions since addition and multiplication are defined in K.
So, one needs to check all the axioms of a vector space. Since an equality
of functions is just equality at all points, these all follow from the corresponding
identities for K.
Solution 1.2 (1.2). If S ⊂ V is a (non-empty) subset of a vector space and
S ⊂ V which is closed under addition and scalar multiplication:
(1.126) v1 , v2 ∈ S, λ ∈ K =⇒ v1 + v2 ∈ S and λv1 ∈ S
then 0 ∈ S, since 0 ∈ K and for any v ∈ S, 0v = 0 ∈ S. Similarly, if v ∈ S
then −v = (−1)v ∈ S. Then all the axioms of a vector space follow from the
corresponding identities in V.
34 1. NORMED AND BANACH SPACES

Solution 1.3. If S ⊂ V be a linear subspace of a vector space consider the


relation on V
(1.127) v1 ∼ v2 ⇐⇒ v1 − v2 ∈ S.
To say that this is an equivalence relation means that symmetry and transitivity
hold. Since S is a subspace, v ∈ S implies −v ∈ S so
v1 ∼ v2 =⇒ v1 − v2 ∈ S =⇒ v2 − v1 ∈ S =⇒ v2 ∼ v1 .
Similarly, since it is also possible to add and remain in S
v1 ∼ v2 , v2 ∼ v3 =⇒ v1 − v2 , v2 − v3 ∈ S =⇒ v1 − v3 ∈ S =⇒ v1 ∼ v3 .
So this is an equivalence relation and the quotient V / ∼= V /S is well-defined –
where the latter is notation. That is, and element of V /S is an equivalence class of
elements of V which can be written v + S :
(1.128) v + S = w + S ⇐⇒ v − w ∈ S.
Now, we can check the axioms of a linear space once we define addition and scalar
multiplication. Notice that
(v + S) + (w + S) = (v + w) + S, λ(v + S) = λv + S
are well-defined elements, independent of the choice of representatives, since adding
an lement of S to v or w does not change the right sides.
Now, to the axioms. These amount to showing that S is a zero element for
addition, −v + S is the additive inverse of v + S and that the other axioms follow
directly from the fact that the hold as identities in V.

Solution 1.4 (1.4). In case you do not know it, go through the basic theory of
finite-dimensional vector spaces. Define a vector space V to be finite-dimensional
if there is an integer N such that any N elements of V are linearly dependent – if
vi ∈ V for i = 1, . . . N, then there exist ai ∈ K, not all zero, such that
N
X
(1.129) ai vi = 0 in V.
i=1

Call the smallest such integer the dimension of V and show that a finite dimensional
vector space always has a basis, ei ∈ V, i = 1, . . . , dim V which are not linearly
dependent and such that any element of V can be written as a linear combination
dim
XV
(1.130) v= bi ei , bi ∈ K.
i=1

Solution 1.5 (1.5). Show that any two norms on a finite dimensional vector
space are equivalent.

Solution 1.6 (1.6). Show that if two norms on a vector space are equivalent
then the topologies induced are the same – the sets open with respect to the distance
from one are open with respect to the distance coming from the other. The converse
is also true, you can use another result from this section to prove it.
15. SOLUTIONS TO PROBLEMS 35

Solution 1.7 (1.7). Write out a proof (you can steal it from one of many
places but at least write it out in your own hand) either for p = 2 or for each p
with 1 ≤ p < ∞ that
X∞
lp = {a : N −→ C; |aj |p < ∞, aj = a(j)}
j=1

is a normed space with the norm


  p1
X∞
kakp =  |aj |p  .
j=1

This means writing out the proof that this is a linear space and that the three
conditions required of a norm hold.
Solution 1.8 (1.8). The ‘tricky’ part in Problem 1.1 is the triangle inequality.
Suppose you knew – meaning I tell you – that for each N
  p1
XN
 |aj |p  is a norm on CN
j=1

would that help?


Solution 1.9 (1.9). Prove directly that each lp as defined in Problem 1.1 is
complete, i.e. it is a Banach space. At the risk of offending some, let me say that
this means showing that each Cauchy sequence converges. The problem here is to
find the limit of a given Cauchy sequence. Show that for each N the sequence in
CN obtained by truncating each of the elements at point N is Cauchy with respect
to the norm in Problem 1.2 on CN . Show that this is the same as being Cauchy
in CN in the usual sense (if you are doing p = 2 it is already the usual sense)
and hence, this cut-off sequence converges. Use this to find a putative limit of the
Cauchy sequence and then check that it works.
Solution 1.10 (1.10). The space l∞ consists of the bounded sequences
(1.131) l∞ = {a : N −→ C; sup |an | < ∞}, kak∞ = sup |an |.
n n
Show that it is a Banach space.
Solution 1.11 (1.11). Another closely related space consists of the sequences
converging to 0 :
(1.132) c0 = {a : N −→ C; lim an = 0}, kak∞ = sup |an |.
n→∞ n

Check that this is a Banach space and that it is a closed subspace of l∞ (perhaps
in the opposite order).
Solution 1.12 (1.12). Consider the ‘unit sphere’ in lp . This is the set of vectors
of length 1 :
S = {a ∈ lp ; kakp = 1}.
(1) Show that S is closed.
(2) Recall the sequential (so not the open covering definition) characterization
of compactness of a set in a metric space (e .g . by checking in Rudin).
36 1. NORMED AND BANACH SPACES

(3) Show that S is not compact by considering the sequence in lp with kth
element the sequence which is all zeros except for a 1 in the kth slot. Note
that the main problem is not to get yourself confused about sequences of
sequences!
Solution 1.13 (1.13). Since the distance between two points is kx − yk the
continuity of the norm follows directly from the ‘reverse triangle inequality’
(1.133) |kxk − kyk| ≤ kx − yk
which in turn follows from the triangle inequality applied twice:-
(1.134) kxk ≤ kx − yk + kyk, kyk ≤ kx − yk + kxk.
Solution 1.14 (1.14). Finish the proof of the completeness of the space B
constructed in DD.
CHAPTER 2

The Lebesgue integral

This semester, ‘Spring’ 2011, I have decided to circumvent the discussion of


step functions below and to proceed directly by completing the Riemann integral.
For the moment I will leave the old material in – I hope the parallel development is
not too confusing. The advantage of this change is (I hope) that the treatment is
a little simpler and faster. The disadvantages are first that I will rely on an earlier
treatement of the Riemann integral (which is covered in 18.100) – so this is not
much of an issue – but slightly more seriously the extension to higher dimensions
becomes a little more involved.
Maybe there are other issues too, we will find out. The new sections have a
2011 in the title.
Things I could possibly add.
(1) Higher dimension more systematically.
(2) Fubini’s theorem
(3) Other measures on the line
(4) Riesz representation on the line – in Problems.
(5) The Banach space L∞ (R).
(6) Duality between Lp (R) and Lq (R), 1/p + 1/q = 1, from forward reference
to Riesz’ representation for Hilbert space.
The treatment of the Lebesgue integral here is intentionally compressed. In
the text everything is done for R but in such a way that the extension to higher
dimensions – carried out in the problems – is not much harder. Some further
extensions are discussed in the problems.
Here is a narrative for a later reading:- If you can go through this item by item,
reconstruct the definitions and results as you go and see how thing fit together then
you are doing well!
• Intervals and length.
• Covering lemma.
• Step functions and the integral.
• Monotonicity lemma.
• L1 (R) and absolutely summable approximation.
1
• L
R (R)1 is a linear space.
• : L (R) −→ C is well defined.
• If f ∈ L1 (R) then |f | ∈ L1 (R) and
Z Z X n Z n
X
(2.1) |f | = lim | fj |, lim |f − fj | = 0
n→∞ n→∞
j=1 j=1

for any absolutely summable approximation.


• Sets of measure zero.
37
38 2. THE LEBESGUE INTEGRAL

• Convergence a.e.
• If {gj } in L1 (R) is absolutely summable then
X
g= gj a.e. =⇒ g ∈ L1 (R),
j
X
{x ∈ R; |gj (x)| = ∞} is of measure zero
(2.2) j
Z XZ Z Z n
X Z n
X
g= gj , |g| = lim | gj |, lim |g − gj | = 0.
n→∞ n→∞
j j=1 j=1

• The space of null functions N = {f ∈ L1 (R); |f | = 0} consists precisely


R

of the functions vanishing almost everywhere, N = {f : R −→ C; f =


0 a.e.}.
• L1 (R) = L1 (R)/N is a Banach space with L1 norm.
• Montonicity for Lebesgue functions.
• Fatou’s Lemma.
• Dominated convergence.
• The Banach spaces Lp (R) = Lp (R)/N , 1 ≤ p < ∞.
• Measurable sets.

1. Continuous functions of compact support (2011)


Recall that the Riemann integral is defined for a continuous function u :
[a, b] −→ C. It depends on the compactness of the interval but can be extended
to an ‘improper integral’ (meaning some of the good properties fail) on certain
functions on the whole line. This is NOT what we will do. Rather we consider the
space of continuous functions ‘with compact support’:
(2.3)
Cc (R) = {u : R −→ C; u is continuous and ∃ R such that u(x) = 0 if |x| > R}.
Thus each element u ∈ Cc (R) vanishes outside an interval [−R, R] where the R
depends on the u.
Lemma 6. The Riemann integral defines a continuous linear functional on the
normed space Cc (R), with respect to the L1 norm
Z Z
u = lim u(x)dx,
R R→∞ [−R,R]
Z
(2.4) kukL1 = lim |u(x)|dx,
R→∞ [−R,R]
Z
| u| ≤ kukL1 .
R

The limits here are trivial in the sense that the functions involved are constant for
large R.

Proof. Basic properties of the Riemann integral. 

With this preamble we can directly define the ‘space’ of Lebesgue integrable
functions on R.
1. CONTINUOUS FUNCTIONS OF COMPACT SUPPORT (2011) 39

Definition 5. A function f : R −→ C is Lebesgue integrable, written f ∈


L1 (R), if there exists a sequence fn ∈ Cc (R) which is absolutely summable,
XZ
(2.5) |fn | < ∞
n

and such that


X X
(2.6) |fn (x)| < ∞ =⇒ fn (x) = f (x).
n n

This is a somewhat convoluted definition. Its virtue is that it is all there. The
problem is that it takes a bit of unravelling. Notice that we only say ‘there exists’
an absolutely summable sequence and that it is required to converge to the function
only at points at which the pointwise sequence is absolutely summable. At other
points anything is permitted.
It is not immediately obvious that this is a linear space, nor is it obvious that
the integral of such a function is defined. We want to define it to be
Z XZ
(2.7) f= fn
R n

which is fine in so far as the series on the left (of complex numbers) does converge –
since we demanded that it converge absolutely – but not fine in so far as the answer
might well depend on which sequence we choose which ‘converges to f ’ in the sense
of the definition.
So, the immediate problem is to prove these two things. First however, observe

Lemma 7. If f ∈ L1 (R) the real and imaginary parts, Re f, Im f ∈ L1 (R) and


for a real function the approximating sequence can be chosen to be real.

Proof. To prove this (it is proved below in the older version) we need to
‘take an approximating sequence apart’. So, suppose fn ∈ Cc (R) is a sequence the
existence of which is guaranteed by the definition above. Then consider gn = Re fn ,
hn = Im fn . These are both elements of Cc (R). Now, consider the doctored sequence

gn
 if k = 3n − 2
(2.8) uk = hn if k = 3n − 1

−hn if k = 3n.

This defines uk ∈ Cc (R) for all k ∈ N. Furthermore, it is an absolutely summable


series since
XZ XZ
(2.9) |uk | ≤ 3 |fn |,
k n

using the factPthat |gn | ≤ |fn | and |hn | ≤ |f


Pn |. P
When is |uk (x)| < ∞? Only if both |gn (x)| < ∞ and |hn (x)| < ∞. This
k n n
is in fact precisely the reason that we go to theP
trouble of inserting the otherwise
useless ±hn in (2.8). Together these imply that |fn (x)| < ∞ and the converse is
n
40 2. THE LEBESGUE INTEGRAL

also true! Thus


X X
(2.10) |uk (x)| < ∞ =⇒ |fn (x)| < ∞
k n
X X
=⇒ fn (x) = f (x) =⇒ gn (x) = Re f (x)
n n
X
=⇒ uk (x) = Re f (x).
k

The last step follows from |hn (x)| → 0 since we are successively adding and then
subtracting hn (x) and by the absolute convergence of the sum, hn (x) → 0. So
indeed, Re f ∈ L1 (R) and it has a real approximating sequence. Reversing the
roles of gn and hn gives the Lebesgue integrability of Im f. 

Next we want to show that the integral is well defined. To do so, we can
now just think about real functions. I also need to recall another theorem from
18.100B. This describes a case where uniform convergence follows from pointwise
convergence. It works on general compact metric space but for the moment I will
just think about the case at hand.

Lemma 8. If un ∈ Cc (R) is a decreasing sequence of non-negative functions


such that limn→∞ un (x) = 0 for each x ∈ R then un → 0 uniformly on R and
Z
(2.11) lim un = 0.
n→∞

Proof. Since all the un (x) ≥ 0 and they are decreasing (which means non-
increasing) any where u1 (x) = 0 all the other un (x) = 0 as well. Thus there is
one R > 0 such that un (x) = 0 if |x| > R for all n. Thus in fact we only need
consider what happens on [−R, R] which is compact. For any  > 0 look at the sets
Sn = {x ∈ [−R, R]; un (x) ≥ }. This can also be written Sn = u−1 n ([, ∞))∩[−R, R]
and since un is continuous this is closed and hence compact. MoreoverT the fact that
the un (x) are decreasing means that Sn+1 ⊂ Sn for all n. Finally, n Sn = ∅ since,
by assumption, un (x) → 0 for each x. Now the property of compact sets in a metric
space that we use is that if such a sequence of decreasing compact sets has empty
intersection then the sets themselves are empty from some n onwards. This means
that there exists N such that supx un (x) <  for all n > N. Since  > 0 was
arbitrary, un → 0 uniformly.
One of the basic properties of the Riemann integral is that the integral of the
limit of a uniformly convergent sequence (even of Riemann integrable functions but
here just continuous) is the limit of the sequence of integrals, which is (2.11) in this
case. 

We can easily extend this in a useful way.

Lemma 9. If vn ∈ Cc (R) is an increasing sequence such that limn→∞ vn (x) ≥ 0


for each x ∈ R (where the possibility vn (x) → ∞ is included) then
Z
(2.12) lim vn dx ≥ 0 including possibly + ∞.
n→∞
1. CONTINUOUS FUNCTIONS OF COMPACT SUPPORT (2011) 41

Proof. This is really a corollary of the preceding lemma. Consider the se-
quence of function
(
0 if vn (x) ≥ 0
(2.13) wn (x) =
−vn (x) if vn (x) < 0.

Since this is the maximum of two continuous functions, namely −vn and 0, it is
continuous and it vanishes for large x, so wn ∈ Cc (R). Since vn (x) is increasing,
wn is decreasing and it follows that lim wn (x) = 0 for all x, either it gets there for
some finite x and then stays 0 or the limit of vn (x) is zero. Thus Lemma 8 applies
to wn so
Z
lim wn (x)dx = 0.
n→∞ R
R R
Now, vn (x) ≥ −wn (x) for all x, so for each Rn, vn ≥ − wn . From properties of
the Riemann integral, vn+1 ≥ vn implies that vn dx is an increasing sequence and
it is bounded below by one that converges to 0, so (2.12) is the only possibility. 

Lemma 10. SupposeP R fn ∈ Cc (R) is an absolutely summable sequence of real-


valued functions, so |fn |dx < ∞, and
n
X X
(2.14) |fn (x)| < ∞ =⇒ fn (x) = 0
n n

then
XZ
(2.15) fn dx = 0.
n

R Proof.RAs already noted, the series (2.15) does converge, since the inequality
| fn |dx ≤ |fn |dx shows that it is absolutely convergent (hence Cauchy, hence
convergent).
The trick is to get ourselves into a position to apply Lemma 9. To do this, just
choose some integer N (large but it doesn’t matter yet). Now consider the sequence
of functions – it depends on N but I will suppress this dependence –
N
X +1
(2.16) F1 (x) = fn (x), Fj (x) = |fN +j (x)|, j ≥ 2.
n=1

P Rthis is a sequence in Cc (R) and it is absolutely summable – the convergence


Now,
of |Fj |dx only depends on the ‘tail’ which is the same as for fn . For the same
j
reason,
X X
(2.17) |Fj (x)| < ∞ ⇐⇒ |fn (x)| < ∞.
j n

Now the sequence of partial sums


p
X N
X +1 p
X
(2.18) gp (x) = Fj (x) = fn (x) + |fN +j |
j=1 n=1 j=2
42 2. THE LEBESGUE INTEGRAL

is increasing with p – since we are adding non-negative functions. If the two equiv-
alent conditions in (2.17) hold then
X N
X +1 ∞
X
(2.19) fn (x) = 0 =⇒ fn (x) + |fN +j (x)| ≥ 0 =⇒ lim gp (x) ≥ 0,
p→∞
n n=1 j=2

since we are only increasing each term. On the other hand if these conditions do
not hold then the tail, any tail, sums to infinity so
(2.20) lim gp (x) = ∞.
p→∞

Thus the conditions of Lemma 9 hold for gp and hence


N
X +1 Z X Z
(2.21) fn + |fj (x)|dx ≥ 0.
n=1 j≥N +2

Using the same inequality as before this implies that


X∞ Z X Z
(2.22) fn ≥ −2 |fj (x)|dx.
n=1 j≥N +2
P R
This is true for any N and as N → ∞, limN →∞ |fj (x)|dx = 0. So
j≥N +2
the fixed number on the left in (2.22), which is what we are interested in, must be
non-negative. In fact the signs in the argument can be reversed, considering instead
N
X +1
(2.23) h1 (x) = − fn (x), hp (x) = |fN +p (x)|, p ≥ 2
n=1

and the final conclusion is the opposite inequality in (2.22). That is, we conclude
what we wanted to show, that
∞ Z
X
(2.24) fn = 0.
n=1

Definition 6. A set E P ⊂ RR is of measure zero if there exists an absolutely


summable series hn ∈ Cc (R), |hn | < ∞ such that
n
X
(2.25) E ⊂ {x ∈ R; |hn (x)| = ∞}.
n

Lemma 11.PIfR f : R −→ C and there exists an absolutely summable series


gn ∈ Cc (R), so |gn | < ∞, such that
n
X
(2.26) f (x) = gn (x) on R \ E
n

for some set of measure zero, E, then f ∈ L1 (R).


So this shows that a slightly weaker definition gives the same result.
1. CONTINUOUS FUNCTIONS OF COMPACT SUPPORT (2011) 43

Proof. Let hn be an absolutely summable series as in (2.25) and consider the


sequence

gn
 if k = 3n − 2
(2.27) fk = hn if k = 3n − 1

−hn if k = 3n.

Then, as in the arguments above, {fk } is absolutely summable and


(2.28)
X X X X
|fk (x)| < ∞ =⇒ |gn (x)| < ∞ and |hn (x)| < ∞ =⇒ x ∈ R\E and f (x) = fk (x)
k n n k

so indeed f ∈ L1 (R). 

This slight weakening of the definition makes the arguments a little less rigid.
We define the notion of equality almost everywhere to mean
f (x) = g(x) on R \ E for some set of measure zero E
(2.29)
⇐⇒ f = g a.e.
where the second line is notation.

Proposition 9. The (Lebesgue) integral of an element f ∈ L1 (R) is well-


defined as
Z XZ
(2.30) f= fn
n
P
for any absolutely summable series fn ∈ Cc (R) such that f = fn a.e. and then
n
Z XZ
(2.31) | f| ≤ |fn |.
n

Proof. This follows from LemmaP10. First, we can weaken the hypothesis
of the lemma to the assumption that fn (x) = 0 a.e. by using the argument in
n
Lemma 11. Then if gn and gn0 are two absolutely summable series in Cc (R) both
converging to f almost everywhere, the sequence with alternating terms gn , −gn0 is
absolutely summable and converges to 0 a.e. so the strengthened Lemma 10 shows
that
XZ XZ
(2.32) gn = gn0
n n

by rearrangement of an absolutely convergent series in C. Thus the limit in (2.30)


is indeed independent of approximating series. The last part, (2.31), follows from
(2.30). 

Now, (in Spring 2011), you can go to Section 10 and proceed, except for the
moment you will have to replace the phrase ‘model functions’ by ‘continuous func-
tion of compact support’ (and there maybe be some backward references that I will
need to fix.
44 2. THE LEBESGUE INTEGRAL

2. Step functions
We will start our development of the Lebesgue integral in a way that is really
a simplification of the approach in Rudin’s book [2] to the Riemann integral. Since
we only deal initially with step functions the discussion is quite elementary – and
in lectures I take quite a lot for granted – until we get to the covering lemmas.
For the discussion of Lebesgue integral we will limit the term interval to mean
a semi-open set [a, b) – an interval closed on the left and open on the right. There
is nothing magical about these – we could certainly use the opposite convention –
but it saves a lot of effort to work with them. The basic property they have is that
we can decompose such an interval into finitely many subintervals without overlap.
This of course is not possible for open or closed intervals and corresponds to the
‘partitions’ in Riemann theory. In fact the finite unions of such intervals, which are
the sets we are working with, form an ‘algebra’ of sets – closed under finite unions,
finite intersections and differences.
In particular we define the length of the interval I = [a, b) to be len(I) = b − a.
This only vanishes when the interval is empty (not true for closed intervals of
course) and if we decompose an interval, in this sense, into two disjoint intervals
by choosing any interior point:

(2.33) [a, b) = [a, t) ∪ [t, b), a < t < b, I = I1 ∪ I2 , I1 ∩ I2 = ∅


then len(I) = len(I1 ) + len(I2 ).

Proposition 10. Any finite union of intervals S = ∪N i=1 [ai , b1 ) can be written
uniquely as a union of such intervals with disjoint closures
k
[
(2.34) S= Ik , Ik = [Ai .Bi ), Ik ∩ Ij = ∅ if k 6= j,
i=1
P
and then len(S) = len(Ik ) has the finite additivity property that
k
[ X
(2.35) S= Jp , Jp ∩ Jq = ∅ if p 6= q =⇒ len(S) = len(Ip ).
p p

In lectures I treat this as ‘obvious’. Let’s make sure that it is true!


Proof. Take the set S in (2.34) which we can assume to be non-empty and
let A1 = inf S. The closure of S is the union of the closures of the intervals, so
A1 is in the closure of one of these, but since it is (no larger than) the infimum
of the interval, it is in one of the intervals – it must be an ai . Then let B1 be the
supremum of the set {b ∈ R; [A1 , b) ⊂ S}. Then B1 > A1 and [A1 , B1 ) ⊂ S. This
can be true for no larger B1 so we have our first interval and either S = [A1 , B1 ) or
A2 = inf{a ∈ S, a ≥ B2 } exists and A2 > B1 . Now continue in this way to generate
a decomposition into disjoint and ‘maximal’ intervals. Any interval contained in
S must be contained in one of the [Ai , Bi ) which gives the uniqueness of this
decomposition. Hence the length is well defined.
Now, to check that (2.35) holds, observe that any disjoint decomposition in
intervals has to be a disjoint decomposition of each of the [Ai , Bi ) so we can assume
that S = [A, B). Now proceed (if you haven’t given up yet) to induction over the
number of intervals in the decomposition. Check the result for one and two and
then check what happens if you remove the top interval and finish. 
2. STEP FUNCTIONS 45

So the standard notion of length is a finitely additive on our sets – which are
the finite unions of semi-open intervals.
SN
Lemma 12. If S0 ⊂ i=1 Si where the Si are each a finite union of semi-open
intervals then
N
X
(2.36) len(S0 ) ≤ len(Si )
i=1
S
and if Tj j = 1, . . . , M are disjoint sets of this type such that S0 ⊃ j Tj then
X
(2.37) len(S0 ) ≥ len(Tj ).
j

Again pretty obvious, but there are sticklers for precision out there!
Proof. To understand this really properly it is perhaps wise to check at this
point something mentioned above; that our sets S – by definition each being a
(possibly empty) finite union of semi-open intervals – form a collection of subsets
of R with the closure properties that if S1 , S2 are of this form then so are
(2.38) S1 ∩ S2 , S1 ∪ S2 and S1 \ S2 .
The second is clear by definition. The first follows from the fact that the intersec-
tion of two intervals is either empty or is an interval – so the intersections of the
non-trivial intervals in the minimal decomposition of S1 and S2 gives a minimal
decomposition of S1 ∩ S2 . Similary [A, B) \ [a, b) is either empty, an interval or the
union of two intervals with disjoint closures so a similar argument applies to the
difference. Now check the properties of lengths
(2.39) len(S1 ∩ S2 ) ≤ len(S1 ), len(S1 ∪ S2 ) ≤ len(S1 ) + len(S2 ),
len(S1 ∪ S2 ) = len(S1 ) + len(S2 ) if S1 ∩ S2 = ∅,
len(S1 ) − len(S2 ) ≤ len(S1 \ S2 ) ≤ len(S1 )
from the disjoint decompositions as intervals. The general case then follows by
induction. 
Now, by a step function
(2.40) f : R −→ C
(although sometimes we will restrict to functions with real values) we mean a func-
tion which vanishes outside some finite union of disjoint intervals and is constant
on each of the intervals. This is equivalent to saying that f (R) is finite – these
function only take finitely many values – and
(2.41) f −1 (c) is a finite union of disjoint intervals, c 6= 0.
Of course f −1 (0) will be the complement in R of a finite union of intervals, which
is not a finite union of intervals in our current sense. It is also often convenient to
write a step function as a sum
N
(
X 1 if x ∈ [a, b)
(2.42) f= ci χ[ai ,bi ) , χ[a,b) (x) =
i=1
0 otherwise,
of multiples of the characteristic functions of our intervals. Note that such a ‘pre-
sentation’ is not unique but can be made so by demanding that the intervals be
46 2. THE LEBESGUE INTEGRAL

disjoint and ‘maximal’ – so f does not take the same value on two intervals with
intersecting closures.
A constant multiple of a step function is a step function and so is the sum of
two step functions – clearly the range is finite, consists of some of the elements
in the sums of the ranges and the inverse image of non-zero values is a union of
intervals.
A similar argument shows that the integral, defined by
Z X
(2.43) f= ci (bi − ai )
R i

from (2.42) is independent of the ‘presentation’ used to define it. It is of course


equal to the Riemann integral which is one way of seeing that it is well-defined (but
of course the result is much more elementary). Note that if f is a step function
then so is |f | and that
Z Z
(2.44) | f | ≤ |f |

since on any interval on which f is constant, |f | is constant too.


Proposition 11. The step functions on R in the sense defined above form a
normed space with respect to the L1 norm
Z
(2.45) kf kL1 = |f |.
R

Proof. The property that a norm only vanishes on the zero function is the
most delicate one to check, but the integral of |f | is the sum of positive terms –
the products of the lengths of the intervals and the values of the function – so can
only vanish if all these vanish, and this does imply that f ≡ 0 because any finite
union of intervals of length 0 is empty. This is one good reason to stick with our
semi-open intervals. 

3. Lebesgue integrable functions


The idea is to complete this normed space instead of the space of continuous
functions – it is both more standard and a little easier. The approach adopted here,
to the direct construction of a ‘concrete’ completion, is due to Mikusiński. Already
at this stage we can define the notion of a Lebesgue integrable function, however
this definition is rather convoluted and we need to do some work to flesh it out.
The idea is to use the notion of an ‘absolutely summable’ series which is discussed
above, in the normed space of step functions.
Definition 7. A function f : R −→ C is Lebesgue integrable if there exists an
absolutely summable sequence of step functions fn , i.e. satisfying
XZ
(2.46) |fn | < ∞,
n

such that
X X
(2.47) f (x) = fn (x) ∀ x ∈ R such that |fn (x)| < ∞.
n n

We will denote the space of such functions as L1 (R).


4. COVERING LEMMAS 47

This definition is at first glance a little weird – there must exist a sequence of
step functions, the sum of the integrals of the absolute values of which converges,
such that the sum itself converges to the function but only at the points of absolute
converge of the (pointwise) series. Tricky, but one can get used to it – and ultimately
we will simplify it. The main attraction of this definition is that it is self-contained
and in principle ‘everything’ can be deduced from it. The basic idea is that the
absolute summability in (2.46) ensures that the pointwise series in (2.47) converges
for ‘most’ points x. So (2.47) almost determines f itself and does so up to an error
which does not affect the integral.
What we need to show then is that the ‘integral’ of such a function is well-
defined by setting it equal to
Z XZ
(2.48) f= fi
i

where {fi } is any approximating sequence of step functions as in the definition.


The sum on the right converges, since it converges absolutely using (2.44) which
shows that (in the real numbers)
X Z XZ
(2.49) | fi | ≤ |fi | < ∞.
i i

The problem of course is to show that the answer in (2.48) does not depend on
which approximating sequence we use. This is not so obvious and is indeed the
core of the definition that we are using.

4. Covering lemmas
The first thing we need is the covering lemma for our semi-open intervals. If
we were proceeding more generally this would be the countable additivity of our
‘measure’ – len – on these sets which are finite unions intervals. The properties of
countable collections of our ‘intervals’ arise as soon as we look at a sequence, or
series, of step functions.
So, we want to extend Lemma 12 to countable collections of intervals.
Proposition 12. If Ci = [ai , bi ), i ∈ N, is a countable collection of intervals
then
X∞
(2.50) Ci ⊂ [a, b) ∀ i and Ci ∩ Cj = ∅ ∀ i 6= j =⇒ (bi − ai ) ≤ (b − a)
i=1

and
N
[ ∞
X
(2.51) [a, b) ⊂ Ci =⇒ (bi − ai ) ≥ (b − a).
i=1 i=1

Proof. You might think these are completely obvious, and the first is. Namely
if we consider any finite collection of the Ci then we can apply Lemma 12 to see
that the result holds, but the infinite sum is the supremum of all the finite sums,
so it too is less than or equal to b − a.
On the other hand (2.51) is not quite so obvious since we cannot just proceed
step by step since there may be all sorts of limit points of the ai and bi . Instead
we use Heine-Borel, which makes the argument ‘non-trivial’. To be able to apply
48 2. THE LEBESGUE INTEGRAL

Lemma 12 choose δ > 0. Now extend each interval by replacing the lower limit by
ai − 2−i δ and consider the open intervals which therefore have the property
[
(2.52) [a, b − δ] ⊂ [a, b) ⊂ (ai − 2−i δ, bi ).
i
Now, by Heine-Borel – the compactness of closed bounded intervals – a finite sub-
collection of these open intervals already covers [a, b − δ) so Lemma 12 does apply
to this finite collection of semi-open intervals [ai −2−1 δ, bi ) and shows that for some
finite N
XN X∞
(bi − ai ) − 2−i δ ≥ b − a − δ =⇒

(2.53) (bi − ai ) ≥ b − a − 2δ
i=1 i=1
where the last sum here might be infinite, but then it is all the more true. The fact
that this is true for all δ > 0 now proves (2.51). 
Welcome to measure theory.

5. Density of step functions (2011)


This section is inserted here to connect the approach via continuous functions
and the Riemann integral, in Section 1, to the more usual approach via step func-
tions starting in Section 2 (which does not use the Riemann integral). We prove
the ‘density’ of step functions in L1 (R) and this leads below to the proof that
Definition 5 is equivalent to Definition 7 so that one can use either.
A step function h : R −→ C is by definition a function which is the sum
of multiples of characteristic functions of (finite) intervals. Mainly for reasons of
consistency we use half-open intervals here, we define χ(a,b] = 1 when x ∈ (a, b]
(which if you like is empty when a ≥ b) and vanishes otherwise. So a step function
is a finite sum
XM
(2.54) h= ci χ(ai ,bi ]
i=1
where it doesn’t matter if the intervals overlap since we can cut them up. Anyway,
that is the definition.
Proposition 13. The linear space of step functions is a subspace of L1 (R),
f ∈ L1 (R) there is an absolutely
R
on which |h| is a norm, and for any P element
R
summable series of step functions {hi }, |hi | < ∞
i
X
(2.55) f (x) = hi (x) a.e.
i

Proof. The characteristic function χ(a,b] ∈ L1 (R) – this is one of the problems,
see Problem**. In fact it is straightforward, just take the decreasing sequence of
continuous functions



 0 if x < a − 1/n
n(x − a + 1/n) if



(2.56) un (x) = 1 if a<x≤b

1 − n(x − b)

 if b < x ≤ b + 1/n


0 if x > b + 1/n.
6. SETS OF MEASURE ZERO 49

This is continuous because each piece is continuous and the limits from the two
sides at the switching points are the same. This is clearly a decreasing sequence of
continuous functions which converges pointwise to χ(a,b] (not uniformly of course).
It follows that detelescopy this sequence gives an absolutely summable series of
continuous functions which converges pointwise to χ(a,b] which is therefore in L1 (R).
Now, conversely, each element f ∈ C(R) is the uniform limit of step functions –
this follows directly from the uniform continuity of continuous functions on compact
sets. It suffices to suppose that f is real and then combine the real and imaginery
parst. Suppose f = 0 outside [−R, R]. Take the subdivision of (−R, R] into 2n
equal intervals of lenght R/n and let hn be the step function which is sup f on the
closure of that interval. Choosing n large enough, sup f − inf f <  on each such
interval, by uniform continuity, and so sup |f − hn | < . Again this is a decreasing
sequence of step functions with integral bounded below so in fact it is the sequence
of partial sums of the absolutely summable series obtained by detelescoping.
Certainly then for each element f ∈ Cc (R) there is a sequence of step functions
with |f − hn | → 0. The same is therefore true of any element g ∈ L1 (R) since
R

then there is a sequence fn ∈ Cc (R) such that kf − fn kL1 → 0. So just choosing a


step function hn with kfn − hn k < 1/n ensures that kf − hn kL1 → 0.
Now to get an absolutely summable series of step function {gn } with kf −
N
P
gn k → 0 we just have to drop elements of the approximating sequence to speed
n=1
up the convergence and then detelescope the sequence. For the moment I do not
say that
X
(2.57) f (x) = gn (x) a.e.
n
although it is true! It follows from the fact that the right side does define an
element of L1 (R) and by the triangle inequality the difference of the two sides
has vanishing L1 norm, i.e. is a null function. So we just need to check that null
functions vanish outside a set of measure zero. This is Proposition 22 below, which
uses Proposition 23. Taking a little out of the proof of that proposition proves
(2.57) directly. 

6. Sets of measure zero


Definition 7 would not make much sense unless the sets on which our absolutely
summable series diverge are ‘ignorable’; we give them a name which makes this seem
likely.
Definition 8. A set E ⊂ R is of measure zero if there exists an absolutely
summable series of step functions fn such that
X
(2.58) |fn (x)| = ∞ ∀ x ∈ E.
n

Although it is really not necessary for the logical development of the subject
– at least not yet – it may be helpful at this stage to focus a little on just what
these sets are. This of course is an exercise in the manipulation of series of step
functions.
Proposition 14. A set E ⊂ R is of measure zero if and only if for every  > 0
there is a countable cover of E by intervals of total length at most .
50 2. THE LEBESGUE INTEGRAL

Proof. Suppose first that E satisfies the condition of the Proposition. Then
we need to find an absolutely summable series of step functions which diverges
(absolutely) on E. Taking  = 2−n for n ∈ N, there must exist a(n at most)
countable collection In,i = [an,i , bn,i ) of intervals with (bn,i > an,i and)
X [
(2.59) (bn,i − an,i ) ≤ 2−n , E ⊂ In,i .
i i

Now, consider the characteristic functions χ[an,i ,bn,i ) and order these in some way
as fk . This is an absolutley summable series of step functions since
XZ X
(2.60) |fk | = (bn,i − an,i ) ≤ 2.
k n,i

Note that all the terms are positive, so the order is immaterial here. On the other
hand, if x ∈ E then for each n there is at least one i such that x ∈ In,i . Thus there
is a subsequence of the fk on which fnk (x) = 1 so the pointwise series does diverge
at all points of E.
To the converse. Supposing we have an absolutely summable series, {fj }, of
step functions which diverges on E, we need to construct a countable cover of E by
intervals with lengths summing to less than 1/n. Using the absolutely summability,
if we choose N large enough
XZ
(2.61) |fk | < 1/n.
k>N

i
|fk (x)| > 2i+1 . This is a finite union of
P
Now for each i ≥ 1 consider the set where
k=1
disjoint intervals (since this is a step function) and the lower bound on the integral
i
Z X
(2.62) 1/n > |fN +k (x)| > 2i+1 len(Jn,i )
k=1

implies that len(Jn,i ) < 2−i−1 /n. Taking all of these countably many intervals
together (for fixed n) the sum of their lengths is therefore at most 1/n and they
must cover E since the infinite series diverges at each point of E. 

Note that the covering lemma, in particular (2.51), then implies

Corollary 3. A set of measure zero cannot contain a non-empty interval


[a, b).

So, this is supposed to indicate to you that these sets are indeed small.
Here is another indication of this.

Proposition 15. If g ∈ L1 (R) and g 0 : R −→ C is such that g 0 = g on R \ E


where E is of measure zero, then g 0 ∈ L1 (R).

Proof. What we have to play with here is an absolutely summable series, fn ,


of
P 0 functions which approximates g and another
step one, fn0 , which is such that
0
|fn (x)| = ∞ for all x ∈ E. So, to approximate g consider the interlaced series
n
7. MONOTONICITY LEMMAS 51

with terms

fk (x)
 n = 3k − 2
0
(2.63) hn (x) = fk (x) n = 3k − 1
 0

−fk (x) n = 3k.
Thus we add, but then subtract, fk0 . This series is absolutely summable with
XZ XZ XZ
(2.64) |hn | = |fk | + 2 |fk0 |.
n k k

When does the pointwise series converge absolutely? We must have


X X
(2.65) |fk (x)| + 2 |fk0 (x)| < ∞.
k k

The finiteness of the second term implies that x ∈ / E and the finiteness of the first
means that
X
(2.66) hn (x) = g(x) = g 0 (x) when (2.65) holds
n

N
0
P
since the finite sum is always fk (x), or this plus fN (x) – which tends to zero with
k=1
N by the absolute convergence of the series in (2.65). So indeed g 0 ∈ L1 (R). 

7. Monotonicity lemmas
Okay, now the basic result on which most of the properties of integrable func-
tions hinge is the following monotonicity lemma.
Lemma 13. Let fn be a sequence of (real-valued) step functions which decreases
monotonically to 0
(2.67) fn (x) ↓ 0 ∀ x ∈ R,
then
Z
(2.68) lim fn = 0.
n→∞

Note that ‘decreases’ here means that fn (x) is, for each x, a non-increasing
sequence which has limit 0. Of course this means that all the fn are non-negative.
Moreover the first one vanishes outside some interval [a, b) hence so do all of them.
The crucial thing about this lemma is that we get the vanishing of the limit of the
sequence of integrals without having to assume uniformity of the limit.

Proof. Going back to the definitionR of the integral of step functions, clearly
fn (x) ≥ fn+1 (x) for all x implies that fn is a decreasing (meaning non-increasing)
sequence. So there are only two possibilities, it converges to 0, as we claim, or it
converges to someR positive value. The second possibility means that there is some
δ > 0 such that fn ≥ δ for all n, so we just need to show that this is not so.
Given an  > 0 consider the sets
(2.69) Sj = {x ∈ [a, b); fj (x) ≤ }
52 2. THE LEBESGUE INTEGRAL

where [a, b) is an interval outside which f1 vanishes. Each of the Sj is a finite union
of intervals. Moreover
[
(2.70) Sj = [a, b)
j

since fn (x) → 0 for each x. In fact the Sj increase with j, Sj+1 ⊃ Sj , and if we
look at the successive differences by setting B1 = S1 and
(2.71) Bj = Sj+1 \ Sj
then the Bj are all disjoint, each consists of finitely many intervals and
[
(2.72) Bj = [a, b).
j

Thus, both halves of Proposition 12 apply to the intervals forming the Bj . Thus
X
(2.73) len(Bj ) = b − a.
j

Now, let A be such that f1 (x) ≤ A – and hence it is an upper bound for all the
fn ’s. In view of (2.73) we can choose N so large that
X
(2.74) len(Bj ) < .
j≥N

Dividing the integral for fk , k ≥ N + 1, into the part over SN +1 , where N is


determined by the choice of , and the rest we see that
Z
(2.75) fk ≤ (b − a) + A.

The first estimate comes for the fact the fact that fk ≤  on SN +1 , and the second
from our having arranged that the total lengths of the remaining intervals forming
[a, b) \ SN +1 is no more than  (and the function is no bigger than A.)
Thus, by choosing (b−a)+A < δ we conclude that the integrals are eventually
smaller than any δ > 0. 
We can make this result look stronger as follows.
Proposition 16. Let gn be a sequence of real-valued step functions which is
non-decreasing and such that
(2.76) lim gn (x) ∈ [0, ∞] ∀ x ∈ R
n→∞

(where we allow +∞ on the right) then


Z
(2.77) lim gn ∈ [0, ∞].
n→∞

I have written out [0, ∞] on the right here to make sure that it is clear that we only
demand that the non-decreasing sequence gn (x) ‘becomes 0 or positive’ in the limit
– including the possibility that it ‘converges’ to +∞ and the same may be true of
the sequence of integrals.
Proof. Consider the functions
(2.78) fn (x) = max(0, −gn (x)) ∀ x ∈ R.
8. LINEARITY OF L1 (R) 53

This is a sequence of non-negative step functions which decreases to 0 at each point


and
Z Z
(2.79) gn ≥ − fn .

Thus, as claimed, the result follows from the lemma. 

8. Linearity of L1 (R)
As promised we will soon proceed to show that the definition of a Lebesgue
integrable function above makes some sense. In particular that the integral is well-
defined by (2.48). However, let’s first do a warm-up exercise to see what the issues
are.
Recall that L1 (R) denotes the space of Lebesgue integrable functions on the
line – as in Definition 7.
Proposition 17. The set L1 (R) is a linear space.
Proof. The space of all functions on R is linear, so we just need to check that
L1 (R) is closed under multiplication by constants and addition. The former is easy
enough – multiplication by 0 gives the zero function which is integrable because we
can use the approximation sequence fj ≡ 0. If g ∈ L1 (R) then by definition there
is an absolutely summable series of step function with elements fn such that
XZ X X
(2.80) |fn | < ∞, g(x) = fn (x) ∀ x s.t. |fn (x)| < ∞.
n n n
P
Then if c 6= 0, cfn ‘works’ for cg with the crucial point being that |cfn (x)|
P n
converges if and only if |fn (x) converges.
n
The sum of two functions g and g 0 ∈ L1 (R) is a little trickier. The ‘obvious’
thing to do is to take the sum of the series of step functions. This will lead to trouble!
Instead, suppose that fn and fn0 are the terms in the series of step functions showing
that g, g 0 ∈ L1 (R). Then consider the ‘intertwined sum’
(
fk n = 2k − 1
(2.81) hn (x) =
fk0 n = 2k.
This is absolutely summable since
XZ XZ XZ
(2.82) |hn | = |fk | + |fk0 | < ∞.
n k k
P
More significantly, the series |hn (x)| converges if and only if both the series
P P 0 n
|fk (x)| and the series |fk (x)| converge. Then, because absolutely convergent
k k
series can be rearranged, it follows that
X X X X
(2.83) |hn (x)| < ∞ =⇒ hn (x) = fk (x) + fk0 (x) = g(x) + g 0 (x).
n n k k
0 1
Thus, g + g ∈ L (R). 
So, the message here is to be a bit careful about the selection of the ‘approxi-
mating’ absolutely summable series.
54 2. THE LEBESGUE INTEGRAL

9. The integral is well-defined


To show that the integral of an element of L(R) is well-defined, observe that
we can restrict our attention to real-valued functions, which is important since our
only tool is monotonicity.
Lemma 14. If f ∈ L1 (R) then Re f and Im f ∈ L1 (R) and can be approximated
by real absolutely summable series.
Of course the converse is clear since we can just use the linearity with f = Re f +
i Im f.

Proof. If fi is an absolutely summable approximating series for f let gi be


the series obtained by intertwining Re fk , Im fk and − Im fk . This is absoutely
summable and converges absolutely exactly where the series for fj converges ab-
|fi (x)| converges). Thus Re f ∈ L1 (R) and linearity shows
P
solutely (meaning
i
Im f ∈ L1 (R) too. 

That the original definition makes some sense, starts to become clear when we
check:
Proposition 18. For any element f ∈ L1 (R) the integral
Z XZ
(2.84) f= fn
n

is well-defined in that it is independent of which approximating absolutely summable


series of step functions satisfying (2.80) is used to define it.
Proof. We can suppose that fn and fn0 are two real absolutely summable
series as in (2.80). Now that we have a little experience, it is probably natural to
look at
(
fk (x) n = 2k − 1
(2.85) hn (x) = 0
−fk (x) n = 2k.
This is absolutely summable and the pointwise series is absolutely convergent only
when both series are absolutely convergent. The individual terms then tend to zero
and so we see that
X X
(2.86) |hn (x)| < ∞ =⇒ hn (x) = 0.
n n

Moreover, from the absolute convergence of the sequence of integrals –


X Z XZ
(2.87) | hn | ≤ |hn | < ∞
n n

it follows that we can rearrange the series to see that


XZ XZ XZ
(2.88) hn = fk − fk0 .
n k k

Now, what we want is that these two sums are equal, so we want to see that the
left side of this equality vanishes. This follows directly from the next result which
is just a little more general than needed here so is separated off. 
9. THE INTEGRAL IS WELL-DEFINED 55

Proposition 19. If an absolutely summable series of step functions satisfies


(2.80) with f = 0 a.e. (meaning f vanishes off a set of measure zero) then
XZ
(2.89) fn = 0.
n

Proof. We can assume that the series consists of real-valued functions. The
only thing we have at our disposal is the monotonicity result above. The trick to
using it is to choose an N ∈ N and consider the new series of step functions with
terms
N
X
(2.90) g1 (x) = fj (x), gk (x) = |fN +k−1 (x)|, k > 1.
j=1

Now, this is absolutely summable, since convergence is a property of the ‘tail’ and
in any case
XZ XZ
(2.91) |gk | ≤ |fn |.
k n

Moreover, since all the terms after the first are non-negative, the partial sums of
the series
p
X
(2.92) Gp (x) = gp (x) is non-decreasing.
k=1

It is again a sequence of step functions.


P Note that there are two possibilities,
depending on x. If the original series |fn (x)| diverges, i.e. converges to +∞,
n
then the P
same is true of Gp – since this is also a property of the tails. On the other
hand, if |fn (x)| is finite, then for large p,
n

N
X p−1
X X−1
p+N
(2.93) Gp (x) = fk (x) + |fN +j (x)| ≥ fk (x).
k=1 j=1 k=1

By assumption the right side converges to zero, so the limit of this series (which is
finite) is non-negative. So, Proposition 16 applies to Gp and shows that
Z
(2.94) lim Gp ≥ 0
p→∞

where divergence to +∞ is a possibility. This however means that, for the N we


originally chose,
XN Z X ∞ Z
(2.95) fk + |fN +k | ≥ 0.
j=1 k=1

This then is true for every N. On the other hand, the series of integrals is finite, so
given δ > 0 there exists M such that if N > M,
XZ N Z
X
(2.96) |fk | < δ =⇒ fk ≥ −δ ∀ N > M.
k≥M j=1
56 2. THE LEBESGUE INTEGRAL

This then implies that


XZ
(2.97) fk ≥ 0.
k

This is half of what we want, but the other half follows by applying the same
reasoning to −fk .
Thus the Proposition is proved for real series. For complex series we can work
with the real and imaginary parts as in Lemma 14 
This is a pretty fine example of measure-integration reasoning.
Corollary 4. The integral is a well-defined linear map
Z
(2.98) : L1 (R) −→ C

defined by setting
Z XZ
(2.99) f= fn
n

for any approximating sequence as in (2.80).


Proof. If one is needed. The alternating differences of two approximating
series to f satisfies the hypothesis of the Proposition above, so the sums of the
integrals are the same. The linearity of this functional is left as an exercise. 
In particular this integral is not trivial. Namely, if f is actually a step func-
tion then the sequence f1 = f, fj = 0 for all j > 1 is absoutely summable and
approximates f in the sense of (2.80) so
Z
(2.100) f is consistent with the integral on step functions.

So, we must be onto something here!


Here is another example of a result – really out of order in the development of
the integral – which uses the same ‘trickery’ and reading the proof now may help
get things clear.
Proposition 20. A countable union of sets of measure zero has measure zero.
Proof. By the definition above, a set E is of measure
PR zero if there exists an
absolutely summable series of step functions, fn , so |fn | < ∞ such that
n
X
(2.101) |fn (x)| = +∞ on E.
n

The data here gives us a countable collection Ej , j = 1, . . . , of sets and for each of
(j)
them we have an absolutely summable series of step functions fn such that
XZ X
(2.102) |fn(j) | < ∞, Ej ⊂ {x ∈ R; |fn(j) (x)| = +∞}.
n n

The idea is to look for one series of step functions which is absolutely summable
(j)
and which diverges absolutely on each Ej . The trick is to first ‘improve’ the fn .
Namely, the divergence in (2.102) is a property of the ‘tail’ – it persists if we toss
R
10. THE SEMINORM |f | 57

out any finite number of terms. The absolute convergence means that for each j
we can choose an Nj such that
X Z
(2.103) |fn(j) | < 2−j ∀ j.
n≥Nj

(j)
Now, simply throw away the all the terms in fn before n = Nj . If we relabel this
(j)
new sequence as fn again, then we still have (2.102) but in addition we have not
only absolute summability but also
XZ XXZ
(2.104) |fn(j) | ≤ 2−j ∀ j =⇒ |fn(j) | < ∞.
n j n

Thus, the double sum (of integrals of absolute values) is absolutely convergent.
(j)
Now, let hk be the fn ordered in some reasonable way – say by working along
each row j + n = p in turn – in fact any enumeration of the double sequence will
work. This is an absolutely summable series, because of (2.104). Moreover the
pointwise series
X X
(2.105) |hk (x)| = +∞ if |fn(j) (x)| = +∞ for any j
k n
P
since the second sum is contained in the first. Thus |hk (x)| diverges at each
k
point of each Ej , so the union has measure zero. 

R
10. The seminorm |f |
This is the first section where the new (as in Spring 2011) and old treatments
come together. I will use the term ‘model function’ to stand for ‘element of Cc (R) if
you are following the 2011 treatment, it means ‘step function’ if you are following
the previous approach.
By now the structure of the proofs should be getting somewhat routine – but
I will go on to the point that I hope it all becomes clear!
The next thing we want to show is that the putative norm on L1 does make
sense.
Proposition 21. If f ∈ L1 (R) then |f | ∈ L1 (R) and if fn is an absolutely
summable series of model functions (meaning elements of Cc (R) or step function
depending on where you are coming from) converging to f almost everywhere (that
is, on the complement of a set of measure zero) then
Z Z N
X Z Z
|f | = lim | fk | =⇒ | f| ≤ |f | and
N →∞
k=1
(2.106) Z n
X
lim |f − fj | = 0.
n→∞
j=1

So in some sense the definition of the Lebesgue integral ‘involves no cancellations’.


There are various extensions of the integral which do exploit cancellations – I invite
you to look into the definition of the Henstock integral (and its relatives).
58 2. THE LEBESGUE INTEGRAL

Proof. By definition if f ∈ L1 (R) then it is the limit, on the set of absolute


convergence, of a summable series of modle functions, {fn }. We need to make such
a series for |f |. The idea in this case is not too surprising – we know that
n
X X
(2.107) fj (x) → f (x) if |fj (x)| < ∞.
j=1 j

So, set
k
X k−1
X
(2.108) g1 (x) = |f1 (x)|, gk (x) = | fj (x)| − | fj (x)| ∀ x ∈ R.
j=1 j=1

Then, for sure,


N
X N
X X
(2.109) gk (x) = | fj (x)| → |f (x)| if |fj (x)| < ∞.
k=1 j=1 j

So, what we need to check, for a start, is that {gj } is an absolutely summable series
of model functions.
The triangle inequality in the ‘reverse’ form ||v| − |w|| ≤ |v − w| shows that, for
k > 1,
k
X k−1
X
(2.110) |gk (x)| = || fj (x)| − | fj (x)|| ≤ |fk (x)|.
j=1 j=1

Thus
XZ XZ
(2.111) |gk | ≤ |fk | < ∞
k k

so the gk ’s do indeed form an absolutely summable series. From its construction


we know that
XN N
X X
(2.112) gk (x) = | fj (x)| → |f (x)| if |fn (x)| < ∞.
k=1 j=1 n
P
So, this is what we want except that the set on which |gk (x)| < ∞ may be larger
k
than the set for which we have convergence here. Now, we have actually handled
this earlier but we can simply make the series converge less rapidly by adding a
‘pointless’ subseries. Namely replace gk by

gk (x)
 if n = 3k − 2
(2.113) hn (x) = fk (x) if n = 3k − 1

−fk (x) if n = 3k.

This series converges absolutely if and only if both the |gk (x)| and |fk (x)| series
converge – the convergence of the latter implies the convergence of the former so
X X
(2.114) |hn (x)| < ∞ ⇐⇒ |fk (x)| < ∞.
n k

On the other hand when this holds,


X
(2.115) hn (x) = |f (x)|
n
11. NULL FUNCTIONS 59

since each partial sum is either a sum for gk , or this with fn (x) added. Since
fn (x) → 0, (2.115) holds whenever the series converges absolutely, so indeed |f | ∈
L1 (R).
Then the first part of (2.106) follows from the definition of the integral applied
to |f | and this series, the second part by comparing the integral of the series term
by term and the third part by applying the same arguments to the seris {fk }k>n ,
Pn
as an absolutely summable series approximating f − fj and observing that the
j=1
integral is bounded by
Z n
X ∞ Z
X
(2.116) |f − fk | ≤ |fk | → 0 as n → ∞.
k=1 k=n+1


11. Null functions


Now, the next thing we want to know is when the R‘norm’, which is in fact only
a seminorm, on L1 (R), vanishes. That is, when does |f | = 0? One way is fairly
easy. The full result we are after is:-
Proposition 22. For an integrable function f ∈ L1 (R), the vanishing of |f |
R

implies that f is a null function in the sense that


(2.117) f (x) = 0 ∀ x ∈ R \ E where E is of measure zero.
Conversely, if (2.117) holds then f ∈ L1 (R) and |f | = 0.
R
R
Proof. The main part of this is the first part, that the vanishing of |f |
implies that f is null. This I will prove using the next Proposition. The converse
is the easier direction in the sense that we have already done it.
Namely, if f is null in the sense of (2.117) then, by the definition of a set of
measure zero, there exists an absolutely summable series of model functions, fn ,
such that
X
(2.118) E ⊂ {x ∈ R; |fn (x)| = ∞}.
n
Note that it is possible that the absolute series here diverges on a larger set than
E. Still, if we consider the alternating series
(
fk (x) if n = 2k − 1
(2.119) gn (x) =
−fk (x) if n = 2k
then
X X
(2.120) gn (x) = 0 whenever |gn (x)| < ∞,
n n
P
since the latter condition is equivalent to |fn (x)| < ∞. So in fact
n
X X
(2.121) |gn (x)| < ∞ =⇒ f (x) = gn (x) = 0
n n

because of (2.118). Thus the null function f ∈ L1 (R), and so is |f | and from (2.121)
Z XZ Z
(2.122) |f | = gk = lim fk = 0
k→∞
k
60 2. THE LEBESGUE INTEGRAL

where the last statement follows from the absolute summability. 


For the converse argument we will use the following result, which is also closely
related to the completeness of L1 (R).
Proposition
PR 23. If fn ∈ L1 (R) is an absolutely summable series, in the sense
that |fn | < ∞ then
n
X
(2.123) E = {x ∈ R; |fn (x)| = ∞} has measure zero
n
and if f : R −→ C and
X
(2.124) f (x) = fn (x) ∀ x ∈ R \ E
n

then f ∈ L1 (R),
Z XZ
f= fn ,
n
Z Z Z n
X XZ
(2.125) | f| ≤ |f | = lim | fj | ≤ |fj |,
n→∞
j=1 j
Z n
X
lim |f − fj | = 0.
n→∞
j=1

So this basically says we can replace ‘model function’ by ‘integrable function’ in


the definition (whichever one you started with) and get the same result. Of course
this makes no sense without the original definition.
Proof. The proof is very like the proof of completeness of the ‘completion’ of
a normed space outlined in the preceding chapter, here it is a little more concrete.
1
Thus, by assumption each fn P ∈L
R (R), so there exists an absolutely summable
series of model functions fn,j , so |fn,j | < ∞ and
j
X X
(2.126) |fn,j (x)| < ∞ =⇒ fn (x) = fn,j (x).
j j

We can expect f (x) to be given by the sum of the fn,j (x) over both n and j, but
in general, this double series is not absolutely summable. However we can make it
so. Simply choose Nn for each n so that
X Z
(2.127) |fn,j | < 2−n .
j>Nn

This is possible by the assumed absolute summability – so the tail of the series is
small. Having done this, we replace the series fn,j by
X
0 0
(2.128) fn,1 = fn,j (x), fn,j (x) = fn,Nn +j−1 (x) ∀ j ≥ 2.
j≤Nn

This still converges to fn on the same set as in (2.126). So in fact we can simply
0
replace fn,j by fn,j and we have in addition the estimate
XZ Z
0
(2.129) |fn,j | ≤ |fn | + 2−n+1 ∀ n.
j
12. THE SPACE L1 (R) 61

This follows from the triangle inequality since, using (2.127),


Z XN Z XZ Z
0 0 0 0 0
(2.130) |fn,1 + fn,j | ≥ |fn,1 | − |fn,j | ≥ |fn,1 | − 2−n
j=2 j≥2
R
and the left side converges to |fn | by (2.106) as N → ∞. Using (2.127) again
gives (2.129).
So, now dropping the primes from the notation and using the new series as fn,j
we can let gk be some enumeration of the fn,j – say be the standard diagonalization
procedure, but it really does not matter. This gives a new series of model functions
which is absolutely summable since
XN Z XZ XZ
(2.131) |gk | ≤ |fn,j | ≤ ( |fn | + 2−n+1 ) < ∞.
k=1 n,j n

Now, using the freedom to rearrange absolutely convergent series we see that
X X XX
(2.132) |fn,j (x)| < ∞ =⇒ f (x) = gk (x) = fn,j (x).
n,j k n j

The set where (2.132) fails is a set of measure zero, by definition. If necessary, take
another absolutely summable series of model functions hk which diverges on E (at
least) and inserting hk and −hk between successive gk ’s as before. This new series
still coverges to f by (2.132) and shows that f ∈ L1 (R) as well as (2.123). The final
result (2.125) also follows by rearranging the double series for the integral (which
is also absolutely convergent). 

For the moment we only need the weakest part, (2.123), of this. That is, for
any absolutely summable series of integrable functions the absolute pointwise series
converges off a set of measure zero – can only diverge on a set of measure zero. It
is rather shocking but thisR allows us to prove the rest of Proposition 22! Namely,
suppose f ∈ L1 (R) and |f | = 0. Then Proposition 23 applies to the series with
each term being |f |. This is absolutely summable since all the integrals are zero.
So it must converge pointwise except on a set of measure zero. Clearly it diverges
whenever f (x) 6= 0,
Z
(2.133) |f | = 0 =⇒ {x; f (x) 6= 0} has measure zero

which is what we wanted to show to finally complete the proof of Proposition 22.

12. The space L1 (R)


Finally this allows us to define the standard Lebesgue space
(2.134) L1 (R) = L1 (R)/N , N = {null functions}
R
and to check that |f | is indeed a norm on this space.
Theorem 7. The space L1 (R) defined by (2.134) is a Banach space in which
the model functions are a dense subspace.
The elements of L1 (R) are equivalence classes of functions
(2.135) [f ] = f + N , f ∈ L1 (R).
62 2. THE LEBESGUE INTEGRAL

That is, we ‘identify’ two elements of L1 (R) if (and only if) their difference is null,
which is to say they are equal off a set of measure zero. Note that the set which is
ignored here is not fixed, but can depend on the functions.
Proof. For an element of L1 (R) the integral of the absolute value is well-
defined by Proposition 18
Z
(2.136) k[f ]kL1 = |f |.
1
R
The integral of the absolute value, |f R | is a semi-norm on L (R) – it satisfies all
the properties of a norm except that |f | = 0 does not imply f = 0, only f ∈ N .
It follows that on the quotient, k[f k is indeed a norm.
The completeness of L1 (R) is a direct consequence of Proposition 23. Namely,
we showed earlier that to show a normed space is complete it is enough to check that
any absolutely summable series converges. Namely, if [fj ] is an absolutely summable
series in L1 (R) then fj is absolutely summable in L1 (R) and by Proposition 23 the
sum of the series exists so we can use (2.124) to define f of the set E and take it to
be zero on E. Then, f ∈ L1 (R) and the last part of (2.125) means precisely that
X Z X
(2.137) lim k[f ] − [fj ]kL1 = lim |f − fj | = 0
n→∞ n→∞
j<n j<n

shows the desired completeness. 


Note that despite the fact that it is technically incorrect, everyone says ‘L1 (R)
is the space of Lebesgue integrable functions’ even though it is really the space
of equivalence classes of these functions modulo equality almost everywhere. Not
much harm can come from this mild abuse of language.

13. The three integration theorems


There are three standard results on convergence of sequences of integrable func-
tions which are powerful enough to cover most situations that arise in practice –
a Monotonicity Lemma, Fatou’s Lemma and Lebesgue’s Dominated Convergence
theorem.
Lemma 15 (Montonicity). If fj ∈ L1 (R) is a monotone sequence, either fj (x) R ≥
fj+1 (x) for all x ∈ R and all j or fj (x) ≤ fj+1 (x) for all x ∈ R and all j, and fj
is bounded then
(2.138) {x ∈ R; lim fj (x) is finite} = R \ E
j→∞

where E has measure zero and


f = lim fj (x) a.e. is an element of L1 (R)
j→∞
(2.139) Z Z Z
with f = lim fj and lim |f − fj | = 0.
j→∞ j→∞

In the usual approach through measure one has the concept of a measureable, non-
negative, function for which the integral ‘exists but is infinite’ – we do not have
this (but we could easily do it, or rather you could). Using this one can drop the
assumption about the finiteness of the integral but the result is not significantly
stronger.
13. THE THREE INTEGRATION THEOREMS 63

Proof. Since we can change the sign of the fi it suffices to assume that the fi
are monotonically increasing. The sequence of integrals is therefore also montonic
increasing and, being bounded, converges. Turning the sequence into a series, by
setting g1 = f1 and gj = fj − fj−1 for j ≥ 1 the gj are non-negative for j ≥ 1 and
XZ XZ Z Z
(2.140) |gj | = gj = lim fn − f1
n→∞
j≥2 j≥2

converges. So this is indeed an absolutely summable series. We therefore know


from Proposition 23 that it converges absolutely a.e., that the limit, f, is integrable
and that
Z XZ Z
(2.141) f= gj = lim fj .
n→∞
j

The second part, corresponding to convergence for the equivalence classes in L1 (R)
follows from the fact established earlier about |f | but here it also follows from the
monotonicity since f (x) ≥ fj (x) a.e. so
Z Z Z
(2.142) |f − fj | = f − fj → 0 as j → ∞.

Now, to Fatou’s Lemma. This really just takes the monotonicity result and
applies it to a sequence of integrable functions with bounded integral. You should
recall that the max and min of two real-valued integrable functions is integrable
and that
Z Z Z
(2.143) min(f, g) ≤ min( f, g).

This follows from the identities


(2.144) 2 max(f, g) = |f − g| + f + g, 2 min(f, g) = −|f − g| + f + g.
Lemma 16 (Fatou). Let fj ∈RL1 (R) be a sequence of real-valued integrable and
non-negative functions such that fj is bounded above then

f (x) = lim inf fn (x) exists a.e., f ∈ L1 (R) and


(2.145) Z n→∞ Z Z
lim inf fn = f ≤ lim inf fn .

Proof. You should remind yourself of the properties of lim inf as necessary!
Fix k and consider
(2.146) Fk,n = min fp (x) ∈ L1 (R).
k≤p≤k+n

As discussed above this is integrable. Moreover, this is a decreasing sequence, as n


increases, because the minimum is over an increasing set of functions. Furthermore
the Fk,n are non-negative so Lemma 15 applies and shows that
Z Z
1
(2.147) gk (x) = inf fp (x) ∈ L (R), gk ≤ fn ∀ n ≥ k.
p≥k
64 2. THE LEBESGUE INTEGRAL

Note that for a decreasing sequence of non-negative numbers the limit exists every-
where and is indeed the infimum. Thus in fact,
Z Z
(2.148) gk ≤ lim inf fn ∀ k.

Now, let k vary. Then, the infimum in (2.147) is over a set which decreases as k
increases. Thus the gk (x) are increasing. The integrals of this
R sequence are bounded
above in view of (2.148) since we assumed a bound on the fn ’s. So, we can apply
the monotonicity result again to see that
f (x) = lim gk (x) exists a.e and f ∈ L1 (R) has
k→∞
(2.149) Z Z
f ≤ lim inf fn .

Since f (x) = lim inf fn (x), by definition of the latter, we have proved the Lemma.

Now, we apply Fatou’s Lemma to prove what we are really after:-
Theorem 8 (Dominated convergence). Suppose fj ∈ L1 (R) is a sequence of
integrable functions such that
∃ h ∈ L1 (R) with |fj (x)| ≤ h(x) a.e. and
(2.150)
f (x) = lim fj (x) exists a.e.
n→∞

then f ∈ L1 (R) and


R R
f = limn→∞ fn (including the assertion that this limit
exists).
Proof. First, we can assume that the fj are real since the hypotheses hold for
the real and imaginary parts of the sequence and together give the desired result.
Moroever, we can change all the fj ’s to make them zero on the set on which the
estimate in (2.150) does not hold. Then this bound on the fj ’s becomes
(2.151) −h(x) ≤ fj (x) ≤ h(x) ∀ x ∈ R.
In particular this means that gj = h − fj is a non-negative sequence of integrable
functions
R andR the sequence
R ofR integrals is also bounded, since (2.150) also implies
that |fj | ≤ h, so gj ≤ 2 h. Thus Fatou’s Lemma applies to the gj . Since we
have assumed that the sequence gj (x) converges a.e. to h − f we know that
h − f (x) = lim inf gj (x) a.e. and
(2.152)
Z Z Z Z Z
h− f ≤ lim inf (h − fj ) = h − lim sup fj .

Notice the change on the right from liminf to limsup because of the sign.
Now we can apply the same argument to gj0 (x) = h(x) + fj (x) since this is also
non-negative and has integrals bounded above. This converges a.e. to h(x) + f (x)
so this time we conclude that
Z Z Z Z Z
(2.153) h + f ≤ lim inf (h + fj ) = h + lim inf fj .
R
In both inequalities (2.152) and (2.153) we can cancel an h and combining them
we find
Z Z Z
(2.154) lim sup fj ≤ f ≤ lim inf fj .
14. NOTIONS OF CONVERGENCE (2011) 65

In particular the limsup on the left is smaller than, or equal to, the liminf on the
right, for the sameR real sequence. This however implies that they are equal and
that the sequence fj converges. Thus indeed
Z Z
(2.155) f = lim fn .
n→∞


Generally in applications it is Lebesgue’s dominated convergence which is used
to prove that some function is integrable. Of course, since we deduced it from
Fatou’s lemma, and the latter from the Monotonicity lemma, you might say that
Lebesgue’s theorem is the weakest of the three! However, it is very handy.
We have been dealing with two basic notions of convergence. Namely conver-
gence almost everywhere and convergence in the mean where the latter means pre-
cisely convergence with respect to the norm in L1 . In fact neither of these implies the
other. Lebesgue’s theorem on dominated convergence gives one connection between
them – if one has convergence almost everywhere and the sequence is dominated
by one integrable function then it converges in the mean. Conversely, a sequence
(of integrable functions) which converges in the mean is necessarily Cauchy in the
mean, i.e. in L1 (R), and has a subsequence which is the sequence of partial sums of
an absolutely summable series. Thus convergence in the mean implies convergence
almost everywhere (and dominated) along a subsequence (of any subsequence).

14. Notions of convergence (2011)


This might be a good point to clarify the different notions of convergence we
are dealing with. Here are four:-
(1) Convergence of a sequence in L1 (R) (or by slight abuse of language in
L1 (R)) – f and fn ∈ L1 (R) and
(2.156) kf − fn kL1 → 0 as n → ∞.
(2) Convergence almost every where:- For some sequence of functions fn and
function f,
(2.157) fn (x) → f (x) as n → ∞ for x ∈ R \ E
where E ⊂ R is of measure zero.
(3) Dominated convergence:- Now fj ∈ L1 (R) (or representatives in L1 (R))
such that |fj | ≤ F (a.e.) for some F ∈ L1 (R) and (2.157) holds.
(4) What we might call ‘absolutely summable convergence’. Again fn ∈ L1 (R)
n
gj where gj ∈ L1 (R) and
P PR
now such that fn = |gj | < ∞. Then
j=1 j
(2.157) holds for some f.
(5) Monotone convergence. If fj ∈ L1 (R) are real valued andRmontonic Rwith
fj bounded then fj → f almost everywhere, in L1 and f = limj fj .
R

So, one important point to know is that 1 does not imply 2. Nor conversely
does 2 imply 1 even if we assume that all the fj and f are in L1 (R).
However, Montone convergence implies Dominated convergence. Namely if
f is the limit then |fj | ≤ |f | and fj → f almost everywhere. Also, Monotone
convergence implies convergence with absolute summability simply by taking the
sequence to have first term f1 and subsequence terms fj − fj−1 (assuming that
66 2. THE LEBESGUE INTEGRAL

fj is monotonic increasing) one gets an absolutely summable series with sequence


of finite sums converging to f. Similarly absolutely summable convergence implies
dominatedP convergence for the sequence of partial sums; by montone convergence
the series |fn (x)| converges a.e. and in L1 to some function F which dominates
n
the partial sums which in turn converge pointwise.

15. The space L2 (R) (2011)


So far we have discussed the Banach space L1 (R). The real aim is to get a
good hold on the (Hilbert) space L2 (R). This can be approached in several ways.
Here I will proceed in close parallel to what has been done above to get L1 as a
completion of Cc (R) with respect to the appropriate norm. So, the initial definition
has the same ‘tricky’ structure as the definition of L1 (R) but we will arrive at a
useful sensible characterization below.
So, here the appropriate norm is given by
Z Z R
2 2
(2.158) kf kL2 = |f | = lim |f (x)|2 dx
R R→∞ −R

where the integral is a Riemann integral. To check that this really is a norm we
use the Cauchy-Schwarz inequality. Namely if f, g ∈ Cc (R) then
Z
(2.159) | f g| ≤ kf kL2 kgkL2 .
R

At this point you should read Section 1 of the next chapter. It is straightforward
to check that Cc (R) with the inner product
Z
(2.160) (f, g) = f g

is a pre-Hilbert space and hence a normed space with the norm being precisely the
L2 -norm.
The objective then is to give a concrete completion of this, following the same
path as for L1 (R) but now with a little more machinery to work with! The result
will be a Hilbert space. Given our experience up to this point I will ‘telescope’ the
definition.
Definition 9. The space L2 (R) consists of those functions f : R −→ C such
that there exists a sequence Fj ∈ Cc (R) which comes from an absolutely ‘square
summable’ in the sense that

X
(2.161) kFn − Fn−1 kL2 < ∞
n=2

and such that


(2.162) f (x) = lim Fj (x) on R \ E, E of measure zero.
j→∞

In particular the sequence in (2.162) must converge off a set of measure zero
for the statement to make sense. Of course we can set f1 = F1 and fn = Fn − Fn−1
for n > 1 and recover our earlier form of absolute summability (now with respect
15. THE SPACE L2 (R) (2011) 67

to the L2 norm):
X
fj ∈ Cc (R), kfj kL2 < ∞
j
(2.163) X
f (x) = fj (x) a.e.
j

It is also convenient to define another space


Definition 10. A function f : R −→ C is in L1loc (R) if for each r > 0
(
f (x) if |x| ≤ r
(2.164) f (r) (x) =
0 if |x| > r
is in L1 (R).
Note that you should be now be used to the fact that this absolute (square
in this case) summability just means that the sequence of partial sums, Fn (x) =
n
fj (x) is Cauchy in Cc (R) with respect to the L2 norm and in a uniform sense,
P
j=1
namely that
(2.165)
X X
kFn − Fn−1 kL2 < ∞ =⇒ if q > p, kFp − Fq kL2 ≤ kfj kL2 → 0 as p → ∞.
n j≥p

So we proceed to examine the properties that follow from this definition and
our work to date. This is to some extent ‘revision’ of the treatment of L1 (R).
Theorem 9. The space L2 (R) is linear and
(1) If f ∈ L2 (R) then |f |, f , Re f, (Re f )+ are in L2R(R).
(2) If f ∈ L2 (R) then |f |2 ∈ L1 (R) and kf k2L2 = |f |2 is a seminorm on
L2 (R) with null space N the space of null functions (vanishing off some
set of measure zero).
(3) If f ∈ L2 (R) then f ∈ L1loc (R).
(4) If f, g ∈ L2 (R) then f g ∈ L1 (R) and (2.159) holds.
(5) The space L2 (R) = L2 (R)/N is a Banach space (and in fact a Hilbert
space) with the norm k · kL2 .
(6) If f ∈ L1loc (R) and |f |2 ∈ L1 (R) then f ∈ L2 (R).
Proof. So, the objective here is not to work too hard.
The linearity of L2 (R) follows as for L1 (R). Namely the zero function is in
L (R) and if c ∈ C and f ∈ L2 (R) has approximating sequence as in the definition
2

then cfj is an approximating sequence for cf. If g ∈ L2 (R) is another element


with approximating sequence gj then take the sequence with alternating fj and gj .
It is absolutely summable and converges to f (x) + g(x) where the two sequences
converge to f (x) and g(x) respectively, which is to say off a set of measure zero.
Similarly if f ∈ L2 (R) has approximating sequence fj then
j
X k−1
X
(2.166) g1 = |f1 (x)|, gj (x) = | fk (x)| − | fk (x)|
k=1 k=1

is a sequence in Cc (R) which converges to |f (x)| when (2.162) holds. So, it is enough
to show that it is absolutely summable but this follows from the triangle inequality
68 2. THE LEBESGUE INTEGRAL

since
j
X k−1
X
(2.167) kgj kL2 = k| fk (x)| − | fk (x)|kL2 ≤ kfj kL2 , j ≥ 2.
k=1 k=1

That Re f, Im f and hence f are in L2 (R) follows by taking the appropriate se-
quences and (Re f )+ = 12 (Re f + | Re f |).
Now, if f ∈ L2 (R) and fj is an absolutely summable approximating sequence
Pj
consider the sequence of partial sums Fj (x) = fk (x) ∈ C(R) and then set
P k=1
gj (x) = |Fj (x)| ≤ |fk (x)| so also, |gj − gj−1 | ≤ |fj |. Expanding out the norm
k≤j
using the Cauchy-Schwarz inequality and using the triangle inequality on the second
factor
Z
|gj2 − gj−1
2
|
Z
= |gj − gj−1 |(gj + gj−1 ) ≤ kgj − gj−1 kL2 kgj + gj−1 kL2

(2.168) j
X
≤ 2kfj kL2 ( kfk kL2 ≤ Ckfj |L2
k=1
Z XZ
2
=⇒ |g1 | + |gj2 − gj−1
2
| < ∞,
j≥2

where the constant in the second last line comes from the absolute summability in
L2 . So, detelescoping as usual, the sequence gj2 = |Fj |2 is the sequence of partial
sums of an absolutely summable series, namely with first term g12 and subsequent
terms gj2 − gj−1
2
, with respect to the L1 norm. Thus, essentially from the definition
1
of L (R), the limit

(2.169) |f (x)|2 = lim |Fj (x)|2 a.e.


j→∞

is an element of L1 (R) as claimed. Moreover,


Z
(2.170) |f |2 = lim k|Fj |2 kL1 exists
j→∞

and satisfies at least the first conditions of a seminorm – weak positivity and ho-
mogeneity. We still need to check the triangle inequality. In fact if f, g ∈ L2 (R)
and fj , gj are approximating sequences then fj + gj is an approximating sequence
for f + g – by the triangle inequaltiy on C(R) it is absolutely summable in L2 and
the series converges to f (x) + g(x) a.e. Thus
Z
|f + g|2 = lim k|Fj + Gj |2 kL1 = lim kFj + Gj k2L2
j→∞ j→∞
(2.171)
≤ lim (kFj kL2 + kGj kL2 ) = (kf kL2 + kgkL2 )2
2
j→∞

so the triangle inequality does hold and k · kL2 is a seminorm on L2 (R).


15. THE SPACE L2 (R) (2011) 69

Next take r > 0 and for an absolutely summable series in L2 consider the
cut-off series with nth term
(
(r) fn (x) if |x| ≤ r
(2.172) fn (x) =
0 if |x| > r
just as in (2.164). This is certainly a sequence of integrable functions (maybe not
continuous since they might jump at the ends of the interval). Moreover
(r)
(2.173) kfj kL2 ≤ kfj kL2
so the cut-off series is absolutely summable in L2 . In fact it is absolutely summable
in L1 by an application of the Cauchy-Schwarz inequality:
Z Z r Z r
(r) 1 1
(2.174) |fj (x)| = |fj | ≤ (2r) 2 ( |fj |2 ) 2
−r −r
P
where we expand using fj = fj · 1. Thus the series fj (x) converges almost
j
everywhere on the interval (−r, r) and to zero outside it, the limit f (r) a.e. is
therefore in L1 (R), which is the statement that f ∈ L1loc (R).
Next consider two elements f, g ∈ L2 (R) with corresponding approximating se-
quences and partial sums Fj , Gj as above. Applying the Cauchy-Schwarz inequality
shows that
kFj Gj − Fj−1 Gj−1 kL1 = kFj Gj − Fj Gj−1 + Fj Gj−1 − Fj−1 Gj−1 kL1
(2.175)
≤ kFj kL2 kGj − Gj−1 kL2 + kGj−1 kL2 kFj − Fj−1 kL2 .
The norms kFj kL2 and kGj kL2 converge and are therefore bounded, so the sequence
Fj Gj is the partial sum of an absolutely summable series in L1 . Thus its limit a.e. is
an element of L1 (R), but this is f g. Thus the product is in L1 (R) and the Cauchy-
Schwarz inequality extends by continuity to L2 (R),
Z Z
(2.176) | f g| = lim | Fj Gj | ≤ lim kFj kL2 kGj kL2 = kf kL2 kgkL2 .
j→∞ j→∞

Next to the completeness of L2 (R) = L2 (R)/N to which the seminorm descends


as a norm. It suffices to show that an absolutely summable series converges. You
can do this readily following the double sequence argument used for L1 (R). Instead
let me use the last part to prove it.
So instead we proceed to prove that
(2.177) f ∈ L1loc (R) and |f |2 ∈ L1 (R) =⇒ f ∈ L2 (R).
Since it is the only approach available, we need to construct an approximating
sequence in Cc (R) which is a absolutely summable with respect to the L2 norm.
First we simplify a little by observing that we can assume that f in (2.177)
is real and non-negative. The initial assumption that f ∈ L1loc (R) implies , Re f
|f | ∈ L1loc (R) (by the properties of L1 (R)). So consider
1
g = (Re f )+ = (Re f + | Re f |) ∈ L1loc (R),
2
we would like to see that g 2 ∈ L1 (R) which is not immediately obvious. Still, for
any r > 0, g (r) ∈ L1 (R) and so we can take a real sequence Fj ∈ Cc (R) such that
Fj → g (r) in L1 and Fj (x) → g (r) (x) almost everywhere (take the partial sums of
an absolutely summable series converging to g (r) ). Moreover, we can replace the Fj
70 2. THE LEBESGUE INTEGRAL

by their positive parts and still get convergence to g (r) a.e. and in L1 (R) (because
it is still absolutely summable so converges with respec to the L1 seminorm to its
limit a.e.) Now consider the sequence of functions
(2.178) hj = min(Fj , |f |).
1
This is a sequence in L (R) – since it vanishes outside some interval (depending
on j) and within that interval is the minimum of two L1 functions. Moreover, h2j ∈
L1 (R) as well, since it has the same properties – using the fact that |f |2 ∈ L1 (R).
Since g (r) ≤ (Re f )+ ≤ |f | wherever Fj (x) → g (r) (x), which is to say a.e., it follows
that hj (x) → g (r) (x) as well – since the limit is between 0 and |f (x)|. So it follows
that h2j → (g (r) (x))2 on the same set, so a.e., but also |hj (x)2 | ≤ |f |2 ∈ L1 (R).
So Lebesgue’s Theorem of dominated convergence implies that the limit, (g (r) )2 ∈
L (R) and that |(g ) | ≤ |f | . Now, let r → ∞; the sequence (g (r) )2 is in
1 (r) 2 2
R R

L1 (R), is increasing and has bounded norm, so by Monotone convergence its limit,
g 2 , is in L1 (R).
So this means that in proving (2.177), which is to say (6) in the statement of
the theorem, we can assume that f ≥ 0. Since we can then apply the result to
(Re f )+ and (Im f )+ and their negative parts – by the same argument applied to
−if etc. Then the result follows for f is a linear combination of these.
Now, what we have gained is positivity, then the fact that f 2 ∈ L1 (R) alone
does indeed imply that f ∈ L2 (R). We can see this by choosing an approximat-
ing, absolutely summable (in L1 norm), sequence fj ∈ Cc (R) with sequence of
partial sums, Fj , converging to f 2 a.e.; then |Fj − f 2 | →P
R
0 as well. The abso-
1
lute summability with respect to L implies that H(x) = |fj (x)| ∈ L1 (R) so
j
the convergence is also dominated, |Fj (x)| ≤ H(x). Now, replace the Fj by their
positive parts. Since the a.e. limit, f 2 , is non-negative this new sequence still con-
verges a.e. to f 2 and it is still dominated, so |(Fj )+ − f 2 | → 0 by Lebesgue’s
R

theorem again. Thus we can assume that Fj ≥ 0 – this is still a sequence in


1
Cc (R). Finally consider hj (x) = (Fj (x)) 2 which is also a sequence in Cc (R). It con-
1
verges a.e. to f (x) = (f 2 (x)) 2 (always taking positve square-roots of course). Thus
|hj (x) − f (x)|2 → 0 a.e. and
(2.179) |hj (x) − f (x)|2 ≤ 2h2j (x) + 2f 2 (x) ≤ 2H(x) + 2f 2 (x)
is bounded by a fixed function in L1 (R). Thus in fact khj −f kL2 → 0 by Dominated
Convergence. From this in turn it follows that hj is Cauchy with respect to the L2
seminorm (it is converging with respect to it) and hence if we pass to a sufficiently
sparse subsequence we can arrange that
(2.180) khj+1 − hj kL2 ≤ 2−j .
This gives (after detelescoping) an absolutely square summable series in Cc (R), i.e.
summable with respect to the L2 norm, which converges a.e. to f. Thus, f ∈ L2 (R)
which is what we wanted to show.
So, we have proved everything in Theorem 9 except perhaps completeness.
However, if gk is now a sequence in L2 (R) which is absolutely summable with
respect to the L2 norm then the limit of the sequence or partial sums, Gk exists
almost everywhere – since the sequence is absolutely summable with respect to L1
on [−r, r]. This limit f ∈ L1loc (R) and |f |2 ∈ L1 (R) is the limit of |Gj |2 in L1 (R)
16. THE SPACES Lp (R) 71

from the argument above. So in fact f ∈ L2 (R) and kGk − f kL2 → 0, which is
what we want. 
So, we have shown that (6) in the Theorem is an equivalent definition of L2 (R).
This, and its generalization, is what is used as the definition in the section on Lp
below. For later purposes we really only need L2 (R) but you might want to read
about Lp (R) as further mental exercise!

16. The spaces Lp (R)


First, we define locally integrable functions. Thus f : R −→ C is locally
integrable if
(
f (x) x ∈ [−N, N ]
(2.181) F[−N,N ] = =⇒ F[−N,N ] ∈ L1 (R) ∀ N.
0 x if |x| > N
For example any continuous function on R is locally integrable.
Lemma 17. The locally integrable functions form a linear space, L1loc (R).
Proof. Follows from the linearity of L1 (R). 
p
Definition 11. The space L (R) for any 1 < p < ∞ consists of those functions
in L1loc such that |f |p ∈ L1 (R).
It is important to note that |f |p ∈ L1 (R) is not, on its own, enough to show
that f ∈ Lp (R) – it does not in general imply the local integrability of f.
What are some examples of elements of Lp (R)? One class, which we use below,
comes from cutting off elements of L1loc (R). Namely, we can cut off outside [−R, R]
and for a real function we can cut off ‘at height R’ (it doesn’t have to be the same
R but I am saving letters)


 0 if |x| > R

R if |x| ≤ R, |f (x)| > R
(2.182) f (R) (x) =


 f (x) if |x| ≤ R, |f (x)| ≤ R
−R if |x| ≤ R, f (x) < −R.

For a complex function apply this separately to the real and imaginary parts. Now,
f (R) ∈ L1 (R) since cutting off outside [−R, R] gives an integrable function and
then we are taking min and max successively with ±Rχ[−R,R] . If we go back to the
definition of L1 (R) but use the insight we have gained from there, we know that
there is an absolutely summable sequence of model functions, fj , with sum con-
verging a.e. to f (R) . The absolute summability means that |fj | is also an absolutely
summable series, and hence its sum a.e., denoted g, is an integrable function by the
Monotonicity Lemma above – it is increasing with bounded integral. Thus if we let
Fn be the partial sum of the series
(2.183) Fn → f (R) a.e., |Fn | ≤ g
and we are in the setting of Dominated convergence – except of course we already
know that the limit is in L1 (R). However, we can replace Fn by the sequence of
(R)
cut-off model functions Fn without changing the convergence a.e. or the bound.
Now,
(2.184) |Fn(R) |p → |f (R) |p a.e., |Fn(R) |p ≤ Rp χ[−R,R]
72 2. THE LEBESGUE INTEGRAL

and we see that |f (R) | ∈ Lp (R) by Lebesgue Dominated convergence.


We can encapsulate this in a lemma:-
Lemma 18. If f ∈ L1loc (R) then with the definition from (2.182), f (R) ∈ Lp (R),
1 ≤ p < ∞ and there exists a sequence sn of model functions converging a.e. to
f (R) with |sn | ≤ Rχ[−R,R] .
Theorem 10. The spaces Lp (R) are linear, the function
Z 1/p
p
(2.185) kf kLp = |f |

is a seminorm on it with null space N , the space of null functions on R, and


Lp (R) = Lp (R)/N is a Banach space in which the model functions include as a
dense subspace.
Proof. First we need to check the linearity of Lp (R). Clearly λf ∈ Lp (R) if
f ∈ Lp (R) and λ ∈ C so we only need consider the sum. Then however, we can
use Lemma 18. Thus, if f and g are in Lp (R) then f (R) and g (R) are in Lp (R) for
any R > 0. Now, the approximation by model functions in the Lemma shows that
f (R) + g (R) ∈ Lp (R) since it is in L1 (R) and |f (R) + g (R) |p ∈ L1 (R) by dominated
convergence (the model functions being bounded). Now, letting R → ∞ we see
that
f (R) (x) + g (R) (x) → f (x) + g(x) ∀ x ∈ R
(2.186)
|f (R) + g (R) |p ≤ ||f (R) | + |g (R) ||p ≤ 2p (|f |p + |g|p )
so by Dominated Convergence, f + g ∈ Lp (R).
That kf kLp is a seminorm on Lp (R) is an integral form of Minkowski’s inequal-
ity. In fact we can deduce if from the finite form. Namely, for two step functions
f and g we can always find a finite collection of intervals on which they are both
constant and outside which they both vanish, so the same is true of the sum. Thus
  p1   p1
Xn n
X
kf kLp =  |ci |p (bi − ai ) , kgkLp =  |di |p (bi − ai ) ,
j=1 j=1
(2.187)   p1
Xn
kf + gkLp = |ci + di |p (bi − ai ) .
j=1
1
Absorbing the lengths into the constants, by setting c0i = ci (bi − ai ) p and d0i =
1
di (bi − ai ) p , Minkowski’s inequality now gives
! p1
X
0 0 p
(2.188) kf + gkLp = |ci + di | ≤ kf kLp + kgkLp
i

which is the integral form for step functions. Thus indeed, kf kLp is a norm on the
step functions.
For general elements f, g ∈ Lp (R) we can use the approximation by step
functions in Lemma 18. Thus for any R, there exist sequences of step functions
sn → fR(R) , tn → g (R) a.e.
R andRbounded by R R on [−R, R] so by Dominated Conver-
gence, |f (R) |p = lim |sn |p , |g (R) |p and |f (R) + g (R) |p = lim |sn + tn |p . Thus
R
16. THE SPACES Lp (R) 73

the triangle inequality holds for f (R) and g (R) . Then again applying dominated
convergence as R → ∞ gives the general case. The other conditions for a seminorm
are clear.
Then the space of functions with |f |p = 0 is again just N , independent of p,
R

is clear since f ∈ N ⇐⇒ |f |p ∈ N . The fact that Lp (R) = Lp (R)/N is a normed


space follows from the earlier general discussion, or as in the proof above for L1 (R).
So, only the comleteness of Lp (R) remains to be checked and we know this
is equivalent to the convergence of any absolutely summable series. So, we can
suppose fn ∈ Lp (R) have

X Z  p1
p
(2.189) |fn | < ∞.
n

Consider the sequence gn = fn χ[−R,R] for some fixed R > 0. This is in L1 (R) and
1
(2.190) kgn kL1 ≤ (2R) q kfn kLp

by the integral form of Hölder’s inequality


(2.191) Z
1 1
f ∈ Lp (R), g ∈ Lq (R), + = 1 =⇒ f g ∈ L1 (R) and | f g| ≤ kf kLp |kgkLq
p q
which can be proved by the same approximation argument as above, see Problem 20.
1
Thus the series gn is absolutely summable
P in L and so converges absolutely almost
everywhere. It follows that the series fn (x) converges absolutely almost every-
P n
where – since it is just gn (x) on [−R, R]. The limit, f, of this series is therefore
n
in L1loc (R).
So, we only need show that f ∈ Lp (R) and that |f − Fn |p → 0 as n → ∞
R
n n
|fk |)p has
P P
where Fn = fk . By Minkowski’s inequality we know that hn = (
k=1 k=1
bounded L1 norm, since
1
n
X X
(2.192) k|hn |kLp 1 = k |fk |kLp . ≤ kfk kLp .
k=1 k

Thus, hn is an increasing sequence of functions in L1 (R) with bounded integral,


so by the Monotonicity Lemma it converges a.e. to a function h ∈ L1 (R). Since
|Fn |p ≤ h and |Fn |p → |f |p a.e. it follows by Dominated convergence that
1 X
(2.193) |f |p ∈ L1 (R), k|f |p kLp 1 ≤ kfn kLp
n

and hence f ∈ Lp (R). Applying the same reasoning to f − Fn which is the sum of
the series starting at term n + 1 gives the norm convergence:
X
(2.194) kf − Fn kLp ≤ kfk kLp → 0 as n → ∞.
k>n


74 2. THE LEBESGUE INTEGRAL

17. Lebesgue measure


In case anyone is interested in how to define Lebesgue measure from where we
are now we can just use the integral.
Definition 12. A set A ⊂ R is measurable if its characteristic function χA is
locally integrable. A measurable set A has finite measure if χA ∈ L1 (R) and then
Z
(2.195) µ(A) = χA

is the Lebesgue measure of A. If A is measurable but not of finite measure then


µ(A) = ∞ by definition.
Functions which are the finite sums of constant multiples of the characteristic
functions of measurable sets of finite measure are called ‘simple functions’ and
behave rather like out step functions. One of the standard approaches to Lebesgue
integration is to ‘complete’ the space of simple functions with respect to the integral.
We know immediately that any interval (a, b) is measurable (indeed whether
open, semi-open or closed) and has finite measure if and only if it is bounded –
then the measure is b − a. Some things to check:-
Proposition 24. The complement of a measurable set is measurable and any
countable union or countable intersection of measurable sets is measurable.
Proof. The first part follows from the fact that the constant function 1 is
locally integrable and hence χR\A = 1 − χA is locally integrable if and only if χA is
locally integrable.
Notice the relationship between a characteristic function and the set it defines:-
(2.196) χA∪B = max(χA , χB ), χA∩B = min(χA , χB ).
S
If we have a sequence of sets An then Bn = k≤n Ak is clearly an increasing
sequence of sets and
X
(2.197) χBn → χB , B = An
n

is an increasing sequence which converges pointwise (at each point it jumps to 1


somewhere and then stays or else stays at 0.) Now, if we multiply by χ[−N,N ] then
(2.198) fn = χ[−N,N ] χBn → χB∩[−N,N ]
is an increasing sequence of integrable functions – assuming that is that the Ak ’s are
measurable – with integral bounded above, by 2N. Thus by the S monotonicity lemma
the limit is integrable so χB is locally integrable and hence n An is measurable.
For countable intersections the argument is similar, with the sequence of char-
acteristic functions decreasing. 

Corollary 5. The (Lebesgue) measurable subsets of R form a collection, M,


of the power set of R, including ∅ and R which is closed under complements, count-
able unions and countable intersections.
Such a collection of subsets of a set X is called a ‘σ-algebra’ – so a σ-algebra
Σ in a set X is a collection of subsets of X containing X, ∅, the complement of
18. MEASURES ON THE LINE 75

any element and countable unions and intersections of any element. A (positive)
measure is usually defined as a map µ : Σ −→ [0, ∞] with µ(∅) = 0 and such that
[ X
(2.199) µ( En ) = µ(En )
n n

for any sequence {Em } of sets in Σ which are disjoint (in pairs).
As for Lebesgue measure a set A ∈ Σ is ‘measurable’ and if µ(A) is not of finite
measure it is said to have infinite measure – for instance R is of infinite measure
in this sense. Since the measure of a set is always non-negative (or undefined if it
isn’t measurable) this does not cause any problems and in fact Lebesgue measure
is countably additive provided as in (2.199) provided we allow ∞ as a value of the
measure. It is a good exercise to prove this!

18. Measures on the line


Going back to starting point for Lebesgue measure and the Lebesgue integral,
the discussion can be generalized, even in the one-dimensional case, by replacing
the measure of an interval by a more general function. As for the Stieltjes integral
this can be given by an increasing (meaning of course non-decreasing) function
m : R −→ R. For the discussion in this chapter to go through with only minor
changes we need to require that
m : R −→ R is non-decreasing and continuous from below
(2.200)
lim x ↑ ym(x) = m(y) ∀ y ∈ R.
Then we can define
(2.201) µ([a, b)) = m(b) − m(a).
For open and closed intervals we will expect that
(2.202) µ((a, b)) = lim m(x) − m(b), µ([a, b]) = m(a) − lim m(x).
x↓a x↓b

To pursue this, the first thing to check is that the analogue of Proposition 10 holds
in this case – namely if [a, b) is decomposed into a finite number of such semi-open
intervals by choice of interior points then
X
(2.203) µ([a, b)) = µ([ai , bi )).
i

Of course this follows from (2.201). Similarly, µ([a, b)) ≥ µ([A, B)) if A ≤ a and
b ≤ B, i.e. if [a, b) ⊂ [A, B). From this it follows that the analogue of Lemma 12
also holds with µ in place of Lebesgue length.
R
Then we can define the µ-integral, f dµ, of a step function, we do R not get
Proposition ?? since we might have intervals of µ length zero. Still, |f |dµ is
a seminorm. The definition of a µ-Lebesgue-integrable function (just called µ-
integrable usually), in terms of absolutely summable series with respect to this
seminorm, can be carried over as in Definition 7.
So far we have not used the continuity condition in (2.202), but now consider
the covering result Proposition 12. The first part has the same proof. For the
second part, the proof proceeds by making the intervals a little longer at the closed
end – to make them open. The continuity condition (2.202) ensures that this can be
done in such a way as to make the difference µ(bi ) − m(ai − i ) < µ([ai , bi )) + δ2−i
76 2. THE LEBESGUE INTEGRAL

for any δ > 0 by choosing i > 0 small enough. This covers [a, b − ] for  > 0 and
this allows the finite cover result to be applied to see that
X
(2.204) µ(b − ) − µ(a) ≤ µ([ai , bi )) + 2δ
i

for any δ > 0 and  > 0. Then taking the limits as  ↓ 0 and δ ↓ 0 gives the ‘outer’
intequality. So Proposition 12 carries over.
From this point the discussion of the µ integral proceeds in the same way with
a few minor exceptions – Corollary 3 doesn’t work again because there may be
intervals of length zero. Otherwise things proceed pretty smoothly right through.
The construction of Lebesgue measure, as in § 17, leasds to a σ-algebra Σµ , of
subsets of R which contains all the intervals, whether open, closed or mixed and all
the compact sets. You can check that the resulting countably additive measure is
a ‘Radon measure’ in that
X [
µ(B) = inf{ µ((ai bi )); B ⊂ (ai , bi )}, ∀ B ∈ Σµ ,
(2.205) i i
µ((a, b)) = sup{µ(K); K ⊂ (a, b), K compact}.
Conversely, every such positive Radon measure arises this way. Continuous func-
tions are locally µ-integrable
R and if µ(R) < ∞ (which corresponds to a choice of m
which is bounded) then f dµ < ∞ for every bounded continuous function which
vanishes at infinity.
Theorem 11. [Riesz’ other representation theorem] For any f ∈ (C0 (R)) there
are four uniquely determined (positive) Radon measures, µi , i = 1, . . . , 4 such that
µi (R) < ∞ and
Z Z Z Z
(2.206) f (u) = f dµ1 − f dµ2 + i f dµ3 − i f dµ4 .

How hard is this to prove? Well, a proof is outlined in the problems.

19. Higher dimensions (2011)


I do not actually plan to cover this in lectures, but put it in here in case someone
is interested (which you should be) or if I have time at the end of the course to
cover a problem in two or more dimensions (I have the Dirichlet problem in mind).
So, we want – with the advantage of a little more experience – to go back to
the beginning and define L1 (Rn ), L1 (Rn ), L2 (Rn ) and L2 (Rn ). In fact relatively
little changes but there are some things that one needs to check a little carefully.
The first hurdle is that I am not assuming that you have covered the Riemann
integral in higher dimensions. Fortunately we do not reall need that since we can
just iterated the one-dimensional Riemann integral for continuous functions. So,
define
(2.207) Cc (Rn ) = {u : Rn −→ C; continuous and such that u(x) = 0 for |x| > R}
where of course the R can depend on the element. Now, if we hold say the last
n−1 variables fixed, we get a continuous function of 1 variable which vanishes when
|x| > R :
(2.208) u(·, x2 , . . . , xn ) ∈ Cc (R) for each (x2 , . . . , xn ) ∈ Rn−1 .
19. HIGHER DIMENSIONS (2011) 77

So we can integrate it and get a function


Z
(2.209) I1 (x2 , . . . , xn ) = u(x, x1 , . . . , xn ), I1 : Rn−1 −→ C.
R

Lemma 19. For each u ∈ Cc (Rn ), I1 ∈ Cc (Rn−1 ).


Proof. Certainly if |(x2 , . . . , xn )| > R then u(·, x2 , . . . , xn ) ≡ 0 as a function of
the first variable and hence I1 = 0 in |(x2 , . . . , xn )| > R. The continuity follows from
the uniform continuity of a function on the compact set |x| ≤ R, |(x2 , . . . , xn ) ≤ R
of Rn . Thus given  > 0 there exists δ > 0 such that
(2.210) |x − x0 | < δ, |y − y 0 |Rn−1 < δ =⇒ |u(x, y) − u(x0 , y 0 )| < .
From the standard estimate for the Riemann integral,
Z R
0
(2.211) |I1 (y) − I1 (y )| ≤ |u(x, y) − u(x, y 0 )|dx ≤ 2R
−R
0
if |y − y | < δ. This implies the (uniform) continuity of I1 . Thus I1 ∈ Cc (Rn−2 ) 

The upshot of this lemma is that we can integrate again, and hence a total of
n times and so define the (iterated) Riemann integral as
Z Z R Z R Z R
(2.212) u(z)dz = ... u(x1 , x2 , x3 , . . . , xn )dx1 dx2 . . . dxn ∈ C.
Rn −R −R −R

Lemma 20. The interated Riemann integral is a well-defined linear map


(2.213) Cc (Rn ) −→ C
which satisfies
Z Z
(2.214) | u| ≤ |u| ≤ (2R)n sup |u| if u ∈ Cc (Rn ) and u(z) = 0 in |z| > R.

Proof. This follows from the standard estimate in one dimension. 

Now, one annoying thing is to check that the integral is independent of the
order of integration (although be careful with the signs here!) Fortunately we can
do this later and not have to worry.
Lemma 21. The iterated integral
Z
(2.215) kukL1 = |u|
Rn

is a norm on Cc (Rn ).
Proof. Straightforward. 

Definition 13. The space L1 (Rn ) (resp. L2 (Rn )) is defined to consist of those
functions f : Rn −→ C such that there exists a sequence {fn } which is absolutely
summable with respect to the L1 norm (resp. the L2 norm) such that
X X
(2.216) |fn (x)| < ∞ =⇒ fn (x) = f (x).
n n
78 2. THE LEBESGUE INTEGRAL

Proposition 25. If f ∈ L1 (Rn ) then |f | ∈ L1 (Rn ), Re f ∈ L1 (Rn ) and the


space L1 (Rn ) is lienar. Moreover if {fj } is an absolutely summable sequence in
Cc (Rn ) with respect to L1 such that
X X
(2.217) |fn (x)| < ∞ =⇒ fn (x) = 0
n n
R
then fn → 0 and in consequence the limit
Z X Z
(2.218) f= fn
Rn n→∞
1 n
is well-defined on L (R ).
Proof. Remarkably enough, nothing new is involved here. For the first part
this is pretty clear, but also holds for the second part. There is a lot to work
through, but it is all pretty much as in the one-dimensional case. 

20. Problems
Missing
Problem 2.1. Let’s consider an example of an absolutely summable sequence
of step functions. For the interval [0, 1) (remember there is a strong preference
for left-closed but right-open intervals for the moment) consider a variant of the
construction of the standard Cantor subset based on 3 proceeding in steps. Thus,
remove the ‘central interval [1/3, 2/3). This leave C1 = [0, 1/3) ∪ [2/3, 1). Then
remove the central interval from each of the remaining two intervals to get C2 =
[0, 1/9) ∪ [2/9, 1/3) ∪ [2/3, 7/9) ∪ [8/9, 1). Carry on in this way to define successive
sets Ck ⊂ Ck−1 , each consisting of a finite union of semi-open intervals. Now,
consider the series of step functions fk where fk (x) = 1 on Ck and 0 otherwise.
(1) Check that this is an absolutely
P summable series.
(2) For which x ∈ [0, 1) does |fk (x)| converge?
k
(3) Describe a function on [0, 1) which is shown to be Lebesgue integrable
(as defined in Lecture 4) by the existence of this series and compute its
Lebesgue integral.
(4) Is this function Riemann integrable (this is easy, not hard, if you check
the definition of Riemann integrability)?
(5) Finally consider the function g which is equal to one on the union of all
the intervals which are removed in the construction and zero elsewhere.
Show that g is Lebesgue integrable and compute its integral.
Problem 2.2. The covering lemma for R2 . By a rectangle we will mean a set
of the form [a1 , b1 ) × [a2 , b2 ) in R2 . The area of a rectangle is (b1 − a1 ) × (b2 − a2 ).
(1) We may subdivide a rectangle by subdividing either of the intervals –
replacing [a1 , b1 ) by [a1 , c1 ) ∪ [c1 , b1 ). Show that the sum of the areas of
rectangles made by any repeated subdivision is always the same as that
of the original.
(2) Suppose that a finite collection of disjoint rectangles has union a rectangle
(always in this same half-open sense). Show, and I really mean prove, that
the sum of the areas is the area of the whole rectange. Hint:- proceed by
subdivision.
20. PROBLEMS 79

(3) Now show that for any countable collection of disjoint rectangles contained
in a given rectange the sum of the areas is less than or equal to that of
the containing rectangle.
(4) Show that if a finite collection of rectangles has union containing a given
rectange then the sum of the areas of the rectangles is at least as large of
that of the rectangle contained in the union.
(5) Prove the extension of the preceeding result to a countable collection of
rectangles with union containing a given rectangle.
Problem 2.3. (1) Show that any continuous function on [0, 1] is the uni-
form limit on [0, 1) of a sequence of step functions. Hint:- Reduce to the
real case, divide the interval into 2n equal pieces and define the step func-
tions to take infimim of the continuous function on the corresponding
interval. Then use uniform convergence.
(2) By using the ‘telescoping trick’ show that any continuous function on [0, 1)
can be written as the sum
X
(2.219) fj (x) ∀ x ∈ [0, 1)
i
P
where the fj are step functions and |fj (x)| < ∞ for all x ∈ [0, 1).
j
(3) Conclude that any continuous function on [0, 1], extended to be 0 outside
this interval, is a Lebesgue integrable function on R and show that the
Lebesgue integral is equal to the Riemann integral.
Problem 2.4. If f and g ∈ L1 (R) are Lebesgue integrable functions on the
line show that
R
(1) If f (x) ≥ 0 a.e. then f R≥ 0. R
(2) If f (x) ≤ g(x) a.e. then f ≤ g.
(3) IfR f is complex
R valued then its real part, Re f, is Lebesgue integrable and
| Re f | ≤ |f |.
(4) For a general complex-valued Lebesgue integrable function
Z Z
(2.220) | f | ≤ |f |.

Hint: You can look up a proof of this


R easilyR enough, but the usual trick
is to choose θ ∈ [0, 2π) so that eiθ f = (eiθ f ) ≥ 0. Then apply the
preceeding estimate to g = eiθ f.
(5) Show that the integral is a continuous linear functional
Z
(2.221) : L1 (R) −→ C.

Problem 2.5. If I ⊂ R is an interval, including possibly (−∞, a) or (a, ∞),


we define Lebesgue integrability of a function f : I −→ C to mean the Lebesgue
integrability of
(
f (x) x ∈ I
(2.222) f˜ : R −→ C, f˜(x) =
0 x ∈ R \ I.
The integral of f on I is then defined to be
Z Z
(2.223) f = f˜.
I
80 2. THE LEBESGUE INTEGRAL

(1) Show that the space of such integrable functions on I is linear, denote it
L1 (I).
(2) Show that is f is integrable on I then so R is |f |.
(3) Show that if f is integrable on I and I |f | = 0 then f = 0 a.e. in the
sense that f (x) = 0 for all x ∈ I \ E where E ⊂ I is of measure zero as a
subset of R.
(4) Show that the set of null functions as in the preceeding question is a linear
R it N (I).
space, denote
(5) Show that I |f | defines a norm on L1 (I) = L1 (I)/N (I).
(6) Show that if f ∈ L1 (R) then
(
f (x) x ∈ I
(2.224) g : I −→ C, g(x) =
0 x∈R\I
is integrable on I.
(7) Show that the preceeding construction gives a surjective and continuous
linear map ‘restriction to I’
(2.225) L1 (R) −→ L1 (I).
(Notice that these are the quotient spaces of integrable functions modulo
equality a.e.)
Problem 2.6. Really continuing the previous one.
(1) Show that if I = [a, b) and f ∈ L1 (I) then the restriction of f to Ix = [x, b)
is an element of L1 (Ix ) for all a ≤ x < b.
(2) Show that the function
Z
(2.226) F (x) = f : [a, b) −→ C
Ix

is continuous.
(3) Prove that the function x−1 cos(1/x) is not Lebesgue integrable on the
interval (0, 1]. Hint: Think about it a bit and use what you have shown
above.
Problem 2.7. [Harder but still doable] Suppose f ∈ L1 (R).
(1) Show that for each t ∈ R the translates
(2.227) ft (x) = f (x − t) : R −→ C
are elements of L1 (R).
(2) Show that
Z
(2.228) lim |ft − f | = 0.
t→0

This is called ‘Continuity in the mean for integrable functions’. Hint: I


will add one!
(3) Conclude that for each f ∈ L1 (R) the map (it is a ‘curve’)
(2.229) R 3 t 7−→ [ft ] ∈ L1 (R)
is continuous.
20. PROBLEMS 81

Problem 2.8. In the last problem set you showed that a continuous function
on a compact interval, extended to be zero outside, is Lebesgue integrable. Using
this, and the fact that step functions are dense in L1 (R) show that the linear space
of continuous functions on R each of which vanishes outside a compact set (which
depends on the function) form a dense subset of L1 (R).
Problem 2.9. (1) If g : R −→ C is bounded and continuous and f ∈
L1 (R) show that gf ∈ L1 (R) and that
Z Z
(2.230) |gf | ≤ sup |g| · |f |.
R

(2) Suppose now that G ∈ C([0, 1]×[0, 1]) is a continuous function (I use C(K)
to denote the continuous functions on a compact metric space). Recall
from the preceeding discussion that we have defined L1 ([0, 1]). Now, using
the first part show that if f ∈ L1 ([0, 1]) then
Z
(2.231) F (x) = G(x, ·)f (·) ∈ C
[0,1]

(where · is the variable in which the integral is taken) is well-defined for


each x ∈ [0, 1].
(3) Show that for each f ∈ L1 ([0, 1]), F is a continuous function on [0, 1].
(4) Show that
(2.232) L1 ([0, 1]) 3 f 7−→ F ∈ C([0, 1])
is a bounded (i.e. continuous) linear map into the Banach space of con-
tinuous functions, with supremum norm, on [0, 1].
Problem 2.10. Let f : R −→ C be an element of L1 (R). Define
(
f (x) x ∈ [−L, L]
(2.233) fL (x) =
0 otherwise.
Show that fL ∈ L1 (R) and that |fL − f | → 0 as L → ∞.
R

Problem 2.11. Consider a real-valued function f : R −→ R which is locally


integrable in the sense that
(
f (x) x ∈ [−L, L]
(2.234) gL (x) =
0 x ∈ R \ [−L, L]
is Lebesgue integrable of each L ∈ N.
(1) Show that for each fixed L the function

gL (x) if gL (x) ∈ [−N, N ]

(N )
(2.235) gL (x) = N if gL (x) > N

−N if gL (x) < −N

is Lebesgue integrable.
R (N )
(2) Show that |gL − gL | → 0 as N → ∞.
(3) Show that there is a sequence, hn , of step functions such that
(2.236) hn (x) → f (x) a.e. in R.
82 2. THE LEBESGUE INTEGRAL

(4) Defining


0 x 6∈ [−L, L]

h (x) if h (x) ∈ [−N, N ], x ∈ [−L, L]
(N ) n n
(2.237) hn,L = .


N if hn (x) > N, x ∈ [−L, L]
−N if hn (x) < −N, x ∈ [−L, L]

R (N ) (N )
Show that |hn,L − gL | → 0 as n → ∞.

Problem 2.12. Show that L2 (R) is a Hilbert space.


First working with real functions, define L2 (R) as the set of functions f : R −→
R which are locally integrable and such that |f |2 is integrable.
(N ) (N )
(1) For such f choose hn and define gL , gL and hn by (2.234), (2.235) and
(2.237).
(N ) (N ) (N )
(2) Show using the sequence hn,L for fixed N and L that gL and (gL )2
(N ) (N )
are in L1 (R) and that |(hn,L )2 − (gL )2 | → 0 as n → ∞.
R
(N )
(3) Show that R(gL )2 ∈ L1 (R) and that |(gL )2 − (gL )2 | → 0 as N → ∞.
R

(4) Show that |(gL )2 − f 2 | → 0 as L → ∞.


(5) Show that f, g ∈ L2 (R) then f g ∈ L1 (R) and that
Z Z Z
(2.238) | f g| ≤ |f g| ≤ kf kL2 kgkL2 , kf k2L2 = |f |2 .

(6) Use these constructions to show that L2 (R) is a linear space.


(7) Conclude that the quotient space L2 (R) = L2 (R)/N , where N is the space
of null functions, is a real Hilbert space.
(8) Extend the arguments to the case of complex-valued functions.
Problem 2.13. Consider the sequence space
 
 X 
(2.239) h2,1 = c : N 3 j 7−→ cj ∈ C; (1 + j 2 )|cj |2 < ∞ .
 
j

(1) Show that


X
(2.240) h2,1 × h2,1 3 (c, d) 7−→ hc, di = (1 + j 2 )cj dj
j

is an Hermitian inner form which turns h2,1 into a Hilbert space.


(2) Denoting the norm on this space by k · k2,1 and the norm on l2 by k · k2 ,
show that
(2.241) h2,1 ⊂ l2 , kck2 ≤ kck2,1 ∀ c ∈ h2,1 .
Problem 2.14. In the separable case, prove Riesz Representation Theorem
directly.
Choose an orthonormal basis {ei } of the separable Hilbert space H. Suppose
T : H −→ C is a bounded linear functional. Define a sequence
(2.242) wi = T (ei ), i ∈ N.
21. SOLUTIONS TO PROBLEMS 83

(1) Now, recall that |T u| ≤ CkukH for some constant C. Show that for every
finite N,
N
X
(2.243) |wi |2 ≤ C 2 .
j=1
2
(2) Conclude that {wi } ∈ l and that
X
(2.244) w= wi ei ∈ H.
i

(3) Show that


(2.245) T (u) = hu, wiH ∀ u ∈ H and kT k = kwkH .
Problem 2.15. If f ∈ L1 (Rk × Rp ) show that there exists a set of measure
zero E ⊂ Rk such that
(2.246) x ∈ Rk \ E =⇒ gx (y) = f (x, y) defines gx ∈ L1 (Rp ),
gx defines an element F ∈ L1 (Rk ) and that
R
that F (x) =
Z Z
(2.247) F = f.
Rk Rk ×Rp

Note: These identities are usually written out as an equality of an iterated


integral and a ‘regular’ integral:
Z Z Z
(2.248) f (x, y) = f.
Rk Rp
It is often used to ‘exchange the order of integration’ since the hypotheses are
the same if we exchange the variables.

21. Solutions to problems


Problem 2.16. Suppose that f ∈ L1 (0, 2π) is such that the constants
Z
ck = f (x)e−ikx , k ∈ Z,
(0,2π)

satisfy
X
|ck |2 < ∞.
k∈Z
2
Show that f ∈ L (0, 2π).
Solution. So, this was a good bit harder than I meant it to be – but still in
principle solvable (even though no one quite got to the end).
First, (for half marks in fact!) we know that the ck exists, since fP∈ L1 (0, 2π)
and e−ikx is continuous so f e−ikx ∈ L1 (0, 2π) and then the condition |ck |2 < ∞
k
implies that the Fourier series does converge in L2 (0, 2π) so there is a function
1 X
(2.249) g= ck eikx .

k∈C

Now, what we want to show is that f = g a .e . since then f ∈ L2 (0, 2π).


84 2. THE LEBESGUE INTEGRAL

Set h = f − g ∈ L1 (0, 2π) since L2 (0, 2π) ⊂ L1 (0, 2π). It follows from (2.249)
that f and g have the same Fourier coefficients, and hence that
Z
(2.250) h(x)eikx = 0 ∀ k ∈ Z.
(0,2π)
So, we need to show that this implies that h = 0 a .e . Now, we can recall from
class that we showed (in the proof of the completeness of the Fourier basis of L2 )
that these exponentials are dense, in the supremum norm, in continuous functions
which vanish near the ends of the interval. Thus, by continuity of the integral we
know that
Z
(2.251) hg = 0
(0,2π)
for all such continuous functions g. We also showed at some point that we can
find such a sequence of continuous functions gn to approximate the characteristic
function of any interval χI . It is not true that gn → χI uniformly, but for any
integrable function h, hgn → hχI in L1 . So, the upshot of this is that we know a
bit more than (2.251), namely we know that
Z
(2.252) hg = 0 ∀ step functions g.
(0,2π)

So, now the trick is to show that (2.252)


R implies that h = 0 almost everywhere.
Well, this would follow if we know that (0,2π) |h| = 0, so let’s aim for that. Here
is the trick. Since g ∈ L1 we know that there is a sequence (the partial sums of
an absolutely convergent series) of step functions hn such that hn → g both in
L1 (0, 2π) and almost everywhere and also |hn | → |h| in both these senses. Now,
consider the functions
(
0 if hn (x) = 0
(2.253) sn (x) = hn (x)
|hn (x)| otherwise.

Clearly sn is a sequence of step functions, bounded (in absolute value by 1 in fact)


and such that sn hn = |hn |. Now, write out the wonderful identity
(2.254) |h(x)| = |h(x)| − |hn (x)| + sn (x)(hn (x) − h(x)) + sn (x)h(x).
Integrate this identity and then apply the triangle inequality to conclude that
Z Z Z
|h| = (|h(x)| − |hn (x)| + sn (x)(hn − h)
(0,2π) (0,2π) (0,2π)
(2.255) Z Z
≤ (||h(x)| − |hn (x)|| + |hn − h| → 0 as n → ∞.
(0,2π) (0,2π)

Here on the first line we have used (2.252) to see that the third term on the right
in (2.254) integrates to zero. Then the fact that |sn | ≤ 1 and the convergence
properties.
Thus in fact h = 0 a .e . so indeed f = g and f ∈ L2 (0, 2π). Piece of cake,
right! Mia culpa.
CHAPTER 3

Hilbert spaces

There are really three ‘types’ of Hilbert spaces (over C). The finite dimensional
ones, essentially just Cn , with which you are pretty familiar and two infinite dimen-
sional cases corresponding to being separable (having a countable dense subset) or
not. As we shall see, there is really only one separable infinite-dimensional Hilbert
space and that is what we are mostly interested in. Nevertheless some proofs (usu-
ally the nicest ones) work in the non-separable case too.
I will first discuss the definition of pre-Hilbert and Hilbert spaces and prove
Cauchy’s inequality and the parallelogram law. This can be found in all the lecture
notes listed earlier and many other places so the discussion here will be kept suc-
cinct. Another nice source is the book of G.F. Simmons, “Introduction to topology
and modern analysis”. I like it – but I think it is out of print.

1. pre-Hilbert spaces
A pre-Hilbert space, H, is a vector space (usually over the complex numbers
but there is a real version as well) with a Hermitian inner product
(, ) : H × H −→ C,
(3.1) (λ1 v1 + λ2 v2 , w) = λ1 (v1 , w) + λ2 (v2 , w),
(w, v) = (v, w)
for any v1 , v2 , v and w ∈ H and λ1 , λ2 ∈ C which is positive-definite
(3.2) (v, v) ≥ 0, (v, v) = 0 =⇒ v = 0.
Note that the reality of (v, v) follows from the second condition in (3.1), the posi-
tivity is an additional assumption as is the positive-definiteness.
The combination of the two conditions in (3.1) implies ‘anti-linearity’ in the
second variable
(3.3) (v, λ1 w1 + λ2 w2 ) = λ1 (v, w1 ) + λ2 (v, w2 )
which is used without comment below.
The notion of ‘definiteness’ for such an Hermitian inner product exists without
the need for positivity – it just means
(3.4) (u, v) = 0 ∀ v ∈ H =⇒ u = 0.
Lemma 22. If H is a pre-Hilbert space with Hermitian inner product (, ) then
1
(3.5) kuk = (u, u) 2
is a norm on H.
85
86 3. HILBERT SPACES

Proof. The first condition on a norm follows from (3.2). Absolute homogene-
ity follows from (3.1) since
(3.6) kλuk2 = (λu, λu) = |λ|2 kuk2 .
So, it is only the triangle inequality we need. This follows from the next lemma,
which is the Cauchy-Schwarz inequality in this setting – (3.8). Indeed, using the
‘sesqui-linearity’ to expand out the norm
(3.7) ku + vk2 = (u + v, u + v)
= kuk2 + (u, v) + (v, u) + kvk2 ≤ kuk2 + 2kukkvk + kvk2
= (kuk + kvk)2 .

Lemma 23. The Cauchy-Schwartz inequality,
(3.8) |(u, v)| ≤ kukkvk ∀ u, v ∈ H
holds in any pre-Hilbert space.
Proof. For any non-zero u, v ∈ H and s ∈ R positivity of the norm shows
that
(3.9) 0 ≤ ku + svk2 = kuk2 + 2s Re(u, v) + s2 kvk2 .
This quadratic polynomial is non-zero for s large so can have only a single minimum
at which point the derivative vanishes, i.e. it is where
(3.10) 2skvk2 + 2 Re(u, v) = 0.
Substituting this into (3.9) gives
(3.11) kuk2 − (Re(u, v))2 /kvk2 ≥ 0 =⇒ | Re(u, v)| ≤ kukkvk
which is what we want except that it is only the real part. However, we know that,
for some z ∈ C with |z| = 1, Re(zu, v) = Re z(u, v) = |(u, v)| and applying (3.11)
with u replaced by zu gives (3.8). 

2. Hilbert spaces
Definition 14. A Hilbert space H is a pre-Hilbert space which is complete
with respect to the norm induced by the inner product.
As examples we know that Cn with the usual inner product
n
X
(3.12) (z, z 0 ) = zj zj0
j=1

is a Hilbert space – since any finite dimensional normed space is complete. The
example we had from the beginning of the course is l2 with the extension of (3.12)
X∞
(3.13) (a, b) = aj bj , a, b ∈ l2 .
j=1

Completeness was shown earlier.


The whole outing into Lebesgue integration was so that we could have the
‘standard example’ at our disposal, namely
(3.14) L2 (R) = {u ∈ L1loc (R); |u|2 ∈ L1 (R)}/N
4. GRAM-SCHMIDT PROCEDURE 87

where N is the space of null functions. and the inner product is


Z
(3.15) (u, v) = uv.

3. Orthonormal sets
Two elements of a pre-Hilbert space H are said to be orthogonal if
(3.16) (u, v) = 0 ⇐⇒ u ⊥ v.
A sequence of elements ei ∈ H, (finite or infinite) is said to be orthonormal if
kei k = 1 for all i and (ei , ej ) = 0 for all i 6= j.
Proposition 26 (Bessel’s inequality). If ei , is an orthonormal sequence in a
pre-Hilbert space H, then
X
(3.17) |(u, ei )|2 ≤ kuk2 ∀ u ∈ H.
i

Proof. Start with the finite case, i = 1, . . . , N. Then, for any u ∈ H set
N
X
(3.18) v= (u, ei )ei .
i=1

This is supposed to be ‘the projection of u onto the span of the ei ’. Anyway,


computing away we see that
N
X
(3.19) (v, ej ) = (u, ei )(ei , ej ) = (u, ej )
i=1

using orthonormality. Thus, u − v ⊥ ej for all j so u − v ⊥ v and hence


(3.20) 0 = (u − v, v) = (u, v) − kvk2 .
Thus kvk2 = |(u, v)| and applying the Cauchy-Schwarz inequality we conclude that
kvk2 ≤ kvkkuk so either v = 0 or kvk ≤ kuk. Expanding out the norm (and
observing that all cross-terms vanish)
N
X
kvk2 = |(u, ei )|2 ≤ kuk2
i=1

which is (3.17).
In case the sequence is infinite this argument applies to any finite subsequence,
since it just uses orthonormality, so (3.17) follows by taking the supremum over
N. 

4. Gram-Schmidt procedure
Definition 15. An orthonormal sequence, {ei }, (finite or infinite) in a pre-
Hilbert space is said to be maximal if
(3.21) u ∈ H, (u, ei ) = 0 ∀ i =⇒ u = 0.
Theorem 12. Every separable pre-Hilbert space contains a maximal orthonor-
mal set.
88 3. HILBERT SPACES

Proof. Take a countable dense subset – which can be arranged as a sequence


{vj } and the existence of which is the definition of separability – and orthonormalize
it. Thus if v1 6= 0 set ei = v1 /kv1 k. Proceeding by induction we can suppose to
have found for a given integer n elements ei , i = 1, . . . , m, where m ≤ n, which are
orthonormal and such that the linear span
(3.22) sp(e1 , . . . , em ) = sp(v1 , . . . , vn ).
To show the inductive step observe that if vn+1 is in the span(s) in (3.22) then the
same ei ’a work for n + 1. So we may as well assume that the next element, vn+1 is
not in the span in (3.22). It follows that
n
X w
(3.23) w = vn+1 − (vn+1 , ej )ej 6= 0 so em+1 =
j=1
kwk

makes sense. By construction it is orthogonal to all the ealier ei ’s so adding em+1


gives the equality of the spans for n + 1.
Thus we may continue indefinitely, since in fact the only way the dense set
could be finite is if we were dealing with the space with one element, 0, in the first
place. There are only two possibilities, either we get a finite set of ei ’s or an infinite
sequence. In either case this must be a complete orthonormal sequence. That is,
we claim
(3.24) H 3 u ⊥ ej ∀ j =⇒ u = 0.
This uses the density of the vn ’s. There must exist a sequence wj where each wj
is a vn , such that wj → u in H, assumed to satisfy (3.24). Now, each each vn , and
hence each wj , is a finite linear combination of ek ’s so, by Bessel’s inequality
X X
(3.25) kwj k2 = |(wj , ek )|2 = |(u − wj , ek )|2 ≤ ku − wj k2
k k

where (u, ej ) = 0 for all j has been used. Thus kwj k → 0 and u = 0. 
Now, although a non-complete but separable Hilbert space has a maximal or-
thonormal sets, these are not much use without completeness.

5. Complete orthonormal bases


Definition 16. A maximal orthonormal sequence in a separable Hilbert space
is called a complete orthonormal basis.
This notion of basis is not quite the same as in the finite dimensional case
(although it is a legitimate extension of it).
Theorem 13. If {ei } is a complete orthonormal basis in a Hilbert space then
for any element u ∈ H the ‘Fourier-Bessel series’ converges to u :

X
(3.26) u= (u, ei )ei .
i=1

Proof. The sequence of partial sums of the Fourier-Bessel series


N
X
(3.27) uN = (u, ei )ei
i=1
7. PARALLELOGRAM LAW 89

is Cauchy. Indeed, if m < m0 then


0
m
X X
2
(3.28) kum0 − um k = |(u, ei )|2 ≤ |(u, ei )|2
i=m+1 i>m

which is small for large m by Bessel’s inequality. Since we are now assuming
completeness, um → w in H. However, (um , ei ) = (u, ei ) as soon as m > i and
|(w − un , ei )| ≤ kw − un k so in fact
(3.29) (w, ei ) = lim (um , ei ) = (u, ei )
m→∞
for each i. Thus in fact u − w is orthogonal to all the ei so by the assumed com-
pleteness of H must vanish. Thus indeed (3.26) holds. 

6. Isomorphism to l2
A finite dimensional Hilbert space is isomorphic to Cn with its standard inner
product. Similarly from the result above
Proposition 27. Any infinite-dimensional separable Hilbert space (over the
complex numbers) is isomorphic to l2 , that is there exists a linear map
(3.30) T : H −→ L2
which is 1-1, onto and satisfies (T u, T v)l2 = (u, v)H and kT ukl2 = kukH for all u,
v ∈ H.
Proof. Choose an orthonormal basis – which exists by the discussion above
and set
(3.31) T u = {(u, ej )}∞
j=1 .

This maps H into l2 by Bessel’s inequality. Moreover, it is linear since the entries
in the sequence are linear in u. It is 1-1 since T u = 0 implies (u, ej ) = 0 for all j
implies u = 0 by the assumed completeness of the orthonormal basis. It is surjective
since if {cj }∞
j=1 then

X
(3.32) u= cj ej
j=1

converges in H. This is the same argument as above – the sequence of partial sums is
Cauchy by Bessel’s inequality. Again by continuity of the inner product, T u = {cj }
so T is surjective.
The equality of the norms follows from equality of the inner products and the
latter follows by computation for finite linear combinations of the ej and then in
general by continuity. 

7. Parallelogram law
What exactly is the difference between a general Banach space and a Hilbert
space? It is of course the existence of the inner product defining the norm. In fact
it is possible to formulate this condition intrinsically in terms of the norm itself.
Proposition 28. In any pre-Hilbert space the parallelogram law holds –
(3.33) kv + wk2 + kv − wk2 = 2kvk2 + 2kwk2 , ∀ v, w ∈ H.
90 3. HILBERT SPACES

Proof. Just expand out using the inner product


(3.34) kv + wk2 = kvk2 + (v, w) + (w, v) + kwk2
and the same for kv − wk2 and see the cancellation. 

Proposition 29. Any normed space where the norm satisfies the parallelogram
law, (3.33), is a pre-Hilbert space in the sense that
1
kv + wk2 − kv − wk2 + ikv + iwk2 − ikv − iwk2

(3.35) (v, w) =
4
is a positive-definite Hermitian inner product which reproduces the norm.

Proof. A problem below. 

So, when we use the parallelogram law and completeness we are using the
essence of the Hilbert space.

8. Convex sets and length minimizer


The following result does not need the hypothesis of separability of the Hilbert
space and allows us to prove the subsequent results – especially Riesz’ theorem –
in full generality.

Proposition 30. If C ⊂ H is a subset of a Hilbert space which is


(1) Non-empty
(2) Closed
(3) Convex, in the sense that v1 , v1 ∈ C implies 21 (v1 + v2 ) ∈ C
then there exists a unique element v ∈ C closest to the origin, i.e. such that
(3.36) kvkH = inf kukH .
u∈C

Proof. By definition of inf there must exist a sequence {vn } in C such that
kvn k → d = inf u∈C kukH . We show that vn converges and that the limit is the
point we want. The parallelogram law can be written
(3.37) kvn − vm k2 = 2kvn k2 + 2kvm k2 − 4k(vn + vm )/2k2 .
Since kvn k → d, given  > 0 if N is large enough then n > N implies 2kvn k2 <
2d2 + 2 /2. By convexity, (vn + vm )/2 ∈ C so k(vn + vm )/2k2 ≥ d2 . Combining
these estimates gives
(3.38) n, m > N =⇒ kvn − vm k2 ≤ 4d2 + 2 − 4d2 = 2
so {vn } is Cauchy. Since H is complete, vn → v ∈ C, since C is closed. Moreover,
the distance is continuous so kvkH = limn→∞ kvn k = d.
Thus v exists and uniqueness follows again from the parallelogram law. If v
and v 0 are two points in C with kvk = kv 0 k = d then (v + v 0 )/2 ∈ C so
(3.39) kv − v 0 k2 = 2kvk2 + 2kv 0 k2 − 4k(v + v 0 )/2k2 ≤ 0 =⇒ v = v 0 .

9. ORTHOCOMPLEMENTS AND PROJECTIONS 91

9. Orthocomplements and projections


Proposition 31. If W ⊂ H is a linear subspace of a Hilbert space then
(3.40) W ⊥ = {u ∈ H; (u, w) = 0 ∀ w ∈ W }
is a closed linear subspace and W ∩ W ⊥ = {0}. If W is also closed then
(3.41) H = W ⊕ W⊥
meaning that any u ∈ H has a unique decomposition u = w + w⊥ where w ∈ W
and w⊥ ∈ W ⊥ .
Proof. That W ⊥ defined by (3.40) is a linear subspace follows from the lin-
earity of the condition defining it. If u ∈ W ⊥ and u ∈ W then u ⊥ u by the
definition so (u, u) = kuk2 = 0 and u = 0. Since the map H 3 u −→ (u, w) ∈ C is
continuous for each w ∈ H its null space, the inverse image of 0, is closed. Thus
\
(3.42) W⊥ = {(u, w) = 0}
w∈W

is closed.
Now, suppose W is closed. If W = H then W ⊥ = {0} and there is nothing to
show. So consider u ∈ H, u ∈
/ W. Consider
(3.43) C = u + W = {u0 ∈ H; u0 = u + w, w ∈ W }.
Then C is closed, since a sequence in it is of the form u0n = u + wn where wn is a
sequence in W and u0n converges if and only if wn converges. Also, C is non-empty,
since u ∈ C and it is convex since u0 = u + w0 and u00 = u + w00 in C implies
(u0 + u00 )/2 = u + (w0 + w00 )/2 ∈ C.
Thus the length minimization result above applies and there exists a unique
v ∈ C such that kvk = inf u0 ∈C ku0 k. The claim is that this v is perpendicular to
W – draw a picture in two real dimensions! To see this consider an aritrary point
w ∈ W and λ ∈ C then v + λw ∈ C and
(3.44) kv + λwk2 = kvk2 + 2 Re(λ(v, w)) + |λ|2 kwk2 .
Choose λ = teiθ where t is real and the phase is chosen so that eiθ (v, w) = |(v, w)| ≥
0. Then the fact that kvk is minimal means that
kvk2 + 2t|(v, w))| + t2 kwk2 ≥ kvk2 =⇒
(3.45)
t(2|(v, w)| + tkwk2 ) ≥ 0 ∀ t ∈ R =⇒ |(v, w)| = 0
which is what we wanted to show.
Thus indeed, given u ∈ H \ W we have constructed v ∈ W ⊥ such that u =
v + w, w ∈ W. This is (3.41) with the uniqueness of the decomposition already
shown since it reduces to 0 having only the decomposition 0 + 0 and this in turn is
W ∩ W ⊥ = {0}. 
Since the construction in the preceding proof associates a unique element in W,
a closed linear subspace, to each u ∈ H, it defines a map
(3.46) ΠW : H −→ W.
This map is linear, by the uniqueness since if ui = vi + wi , wi ∈ W, (vi , wi ) = 0 are
the decompositions of two elements then
(3.47) λ1 u1 + λ2 u2 = (λ1 v1 + λ2 v2 ) + (λ1 w1 + λ2 w2 )
92 3. HILBERT SPACES

must be the corresponding decomposition. Moreover ΠW w = w for any w ∈ W


and kuk2 = kvk2 + kwk2 , Pythagorus’ Theorem, shows that
(3.48) Π2W = ΠW , kΠW uk ≤ kuk =⇒ kΠW k ≤ 1.
Thus, projection onto W is an operator of norm 1 (unless W = {0}) equal to its
own square. Such an operator is called a projection or sometimes an idempotent
(which sounds fancier).
Lemma 24. If {ej } is any finite or countable orthonormal set in a Hilbert space
then the orthogonal projection onto the closure of the span of these elements is
X
(3.49) Pu = (u, ek )ek .

Proof. We know that the series in (3.49) converges and defines a bounded
linear operator of norm at most one by Bessel’s inequality. Clearly P 2 = P by the
same argument. If W is the closure of the span then (u−P u) ⊥ W since (u−P u) ⊥
ek for each k and the inner product is continuous. Thus u = (u − P u) + P u is the
orthogonal decomposition with respect to W. 

10. Riesz’ theorem


The most important application of these results is to prove Riesz’ representation
theorem (for Hilbert space, there is another one to do with measures).
Theorem 14. If H is a Hilbert space then for any continuous linear functional
T : H −→ C there exists a unique element φ ∈ H such that
(3.50) T (u) = (u, φ) ∀ u ∈ H.
Proof. If T is the zero functional then w = 0 satisfies (3.50). Otherwise there
exists some u0 ∈ H such that T (u0 ) 6= 0 and then there is some u ∈ H, namely
u = u0 /T (u0 ) will work, such that T (u) = 1. Thus
(3.51) C = {u ∈ H; T (u) = 1} = T −1 ({1}) 6= ∅.
The continuity of T and the second form shows that C is closed, as the inverse
image of a closed set under a continuous map. Moreover C is convex since
(3.52) T ((u + u0 )/2) = (T (u) + T (u0 ))/2.
Thus, by Proposition 30, there exists an element v ∈ C of minimal length.
Notice that C = {u + v; v ∈ N } where N = T −1 ({0}) is the null space of T.
Thus, as in Proposition 31 above, v is orthogonal to N. In this case it is the unique
element orthogonal to N with T (v) = 1.
Now, for any u ∈ H,
(3.53)
u−T (u)v satisfies T (u−T (u)v) = T (u)−T (u)T (v) = 0 =⇒ u = w+T (u)v, w ∈ N.
Then, (u, v) = T (u)kvk2 since (w, v) = 0. Thus if φ = v/kvk2 then
(3.54) u = w + (u, φ)v =⇒ T (u) = (u, φ)T (v) = (u, φ).

11. ADJOINTS OF BOUNDED OPERATORS 93

11. Adjoints of bounded operators


As an application of Riesz’ we can see that to any bounded linear operator on
a Hilbert space
(3.55) A : H −→ H, kAukH ≤ CkukH ∀ u ∈ H
there corresponds a unique adjoint operator.
Proposition 32. For any bounded linear operator A : H −→ H on a Hilbert
space there is a unique bounded linear operator A∗ : H −→ H such that
(3.56) (Au, v)H = (u, A∗ v)H ∀ u, v ∈ H and kAk = kA∗ k.
Proof. To see the existence of A∗ v we need to work out what A∗ v ∈ H should
be for each fixed v ∈ H. So, fix v in the desired identity (3.56), which is to say
consider
(3.57) H 3 u −→ (Au, v) ∈ C.
This is a linear map and it is clearly bounded, since
(3.58) |(Au, v)| ≤ kAukH kvkH ≤ (kAkkvkH )kukH .
Thus it is a continuous linear functional on H which depends on v. In fact it is just
the composite of two continuous linear maps
u7−→Au w7−→(w,v)
(3.59) H −→ H −→ C.
By Riesz’ theorem there is a unique element in H, which we can denote A∗ v (since
it only depends on v) such that
(3.60) (Au, v) = (u, A∗ v) ∀ u ∈ H.
Now this defines the map A∗ : H −→ H but we need to check that it is linear and
continuous. Linearity follows from the uniqueness part of Riesz’ theorem. Thus if
v1 , v2 ∈ H and c1 , c2 ∈ C then
(3.61) (Au, c1 v1 + c2 v2 ) = c1 (Au, v1 ) + c2 (Au, v2 )
= c1 (u, A∗ v1 ) + c2 (u, A∗ v2 ) = (u, c1 A∗ v2 + c2 A∗ v2 )
where we have used the definitions of A∗ v1 and A∗ v2 – by uniqueness we must have
A∗ (c1 v1 + c2 v2 ) = c1 A∗ v1 + c2 A∗ v2 .
Since we know the optimality of Cauchy’s inequality
(3.62) kvkH = sup |(u, v)|
kuk=1

it follows that
(3.63) kA∗ vk = sup |(u, A∗ v)| = sup |(Au, v)| ≤ kAkkvk.
kuk=1 kuk=1

So in fact
(3.64) kA∗ k ≤ kAk
which shows that A∗ is bounded.
The defining identity (3.56) also shows that (A∗ )∗ = A so the reverse equality
in (3.64) also holds and so
(3.65) kA∗ k = kAk.

94 3. HILBERT SPACES

12. Compactness and equi-small tails


A compact subset in a general metric space is one with the property that any
sequence in it has a convergent subsequence, with its limit in the set. You will recall,
with pleasure no doubt, the equivalence of this condition to the (more general since
it makes good sense in an arbitrary topological space) covering condition, that any
open cover of the set has a finite subcover. So, in a separable Hilbert space the
notion of a compact set is already fixed. We want to characterize it, actually in
several ways.
A general result in a metric space is that any compact set is both closed and
bounded, so this must be true in a Hilbert space. The Heine-Borel theorem gives a
converse to this, for Rn or Cn (and hence in any finite dimensional normed space)
in which any closed and bounded set is compact. Also recall that the convergence
of a sequence in Cn is equivalent to the convergence of the n sequences given by its
components and this is what is used to pass first from R to C and then to Cn . All
of this fails in infinite dimensions and we need some condition in addition to being
bounded and closed for a set to be compact.
To see where this might come from, observe that
Lemma 25. In any metric space a set, S, consisting of the points of a convergent
sequence, s : N −→ M, together with its limit, s, is compact.
Proof. The set here is the image of the sequence, thought of as a map from
the integers into the metric space, together with the limit (which might or might
not already be in the image of the sequence). Certainly this set is bounded, since
the distance from the intial point is bounded. Moreover it is closed. Indeed, the
complement M \ S is open – if p ∈ M \ S then it is not the limit of the sequence,
so for some  > 0 and some N, if n > N then s(n) ∈ / B(p, ). Shrinking  further if
necessary, we can make sure that all the s(k) for k ≤ N are not in the ball either
– since they are each at a positive distance from p. Thus B(p, ) ⊂ M \ S.
Finally, S is compact since any sequence in S has a convergent subsequence.
To see this, observe that a sequence {tj } in S either has a subsequence converging
to the limit s of the original sequence or it does not. So we only need consider the
latter case, but this means that for some  > 0 d(tj , s) > ; but then tj takes values
in a finite set, since S \ B(s, ) is finite – hence some value is repeated infinitely
often and there is a convergent subsequence. 

Lemma 26. The image of a convergent sequence in a Hilbert space is a set with
equi-small tails with respect to any orthonormal sequence, i.e. if ek is an othonormal
sequence and un → u is a convergent sequence then given  > 0 there exists N such
that
X
(3.66) |(un , ek )|2 < 2 ∀ n.
k>N

Proof. Bessel’s inequality shows that for any u ∈ H,


X
(3.67) |(u, ek )|2 ≤ kuk2 .
k

The convergence of this series means that (3.66) can be arranged for any single
element un or the limit u by choosing N large enough, thus given  > 0 we can
12. COMPACTNESS AND EQUI-SMALL TAILS 95

choose N 0 so that
X
(3.68) |(u, ek )|2 < 2 /2.
k>N 0
Consider the closure of the subspace spanned by the ek with k > N. The
orthogonal projection onto this space (see Lemma 24) is
X
(3.69) PN u = (u, ek )ek .
k>N

Then the convergence un → u implies the convergence in norm kPN un k → kPN uk,
so
X
(3.70) kPN un k2 = |(u, ek )|2 < 2 , n > n0 .
k>N

So, we have arranged (3.66) for n > n0 for some N. This estimate remains valid if
N is increased – since the tails get smaller – and we may arrange it for n ≤ n0 by
chossing N large enough. Thus indeed (3.66) holds for all n if N is chosen large
enough. 
This suggest one useful characterization of compact sets in a separable Hilbert
space.
Proposition 33. A set K ⊂ H in a separable Hilbert space is compact if
and only if it is bounded, closed and has equi-small tails with respect to any (one)
complete orthonormal basis.
Proof. We already know that a compact set in a metric space is closed and
bounded. Suppose the equi-smallness of tails condition fails with respect to some
orthonormal basis ek . This means that for some  > 0 and all p there is an element
unp ∈ K, such that
X
(3.71) |(unp , ek )|2 ≥ 2 .
k>p

Consider the subsequence {unp } generated this way. No subsequence of it can have
equi-small tails (recalling that the tail decreases with p). Thus, by Lemma 26, it
cannot have a convergent subsequence, so K cannot be compact in this case.
Thus we have proved the equi-smallness of tails condition to be necessary for
the compactness of a closed, bounded set. It remains to show that it is sufficient.
So, suppose K is closed, bounded and satisfies the equi-small tails condition
with respect to an orthonormal basis ek and {un } is a sequence in K. We only
need show that {un } has a Cauchy subsequence, since this will converge (H being
complete) and the limit will be in K (since it is closed). Consider each of the
sequences of coefficients (un , ek ) in C. Here k is fixed. This sequence is bounded:
(3.72) |(un , ek )| ≤ kun k ≤ C
by the boundedness of K. So, by the Heine-Borel theorem, there is a subsequence
of unl such that (unl , ek ) converges as l → ∞.
We can apply this argument for each k = 1, 2, . . . . First extracting a subse-
quence of {un } so that the sequence (un , e1 ) converges ‘along this subsequence’.
Then extract a subsequence of this subsequence so that (un , e2 ) also converges
along this sparser subsequence, and continue inductively. Then pass to the ‘diago-
nal’ subsequence of {un } which has kth entry the kth term in the kth subsequence.
96 3. HILBERT SPACES

It is ‘eventually’ a subsequence of each of the subsequences previously constructed


– meaning it coincides with a subsequence from some point onward (namely the
kth term onward for the kth subsquence). Thus, for this subsequence each of the
(unl , ek ) converges.
Now, let’s relabel this subsequence vn for simplicity of notation and consider
Bessel’s identity (the orthonormal set ek is complete by assumption) for the differ-
ence
X X
kvn − vn+l k2 = |(vn − vn+l , ek )|2 + |(vn − vn+l , ek )|2
k≤N k>N
(3.73) X X X
2
≤ |(vn − vn+l , ek )| + 2 |(vn , ek )|2 + 2 |(vn+l , ek )|2
k≤N k>N k>N

where the parallelogram law on C has been used. To make this sum less than 2
we may choose N so large that the last two terms are less than 2 /2 and this may
be done for all n and l by the equi-smallness of the tails. Now, choose n so large
that each of the terms in the first sum is less than 2 /2N, for all l > 0 using the
Cauchy condition on each of the finite number of sequence (vn , ek ). Thus, {vn } is
a Cauchy subsequence of {un } and hence as already noted convergent in K. Thus
K is indeed compact. 

13. Finite rank operators


Now, we need to starting thinking a little more seriously about operators on a
Hilbert space.
Definition 17. An operator T ∈ B(H) is of finite rank if its range has fi-
nite dimension (and that dimension is called the rank of T ); the set of finite rank
operators is denoted R(B).
Why not F(B)? Because we want to use this for the Fredholm operators.
Clearly the sum of two operators of finite rank has finite rank, since the range
is contained in the sum of the ranges (but is often smaller):
(3.74) (T1 + T2 )u ∈ Ran(T1 ) + Ran(T2 ) ∀ u ∈ H.
Since the range of a constant multiple of T is contained in the range of T it follows
that the finite rank operators form a linear subspace of B(H).
What does a finite rank operator look like? It really looks like a matrix.
Lemma 27. If T : H −→ H has finite rank then there is a finite orthonormal
set {ek }L
k=1 in H such that
L
X
(3.75) Tu = cij (u, ej )ei .
i,j=1

Proof. By definition, the range of T, R = T (H) is a finite dimensional sub-


space. So, it has a basis which we can diagonalize in H to get an orthonormal basis,
ei , i = 1, . . . , p. Now, since this is a basis of the range, T u can be expanded relative
to it for any u ∈ H :
p
X
(3.76) Tu = (T u, ei )ei .
i=1
13. FINITE RANK OPERATORS 97

On the other hand, the map u −→ (T u, ei ) is a continuous linear functional on H,


so (T u, ei ) = (u, vi ) for some vi ∈ H; notice in fact that vi = T ∗ ei . This means the
formula (3.76) becomes
p
X
(3.77) Tu = (u, vi )ei .
i=1
Now, we can apply Gram-Schmidt to orthonormalize the sequence e1 , . . . , ep , v1 . . . , vp
resulting in e1 , . . . , eL . This means that eact vi is a linear combination which we
can write as
XL
(3.78) vi = cij ej .
j=1

Inserting this into (3.77) gives (3.75) (where the constants for i > p are zero). 
It is clear that
(3.79) B ∈ B(H) and T ∈ R(B) then BT ∈ R(B).
Indeed, the range of BT is the range of B restricted to the range of T and this is
certainly finite dimensional since it is spanned by the image of a basis of Ran(T ).
Similalry T B ∈ R(H) since the range of T B is contained in the range of T. Thus
we have in fact proved most of
Proposition 34. The finite rank operators form a ∗-closed ideal in B(H),
which is to say a linear subspace such that
(3.80) B1 , B2 ∈ B(H), T ∈ R(H) =⇒ B1 T B2 , T ∗ ∈ R(H).
Proof. It is only left to show that T ∗ is of finite rank if T is, but this is an
immediate consequence of Lemma 27 since if T is given by (3.75) then
N
X
(3.81) T ∗u = cij (u, ei )ej
i=1
is also of finite rank. 
Lemma 28 (Row rank=Colum rank). For any finite rank operator on a Hilbert
space, the dimension of the range of T is equal to the dimension of the range of T ∗ .
Proof. From the formula (3.77) for a finite rank operator, it follows that the
vi , i = 1, . . . , p must be linearly independent – since the ei form a basis for the
range and a linear relation between the vi would show the range had dimension less
than p. Thus in fact the null space of T is precisely the orthocomplement of the
span of the vi – the space of vectors orthogonal to each vi . Since
Xp
(T u, w) = (u, vi )(ei , w) =⇒
i=1
p
X
(3.82) (w, T u) = (vi , u)(w, ei ) =⇒
i=1
p
X
T ∗w = (w, ei )vi
i=1
the range of T ∗ is the span of the vi , so is also of dimension p. 
98 3. HILBERT SPACES

14. Compact operators


Definition 18. An element K ∈ B(H), the bounded operators on a separable
Hilbert space, is said to be compact (the old terminology was ‘totally bounded’
or ‘completely continuous’) if the image of the unit ball is precompact, i.e. has
compact closure – that is if the closure of K{u ∈ H; kukH ≤ 1} is compact in H.
Notice that in a metric space, to say that a set has compact closure is the same
as saying it is contained in a compact set.
Proposition 35. An operator K ∈ B(H), bounded on a separable Hilbert space,
is compact if and only if it is the limit of a norm-convergent sequence of finite rank
operators.
Proof. So, we need to show that a compact operator is the limit of a conver-
gent sequence of finite rank operators. To do this we use the characterizations of
compact subsets of a separable Hilbert space discussed earlier. Namely, if {ei } is
an orthonormal basis of H then a subset I ⊂ H is compact if and only if it is closed
and bounded and has equi-small tails with respect to {ei }, meaning given  > 0
there exits N such that
X
(3.83) |(v, ei )|2 < 2 ∀ v ∈ I.
i>N

Now we shall apply this to the set K(B(0, 1)) where we assume that K is
compact (as an operator, don’t be confused by the double usage, in the end it turns
out to be constructive) – so this set is contained in a compact set. Hence (3.83)
applies to it. Namely this means that for any  > 0 there exists n such that
X
(3.84) |(Ku, ei )|2 < 2 ∀ u ∈ H, kukH ≤ 1.
i>n

For each n consider the first part of these sequences and define
X
(3.85) Kn u = (Ku, ei )ei .
k≤n

This is clearly a linear operator and has finite rank – since its range is contained in
the span of the first n elements of {ei }. Since this is an orthonormal basis,
X
(3.86) kKu − Kn uk2H = |(Ku, ei )|2
i>n

Thus (3.84) shows that kKu − Kn ukH ≤ . Now, increasing n makes kKu − Kn uk
smaller, so given  > 0 there exists n such that for all N ≥ n,
(3.87) kK − KN kB = sup kKu − Kn ukH ≤ .
kuk≤1

Thus indeed, Kn → K in norm and we have shown that the compact operators are
contained in the norm closure of the finite rank operators.
For the converse we assume that Tn → K is a norm convergent sequence in
B(H) where each of the Tn is of finite rank – of course we know nothing about the
rank except that it is finite. We want to conclude that K is compact, so we need to
show that K(B(0, 1)) is precompact. It is certainly bounded, by the norm of K. By
a result above on compactness of sets in a separable Hilbert space we know that it
suffices to prove that the closure of the image of the unit ball has uniformly small
15. WEAK CONVERGENCE 99

tails. Let ΠN be the orthogonal projection off the first N elements of a complete
orthonormal basis {ek } – so
X
(3.88) u= (u, ek )ek + ΠN u.
k≤N

Then we know that kΠN k = 1 (assuming the Hilbert space is infinite dimensional)
and kΠN uk is the ‘tail’. So what we need to show is that given  > 0 there exists
n such that
(3.89) kuk ≤ 1 =⇒ kΠN Kuk < .
Now,
(3.90) kΠN Kuk ≤ kΠN (K − Tn )uk + kΠN Tn uk
so choosing n large enough that kK − Tn k < /2 and then using the compactness
of Tn (which is finite rank) to choose N so large that
(3.91) kuk ≤ 1 =⇒ kΠN Tn uk ≤ /2
shows that (3.89) holds and hence K is compact. 
Proposition 36. For any separable Hilbert space, the compact operators form
a closed and ∗-closed two-sided ideal in B(H).
Proof. In any metric space (applied to B(H)) the closure of a set is closed,
so the compact operators are closed being the closure of the finite rank operators.
Similarly the fact that it is closed under passage to adjoints follows from the same
fact for finite rank operators. The ideal properties also follow from the correspond-
ing properties for the finite rank operators, or we can prove them directly anyway.
Namely if B is bounded and T is compact then for some c > 0 (namely 1/kBk
unless it is zero) cB maps B(0, 1) into itself. Thus cT B = T cB is compact since
the image of the unit ball under it is contained in the image of the unit ball under
T ; hence T B is also compact. Similarly BT is compact since B is continuous and
then
(3.92) BT (B(0, 1)) ⊂ B(T (B(0, 1))) is compact
since it is the image under a continuous map of a compact set. 

15. Weak convergence


It is convenient to formalize the idea that a sequence be bounded and that each
of the (un , ek ), the sequence of coefficients of some particular Fourier-Bessel series,
should converge.
Definition 19. A sequence, {un }, in a Hilbert space, H, is said to converge
weakly to an element u ∈ H if it is bounded in norm and (uj , v) → (u, v) converges
in C for each v ∈ H. This relationship is written
(3.93) un * u.
In fact as we shall see below, the assumption that kun k is bounded and that u
exists are both unnecessary. That is, a sequence converges weakly if and only if
(un , v) converges in C for each v ∈ H. Conversely, there is no harm in assuming
it is bounded and that the ‘weak limit’ u ∈ H exists. Note that the weak limit is
unique since if u and u0 both have this property then (u − u0 , v) = limn→∞ (un , v) −
limn→∞ (un , v) = 0 for all v ∈ H and setting v = u − u0 it follows that u = u0 .
100 3. HILBERT SPACES

Lemma 29. A (strongly) convergent sequence is weakly convergent with the


same limit.
Proof. This is the continuity of the inner product. If un → u then
(3.94) |(un , v) − (u, v)| ≤ kun − ukkvk → 0
for each v ∈ H shows weak convergence. 
Lemma 30. For a bounded sequence in a separable Hilbert space, weak con-
vergence is equivalent to component convergence with respect to an orthonormal
basis.
Proof. Let ek be an orthonormal basis. Then if un is weakly convergent
it follows immediately that (un , ek ) → (u, ek ) converges for each k. Conversely,
suppose this is true for a bounded sequence, just that (un , ek ) → ck in C for each
k. The norm boundedness and Bessel’s inequality show that
X X
(3.95) |ck |2 = lim |(un , ek )|2 ≤ C 2 sup kun k2
n→∞ n
k≤p k≤p

for all p. Thus in fact {ck } ∈ l2 and hence


X
(3.96) u= ck ek ∈ H
k

by the completeness of H. Clearly (un , ek ) → (u, ek ) for each k. It remains to show


that (un , v) → (u, v) for all v ∈ H. This is certainly true for any finite linear
combination of the ek and for a general v we can write

(3.97) (un , v) − (u, v) = (un , vp ) − (u, vp ) + (un , v − vp ) − (u, v − vp ) =⇒


|(un , v) − (u, v)| ≤ |(un , vp ) − (u, vp )| + 2Ckv − vp k
P
where vp = (v, ek )ek is a finite part of the Fourier-Bessel series for v and C is a
k≤p
bound for kun k. Now the convergence vp → v implies that the last term in (3.97)
can be made small by choosing p large, independent of n. Then the second last term
can be made small by choosing n large since vp is a finite linear combination of the
ek . Thus indeed, (un , v) → (u, v) for all v ∈ H and it follows that un converges
weakly to u. 
Proposition 37. Any bounded sequence {un } in a separable Hilbert space has
a weakly convergent subsequence.
This can be thought of as an analogue in infinite dimensions of the Heine-Borel
theorem if you say ‘a bounded closed subset of a separable Hilbert space is weakly
compact’.
Proof. Choose an orthonormal basis {ek } and apply the procedure in the
proof of Proposition 33 to extract a subsequence of the given bounded sequence
such that (unp , ek ) converges for each k. Now apply the preceeding Lemma to
conclude that this subsequence converges weakly. 
Lemma 31. For a weakly convergent sequence un * u
(3.98) kuk ≤ lim inf kun k.
15. WEAK CONVERGENCE 101

Proof. Choose an orthonormal basis ek and observe that


X X
(3.99) |(u, ek )|2 = lim |(un , ek )|2 .
n→∞
k≤p k≤p

2
The sum on the right is bounded by kun k independently of p so
X
(3.100) ku, ek k2 ≤ lim inf kun k2
n
k≤p

by the definition of lim inf . Then let p → ∞ to conclude that


(3.101) kuk2 ≤ lim inf kun k2
n

from which (3.98) follows. 

Lemma 32. An operator K ∈ B(H) is compact if and only if the image Kun
of any weakly convergent sequence {un } in H is strongly, i.e. norm, convergent.
Proof. First suppose that un * u is a weakly convergent sequence in H and
that K is compact. We know that kun k < C is bounded so the sequence Kun
is contained in CK(B(0, 1)) and hence in a compact set (clearly if D is compact
then so is cD for any constant c.) Thus, any subsequence of Kun has a convergent
subseqeunce and the limit is necessarily Ku since Kun * Ku (true for any bounded
operator by computing
(3.102) (Kun , v) = (un , K ∗ v) → (u, K ∗ v) = (Ku, v).)
But the condition on a sequence in a metric space that every subsequence of it has
a subsequence which converges to a fixed limit implies convergence. (If you don’t
remember this, reconstruct the proof: To say a sequence vn does not converge to
v is to say that for some  > 0 there is a subsequence along which d(vnk , v) ≥ .
This is impossible given the subsequence of subsequence condition (converging to
the fixed limit v.))
Conversely, suppose that K has this property of turning weakly convergent
into strongly convergent sequences. We want to show that K(B(0, 1)) has compact
closure. This just means that any sequence in K(B(0, 1)) has a (strongly) con-
vergent subsequence – where we do not have to worry about whether the limit is
in the set or not. Such a sequence is of the form Kun where un is a sequence in
B(0, 1). However we know that the ball is weakly compact, that is we can pass to
a subsequence which converges weakly, unj * u. Then, by the assumption of the
Lemma, Kunj → Ku converges strongly. Thus un does indeed have a convergent
subsequence and hence K(B(0, 1)) must have compact closure. 

As noted above, it is not really necessary to assume that a sequence in a Hilbert


space is bounded, provided one has the Uniform Boundedness Principle, Theorem 3,
at the ready.
Proposition 38. If un ∈ H is a sequence in a Hilbert space and for all v ∈ H
(3.103) (un , v) → F (v) converges in C
then kun kH is bounded and there exists w ∈ H such that un * w (converges
weakly).
102 3. HILBERT SPACES

Proof. Apply the Uniform Boundedness Theorem to the continuous function-


als
(3.104) Tn (u) = (u, un ), Tn : H −→ C
where we reverse the order to make them linear rather than anti-linear. Thus, each
set |Tn (u)| is bounded in C since it is convergent. It follows from the Uniform
Boundedness Principle that there is a bound
(3.105) kTn k ≤ C.
However, this norm as a functional is just kTn k = kun kH so the original sequence
must be bounded in H. Define T : H −→ C as the limit for each u :
(3.106) T (u) = lim Tn (u) = lim (u, un ).
n→∞ n→∞

This exists for each u by hypothesis. It is a linear map and from (3.105) it is
bounded, kT k ≤ C. Thus by the Riesz Representation theorem, there exists w ∈ H
such that
(3.107) T (u) = (u, w) ∀ u ∈ H.
Thus (un , u) → (w, u) for all u ∈ H so un * w as claimed. 

16. The algebra B(H)


Recall the basic properties of the Banach space, and algebra, of bounded oper-
ators B(H) on a separable Hilbert space H. In particular that it is a Banach space
with respect to the norm
(3.108) kAk = sup kAukH
kukH =1

and that the norm satisfies


(3.109) kABk ≤ kAkkBk
as follows from the fact that
kABuk ≤ kAkkBuk ≤ kAkkBkkuk.
Consider the set of invertible elements:
(3.110) GL(H) = {A ∈ B(H); ∃ B ∈ H(H), BA = AB = Id}.
Note that this is equivalent to saying A is 1-1 and onto in view of the Open Mapping
Theorem, Theorem 4.
This set is open, to see this consider a neighbourhood of the identity.
Lemma 33. If A ∈ B(H) and kAk < 1 then
(3.111) Id −A ∈ GL(H).
Proof. This follows from the convergence of the Neumann series for A. If
kAk < 1 then kAj k ≤ kAkj , from (3.109), and it follows that the Neumann series
X
(3.112) B= Aj
j=0∞
16. THE ALGEBRA B(H) 103

1
kAj k converges. Since B(H) is a Banach
P
is absolutely summable in B(H) since
j=0
space, the sum converges. Moreover by the continuity of the product with respect
to the norm
Xn n+1
X
(3.113) AB = A lim Aj = lim Aj = B − Id
n→∞ n→∞
j=0 j=1

and similarly BA = B − Id . Thus (Id −A)B = B(Id −A) = Id shows that B is a


(and hence the) 2-sided inverse of Id −A. 
Proposition 39. The invertible elements form an open subset GL(H) ⊂ B(H).
Proof. Suppose G ∈ GL(H), meaning it has a two-sided (and unique) inverse
G−1 ∈ B(H) :
(3.114) G−1 G = GG−1 = Id .
Then we wish to show that B(G; ) ⊂ GL(H) for some  > 0. In fact we shall see
that we can take  = kG−1 k−1 . To show that G + B is invertible set
(3.115) E = −G−1 B =⇒ G + B = G(Id +G−1 B) = G(Id −E)
From Lemma 33 we know that
(3.116) kBk < 1/kG−1 k =⇒ kG−1 Bk < 1 =⇒ Id −E is invertible.
Then (Id −E)−1 G−1 satisfies
(3.117) (Id −E)−1 G−1 (G + B) = (Id −E)−1 (Id −E) = Id .
Moreover E 0 = −BG−1 also satisfies kE 0 k ≤ kBkkG−1 k < 1 and
(3.118) (G + B)G−1 (Id −E 0 )−1 = (Id −E 0 )(Id −E 0 )−1 = Id .
Thus G + B has both a ‘left’ and a ‘right’ inverse. The associtivity of the operator
product (that A(BC) = (AB)C then shows that
(3.119) G−1 (Id −E 0 )−1 = (Id −E)−1 G−1 (G+B)G−1 (Id −E 0 )−1 = (Id −E)−1 G−1
so the left and right inverses are equal and hence G + B is invertible. 
Thus GL(H) ⊂ B(H), the set of invertible elements, is open. It is also a group
– since the inverse of G1 G2 if G1 , G2 ∈ GL(H) is G−1 −1
2 G1 .
This group of invertible elements has a smaller subgroup, U(H), the unitary
group, defined by
(3.120) U(H) = {U ∈ GL(H); U −1 = U ∗ }.
The unitary group consists of the linear isometric isomorphisms of H onto itself –
thus
(3.121) (U u, U v) = (u, v), kU uk = kuk ∀ u, v ∈ H, U ∈ U(H).
This is an important object and we will use it a little bit later on.
The groups GL(H) and U(H) for a separable Hilbert space may seem very
similar to the familiar groups of invertible and unitary n × n matrices, GL(n) and
U(n), but this is somewhat deceptive. For one thing they are much bigger. In fact
there are other important qualitative differences – you can find some of this in the
problems. One important fact that you should know, even though we will not try
prove it here, is that both GL(H) and U(H) are contractible as a metric spaces –
104 3. HILBERT SPACES

they have no significant topology. This is to be constrasted with the GL(n) and
U(n) which have a lot of topology, and are not at all simple spaces – especially for
large n. One upshot of this is that U(H) does not look much like the limit of the
U(n) as n → ∞. Another important fact that we will show is that GL(H) is not
dense in B(H), in contrast to the finite dimensional case.

17. Spectrum of an operator


Another direct application of Lemma 33, the convergence of the Neumann se-
ries, is that if A ∈ B(H) and λ ∈ C has |λ| > kAk then kλ−1 Ak < 1 so (Id −λ−1 A)−1
exits and satisfies
(3.122) (λ Id −A)λ−1 (Id −λ−1 A)−1 = Id = λ−1 (Id −λ−1 A)−1 (λ − A).
This, λ−A ∈ GL(H) has inverse (λ−A)−1 = λ−1 (Id −λ−1 A)−1 . Thus the resolvent
set
(3.123) {λ ∈ C; (λ Id −A) ∈ GL(H)} ⊂ C
is an open, and non-empty, set called the resolvent set (usually (A − λ)−1 is called
the resolvent). The complement of the resolvent set is called the spectrum of A
(3.124) Spec(A) = {λ ∈ C; λ Id −A ∈
/ GL(H)}.
As follows from the discussion above it is a compact set – it cannot be empty.
For a bounded self-adjoint operator we can say more.
Proposition 40. If A : H −→ H is a bounded operator on a Hilbert space
and A∗ = A then A − λ Id is invertible for all λ ∈ C \ R and either A − kAk Id or
A + kAk Id is not invertible.
The proof of the last part depends on a different characterization of the norm
in the self-adjoint case.
Lemma 34. If A∗ = A then
(3.125) kAk = sup (Au, u).
kuk=1

Proof. Certainly, |hAu, ui| ≤ kAkkuk2 so the right side can only be smaller
than or equal to the left. Suppose that
sup |hAu, ui| = a.
kuk=1

Then for any u, v ∈ H, |hAu, vi| = hAeiθ u, vi so we can arrange that hAu, vi =
|hAu0 , vi| is non-negative and ku0 k = 1 = kuk = kvk. Dropping the primes and
computing using the polarization identity
(3.126)
4hAu, vi = hA(u+v), u+vi−hA(u−v), u−vi+ihA(u+iv), u+ivi−ihA(u−iv), u−ivi.
By the reality of the left side we can drop the last two terms and use the bound to
see that
(3.127) 4hAu, vi =≤ a(ku + vk2 + ku − vk2 ) = 2C(kuk2 + kvk2 ) = 4a
Thus, kAk = supkuk=kvk=1 |hAu, vi| ≤ a and hence kAk = a. 
18. SPECTRAL THEOREM FOR COMPACT SELF-ADJOINT OPERATORS 105

Proof of Proposition 40. If λ = s+it where t 6= 0 then A−λ = (A−s)−it


and A − s is bounded and selfadjoint, so it is enough to consider the special case
that λ = it. Then for any u ∈ H,
(3.128) Imh(A − it)u, ui = −tkuk2 .
So, certainly A − it is injective, since (A − it)u = 0 implies u = 0 if t 6= 0. The
adjoint of A − it is A + it so the adjoint is injective too. It follows that the range
of A − it is dense in H. Indeed, if v ∈ H and v ⊥ (A − it)u for all u ∈ H, so v is
orthogonal to the range, then
(3.129) 0 = Imh(A − it)v, vi = tkvk2 .
It follows that the range is dense. So, if w ∈ H there exists a sequence un in
H with (A − it)un → w. But this implies that kun k is bounded, since tkuk2 =
− Imh(A − it)un , un i and hence we can pass to a weakly convergent subsequence,
un * u. Then (A − it)un * (A − it)u = v so A − it is 1-1 and onto. From the
Open Mapping Theorem, (A − it) is invertible.
Finally then we need to show that one of A ± kAk Id is NOT invertible. This
follows from (3.125). Indeed, by the definition of sup there is a sequence un ∈ H
with kun k = 1 such that either hAun , un i → kAk or hAun , un i → −kAk. In the first
case this means h(A − kuk)un , un i → 0. Then
(3.130) k(A − kAk)2 un k2 = kAun k2 − 2h(A − kAk)un , um i + kAk2 kun k2 .
The second two terms here have limit kAk2 by assumption and the first term is
less than or equal to kAk2 . Since the sequence it positive it follows that k(A −
kAk)2 un k → 0. This means that A − kAk Id is not invertible, since if it had a
bounded inverse B then 1 = kun k ≤ kBkk(A − kAk)2 un k which is impossible.
The other case is similar (or you can replace A by −A) so one of A ± kAk is not
invertible. 

18. Spectral theorem for compact self-adjoint operators


One of the important differences between a general bounded self-adjoint op-
erator and a compact self-adjoint operator is that the latter has eigenvalues and
eigenvectors – lots of them.
Theorem 15. If A ∈ K(H) is a self-adjoint, compact operator on a separable
Hilbert space, so A∗ = A, then H has an orthonormal basis consisting of eigenvec-
tors of A, uj such that
(3.131) Auj = λj uj , λj ∈ R \ {0},
consisting of an orthonormal basis for the possibly infinite-dimensional (closed)
null space and eigenvectors with non-zero eigenvalues which can be arranged into a
sequence such that |λj | is a non-increasing and λj → 0 as j → ∞ (in case Nul(A)⊥
is finite dimensional, this sequence is finite).
The operator A maps Nul(A)⊥ into itself so it may be clearer to first split off the null
space and then look at the operator acting on Nul(A)⊥ which has an orthonormal
basis of eigenvectors with non-vanishing eigenvalues.
Before going to the proof, let’s notice some useful conclusions. One is that we
have ‘Fredholm’s alternative’ in this case.
106 3. HILBERT SPACES

Corollary 6. If A ∈ K(H) is a compact self-adjoint operator on a separable


Hilbert space then the equation
(3.132) u − Au = f
either has a unique solution for each f ∈ H or else there is a non-trivial finite
dimensional space of solutions to
(3.133) u − Au = 0
and then (3.132) has a solution if and only if f is orthogonal to all these solutions.
Proof. This is just saying that the null space of Id −A is a complement to
the range – which is closed. So, either Id −A is invertible or if not then the range
is precisely the orthocomplement of Nul(Id −A). You might say there is not much
alternative from this point of view, since it just says the range is always the ortho-
complement of the null space. 
Let me separate off the heart of the argument from the bookkeeping.
Lemma 35. If A ∈ K(H) is a self-adjoint compact operator on a separable
(possibly finite-dimensional) Hilbert space then
(3.134) F (u) = (Au, u), F : {u ∈ H; kuk = 1} −→ R
is a continuous function on the unit sphere which attains its supremum and infimum
where
(3.135) sup |F (u)| = kAk.
kuk=1

Furthermore, if the maximum or minimum of F (u) is non-zero it is attained at an


eivenvector of A with this extremal value as eigenvalue.
Proof. Since |F (u)| is the function considered in (3.125), (3.135) is a direct
consequence of Lemma 34. Moreover, continuity of F follows from continuity of A
and of the inner product so
(3.136) |F (u)−F (u0 )| ≤ |(Au, u)−(Au, u0 )|+|(Au, u0 )−(Au0 , u0 )| ≤ 2kAkku−u0 k
since both u and u0 have norm one.
If we were in finite dimensions this finishes the proof, since the sphere is then
compact and a continuous function on a compact set attains its sup and inf. In the
general case we need to use the compactness of A. Certainly F is bounded,
(3.137) |F (u)| ≤ sup |(Au, u)| ≤ kAk.
kuk=1

Thus, there is a sequence u+ +
n such that F (un ) → sup F and another un such that

F (un ) → inf F. The weak compactness of the unit sphere means that we can pass
to a weakly convergent subsequence in each case, and so assume that u± n * u
±

converges weakly. Then, by the compactness of A, Au±n → Au ±


converges strongly,
i.e. in norm. But then we can write
(3.138) |F (u± ± ± ± ± ± ± ±
n ) − F (u )| ≤ |(A(un − u ), un )| + |(Au , un − u )|

= |(A(u± ± ± ± ± ± ± ±
n − u ), un )| + |(u , A(un − u ))| ≤ 2kAun − Au k

to deduce that F (u± ) = lim F (u±n ) are respectively the sup and inf of F. Thus
indeed, as in the finite dimensional case, the sup and inf are attained, as in max
18. SPECTRAL THEOREM FOR COMPACT SELF-ADJOINT OPERATORS 107

and min. Note that this is NOT typically true if A is not compact as well as
self-adjoint.
Now, suppose that Λ+ = sup F > 0. Then for any v ∈ H with v ⊥ u+ the curve
(3.139) Lv : (−π, π) 3 θ 7−→ cos θu+ + sin θv
lies in the unit sphere. Computing out
(3.140) F (Lv (θ)) =
(ALv (θ), Lv (θ)) = cos2 θF (u+ ) + 2 sin(2θ) Re(Au+ , v) + sin2 (θ)F (v)
we know that this function must take its maximum at θ = 0. The derivative there
(it is certainly continuously differentiable on (−π, π)) is Re(Au+ , v) which must
therefore vanish. The same is true for iv in place of v so in fact
(3.141) (Au+ , v) = 0 ∀ v ⊥ u+ , kvk = 1.
Taking the span of these v’s it follows that (Au+ , v) = 0 for all v ⊥ u+ so A+ u
must be a multiple of u+ itself. Inserting this into the definition of F it follows
that Au+ = Λ+ u+ is an eigenvector with eigenvalue Λ+ = sup F.
The same argument applies to inf F if it is negative, for instance by replacing
A by −A. This completes the proof of the Lemma. 
Proof of Theorem 15. First consider the Hilbert space H0 = Nul(A)⊥ ⊂
H. Then, as noted above, A maps H0 into itself, since
(3.142) (Au, v) = (u, Av) = 0 ∀ u ∈ H0 , v ∈ Nul(A) =⇒ Au ∈ H0 .
Moreover, A0 , which is A restricted to H0 , is again a compact self-adjoint operator
– where the compactness follows from the fact that A(B(0, 1)) for B(0, 1) ⊂ H0 is
smaller than (actually of course equal to) the whole image of the unit ball.
Thus we can apply the Lemma above to A0 , with quadratic form F0 , and find
an eigenvector. Let’s agree to take the one associated to sup FA0 unless supA0 <
− inf F0 in which case we take one associated to the inf . Now, what can go wrong
here? Nothing except if F0 ≡ 0. However in that case we know from Lemma 34
that kAk = 0 so A = 0.
So, we now know that we can find an eigenvector with non-zero eigenvalue
unless A ≡ 0 which would implies Nul(A) = H. Now we proceed by induction.
Suppose we have found N mutually orthogonal eigenvectors ej for A all with norm
1 and eigenvectors λj – an orthonormal set of eigenvectors and all in H0 . Then we
consider
(3.143) HN = {u ∈ H0 = Nul(A)⊥ ; (u, ej ) = 0, j = 1, . . . , N }.
From the argument above, A maps HN into itself, since
(3.144) (Au, ej ) = (u, Aej ) = λj (u, ej ) = 0 if u ∈ HN =⇒ Au ∈ HN .
Moreover this restricted operator is self-adjoint and compact on HN as before so
we can again find an eigenvector, with eigenvalue either the max of min of the new
F for HN . This process will not stop uness F ≡ 0 at some stage, but then A ≡ 0
on HN and since HN ⊥ Nul(A) which implies HN = {0} so H0 must have been
finite dimensional.
Thus, either H0 is finite dimensional or we can grind out an infinite orthonormal
sequence ei of eigenvectors of A in H0 with the corresponding sequence of eigen-
values such that |λi | is non-increasing – since the successive FN ’s are restrictions
108 3. HILBERT SPACES

of the previous ones the max and min are getting closer to (or at least no further
from) 0.
So we need to rule out the possibility that there is an infinite orthonormal
sequence of eigenfunctions ej with corresponding eigenvalues λj where inf j |λj | =
a > 0. Such a sequence cannot exist since ej * 0 so by the compactness of A,
Aej → 0 (in norm) but |Aej | ≥ a which is a contradiction. Thus if null(A)⊥ is
not finite dimensional then the sequence of eigenvalues constructed above must
converge to 0.
Finally then, we need to check that this orthonormal sequence of eigenvectors
constitutes an orthonormal basis of H0 . If not, then we can form the closure of the
span of the ei we have constructed, H0 , and its orthocomplement in H0 – which
would have to be non-trivial. However, as before F restricts to this space to be
F 0 for the restriction of A0 to it, which is again a compact self-adjoint operator.
So, if F 0 is not identically zero we can again construct an eigenfunction, with non-
zero eigenvalue, which contracdicts the fact the we are always choosing a largest
eigenvalue, in absolute value at least. Thus in fact F 0 ≡ 0 so A0 ≡ 0 and the
eigenvectors form and orthonormal basis of Nul(A)⊥ . This completes the proof of
the theorem. 

19. Functional Calculus


So the non-zero eigenvalues of a compact self-adjoint operator form the image of
a sequence in [−kAk, kAk] either converging to zero or finite. If f ∈ C 0 ([−kAk, kAk)
then one can define an operator
X
(3.145) f (A) ∈ B(H), f (A)u = f (λu )(u, ei )ei
i

where {ei } is a complete orthonormal basis of eigenfunctions. Provided f (0) = 0


this is compact and if f is real it is self-adjoint. This formula actually defines a
linear map

(3.146) C 0 ([−kAk, kAk]) −→ B(H) with f (A)g(A) = (f g)(A).

Such a map exists for any bounded self-adjoint operator. Even though it may
not have eigenfunctions – or not a complete orthonormal basis of them anyway, it
is still possible to define f (A) for a continous function defined on [−kAk, kAk] (in
fact it only has to be defined on Spec(A) ⊂ [−kAk, kAk] which might be quite a lot
smaller). This is an effective replacement for the spectral theorem in the compact
case.
How does one define f (A)? Well, it is easy enough in case f is a polynomial,
since then we can factorize it and set
(3.147)
f (z) = c(z − z1 )(z − z2 ) . . . (z − zN ) =⇒ f (A) = c(A − z1 )(A − z2 ) . . . (A − zN ).

Notice that the result does not depend on the order of the factors or anything like
that. To pass to the case of a general continuous function on [−kAk, kAk] one can
use the norm estimate in the polynomial case, that

(3.148) kf (A)k ≤ sup |f (z)|.


z∈[−kAk,kAk
20. COMPACT PERTURBATIONS OF THE IDENTITY 109

This allows one to pass f in the uniform closure of the polynomials, which by the
Stone-Weierstrass theorem is the whole of C 0 ([−kAk, kAk]). The proof of (3.148) is
outlined in Problem 3.3 below.

20. Compact perturbations of the identity


I have generally not had a chance to discuss most of the material in this section,
or the next, in the lectures.
Next we head towards the spectral theory of self-adjoint compact operators.
This is rather similar to the spectral theory of self-adjoint matrices and has many
useful applications as we shall see. There is a very effective spectral theory of
general bounded but self-adjoint operators but I do not expect to have time to do
this. There is also a pretty satisfactory spectral theory of non-selfadjoint compact
operators. There is no satisfactory spectral theory for general non-compact and
non-self-adjoint operators as you can easily see from examples (such as the shift
operator).
Compact operators are, as we know, ‘small’ in the sense that the are norm
limits of finite rank operators. If you accept this, then you will want to say that an
operator such as
(3.149) Id −K, K ∈ K(H)
is ‘big’. We are quite interested in this operator because of spectral theory. To say
that λ ∈ C is an eigenvalue of K is to say that there is a non-trivial solution of
(3.150) Ku − λu = 0
where non-trivial means other than than the solution u = 0 which always exists. If
λ is an eigenvalue of K then certainly λ ∈ Spec(K), since λ−K cannot be invertible.
For general operators the converse is not correct, but for compact operators it is.
Lemma 36. If K ∈ B(H) is a compact operator then λ ∈ C\{0} is an eigenvalue
of K if and only if λ ∈ Spec(K).
Proof. Since we can divide by λ we may replace K by λ−1 K and consider the
special case λ = 1. Now, if K is actually finite rank the result is straightforward.
By Lemma 27 we can choose a basis so that (3.75) holds. Let the span of the ei
be W – since it is finite dimensional it is closed. Then Id −K acts rather simply –
decomposing H = W ⊕ W ⊥ , u = w + w0
(3.151) (Id −K)(w + w0 ) = w + (IdW −K 0 )w0 , K 0 : W −→ W
being a matrix with respect to the basis. Now, 1 is an eigenvalue of K if and only
if 1 is an eigenvalue of K 0 as an operator on the finite-dimensional space W. Now,
a matrix, such as IdW −K 0 , is invertible if and only if it is inective, or equivalently
surjective. So, the same is true for Id −K.
Now in the general case we use the approximability of K by finite rank opera-
tors. Thus, we can choose a finite rank operator F such that kK − F k < 1/2. Thus,
(Id −K + F )−1 = Id −B is invertible. Then we can write
(3.152) Id −K = Id −(K − F ) − F = (Id −(K − F ))(Id −L), L = (Id −B)F.
Thus, Id −K is invertible if and only if Id −L is invertible. Thus, if Id −K is not
invertible then Id −L is not invertible and hence has null space and from (3.152) it
follows that Id −K has non-trivial null space, i.e. K has 1 as an eigenvalue. 
110 3. HILBERT SPACES

A little more generally:-


Proposition 41. If K ∈ K(H) is a compact operator on a separable Hilbert
space then
null(Id −K) = {u ∈ H; (IdK )u = 0} is finite dimensional
(3.153) Ran(Id −K) = {v ∈ H; ∃u ∈ H, v = (Id −K)u} is closed and
Ran(Id −K)⊥ = {w ∈ H; (w, Ku) = 0 ∀ u ∈ H} is finite dimensional
and moreover
dim (null(Id −K)) = dim Ran(Id −K)⊥ .

(3.154)
Proof of Proposition 41. First let’s check this in the case of a finite rank
operator K = T. Then
(3.155) Nul(Id −T ) = {u ∈ H; u = T u} ⊂ Ran(T ).
A subspace of a finite dimensional space is certainly finite dimensional, so this
proves the first condition in the finite rank case.
Similarly, still assuming that T is finite rank consider the range
(3.156) Ran(Id −T ) = {v ∈ H; v = (Id −T )u for some u ∈ H}.
Consider the subspace {u ∈ H; T u = 0}. We know that this this is closed, since T
is certainly continuous. On the other hand from (3.156),
(3.157) Ran(Id −T ) ⊃ Nul(T ).
Remember that a finite rank operator can be written out as a finite sum
N
X
(3.158) Tu = (u, ei )fi
i=1
where we can take the fi to be a basis of the range of T. We also know in this
case that the ei must be linearly independent – if they weren’t then we could
P write
one of them, say the last since we can renumber, out as a sum, eN = ci ej , of
j<N
multiples of the others and then find
N
X −1
(3.159) Tu = (u, ei )(fi + cj fN )
i=1
showing that the range of T has dimension at most N − 1, contradicting the fact
that the fi span it.
So, going back to (3.158) we know that Nul(T ) has finite codimension – every
element of H is of the form
N
X
(3.160) u = u0 + di ei , u0 ∈ Nul(T ).
i=1
So, going back to (3.157), if Ran(Id −T ) 6= Nul(T ), and it need not be equal, we
can choose – using the fact that Nul(T ) is closed – an element g ∈ Ran(Id −T ) \
Nul(T ) which is orthogonal to Nul(T ). To do this, start with any a vector g 0 in
Ran(Id −T ) which is not in Nul(T ). It can be split as g 0 = u00 + g where g ⊥
Nul(T ) (being a closed subspace) and u00 ∈ Nul(T ), then g 6= 0 is in Ran(Id −T )
and orthongonal to Nul(T ). Now, the new space Nul(T ) ⊕ Cg is again closed and
contained in Ran(Id −T ). But we can continue this process replacing Nul(T ) by
20. COMPACT PERTURBATIONS OF THE IDENTITY 111

this larger closed subspace. After a a finite number of steps we conclude that
Ran(Id −T ) itself is closed.
What we have just proved is:
Lemma 37. If V ⊂ H is a subspace of a Hilbert space which contains a closed
subspace of finite codimension in H – meaning V ⊃ W where W is closed and there
are finitely many elements ei ∈ H, i = 1, . . . , N such that every element u ∈ H is
of the form
N
X
(3.161) u = u0 + ci ei , ci ∈ C,
i=1

then V itself is closed.


So, this takes care of the case that K = T has finite rank! What about the
general case where K is compact? Here we just use a consequence of the approxi-
mation of compact operators by finite rank operators proved last time. Namely, if
K is compact then there exists B ∈ B(H) and T of finite rank such that
1
(3.162) K = B + T, kBk < .
2
Now, consider the null space of Id −K and use (3.162) to write
(3.163) Id −K = (Id −B) − T = (Id −B)(Id −T 0 ), T 0 = (Id −B)−1 T.
Here we have used the convergence of the Neumann series, so (Id −B)−1 does exist.
Now, T 0 is of finite rank, by the ideal property, so
(3.164) Nul(Id −K) = Nul(Id −T 0 ) is finite dimensional.
Here of course we use the fact that (Id −K)u = 0 is equivalent to (Id −T 0 )u = 0
since Id −B is invertible. So, this is the first condition in (3.153).
Similarly, to examine the second we do the same thing but the other way around
and write
(3.165) Id −K = (Id −B) − T = (Id −T 00 )(Id −B), T 00 = T (Id −B)−1 .
Now, T 00 is again of finite rank and
(3.166) Ran(Id −K) = Ran(Id −T 00 ) is closed
again using the fact that Id −B is invertible – so every element of the form (Id −K)u
is of the form (Id −T 00 )u0 where u0 = (Id −B)u and conversely.
So, now we have proved all of (3.153) – the third part following from the first
as discussed before.
What about (3.154)? This time let’s first check that it is enough to consider
the finite rank case. For a compact operator we have written
(3.167) (Id −K) = G(Id −T )
1
where G = Id −B with kBk < 2 is invertible and T is of finite rank. So what we
want to see is that
(3.168) dim Nul(Id −K) = dim Nul(Id −T ) = dim Nul(Id −K ∗ ).
However, Id −K ∗ = (Id −T ∗ )G∗ and G∗ is also invertible, so
(3.169) dim Nul(Id −K ∗ ) = dim Nul(Id −T ∗ )
112 3. HILBERT SPACES

and hence it is enough to check that dim Nul(Id −T ) = dim Nul(Id −T ∗ ) – which is
to say the same thing for finite rank operators.
Now, for a finite rank operator, written out as (3.158), we can look at the
vector space W spanned by all the fi ’s and all the ei ’s together – note that there is
nothing to stop there being dependence relations among the combination although
separately they are independent. Now, T : W −→ W as is immediately clear and
N
X
(3.170) T ∗v = (v, fi )ei
i=1

so T : W −→ W too. In fact T w = 0 and T ∗ w0 = 0 if w0 ∈ W ⊥ since then


0

(w0 , ei ) = 0 and (w0 , fi ) = 0 for all i. It follows that if we write R : W ←→ W for


the linear map on this finite dimensional space which is equal to Id −T acting on
it, then R∗ is given by Id −T ∗ acting on W and we use the Hilbert space structure
on W induced as a subspace of H. So, what we have just shown is that
(3.171)
(Id −T )u = 0 ⇐⇒ u ∈ W and Ru = 0, (Id −T ∗ )u = 0 ⇐⇒ u ∈ W and R∗ u = 0.
Thus we really are reduced to the finite-dimensional theorem
(3.172) dim Nul(R) = dim Nul(R∗ ) on W.
You no doubt know this result. It follows by observing that in this case, every-
thing now on W, Ran(W ) = Nul(R∗ )⊥ and finite dimensions
(3.173) dim Nul(R) + dim Ran(R) = dim W = dim Ran(W ) + dim Nul(R∗ ).


21. Fredholm operators


Definition 20. A bounded operator F ∈ B(H) on a Hilbert space is said to be
Fredholm if it has the three properties in (3.153) – its null space is finite dimensional,
its range is closed and the orthocomplement of its range is finite dimensional.
For general Fredholm operators the row-rank=colum-rank result (3.154) does not
hold. Indeed the difference of these two integers
ind(F ) = dim (null(Id −K)) − dim Ran(Id −K)⊥

(3.174)
is a very important number with lots of interesting properties and uses.
Notice that the last two conditions in (3.153) are really independent since the
orthocomplement of a subspace is the same as the orthocomplement of its closure.
There is for instance a bounded operator on a separable Hilbert space with trivial
null space and dense range which is not closed. How could this be? Think for
instance of the operator on L2 (0, 1) which is multiplication by the function x.
This is assuredly bounded and an element of the null space would have to satisfy
xu(x) = 0 almost everywhere, and hence vanish almost everywhere. Moreover the
density of the L2 functions vanishing in x <  for some (non-fixed)  > 0 shows
that the range is dense. However it is clearly not invertible.
Before proving this result let’s check that the third condition in (3.153) really
follows from the first. This is a general fact which I mentioned, at least, earlier but
let me pause to prove it.
22. PROBLEMS 113

Proposition 42. If B ∈ B(H) is a bounded operator on a Hilbert space and


B ∗ is its adjoint then
(3.175) Ran(B)⊥ = (Ran(B))⊥ = {v ∈ H; (v, w) = 0 ∀ w ∈ Ran(B)} = Nul(B ∗ ).
Proof. The definition of the orthocomplement of Ran(B) shows immediately
that

(3.176) v ∈ (Ran(B))⊥ ⇐⇒ (v, w) = 0 ∀ w ∈ Ran(B) ←→ (v, Bu) = 0 ∀ u ∈ H


⇐⇒ (B ∗ v, u) = 0 ∀ u ∈ H ⇐⇒ B ∗ v = 0 ⇐⇒ v ∈ Nul(B ∗ ).
On the other hand we have already observed that V ⊥ = (V )⊥ for any subspace –
since the right side is certainly contained in the left and (u, v) = 0 for all v ∈ V
implies that (u, w) = 0 for all w ∈ V by using the continuity of the inner product
to pass to the limit of a sequence vn → w. 

Thus as a corrollary we see that if Nul(Id −K) is always finite dimensional for
K compact (i. e. we check it for all compact operators) then Nul(Id −K ∗ ) is finite
dimensional and hence so is Ran(Id −K)⊥ .

22. Problems
Problem 3.1. Let H be a normed space in which the norm satisfies the par-
allelogram law:
(3.177) ku + vk2 + ku − vk2 = 2(kuk2 + kvk2 ) ∀ u, v ∈ H.
Show that the norm comes from a positive definite sesquilinear (i.e. ermitian) inner
product. Big Hint:- Try
1
ku + vk2 − ku − vk2 + iku + ivk2 − iku − ivk2 !

(3.178) (u, v) =
4
Problem 3.2. Let H be a finite dimensional (pre)Hilbert space. So, by defi-
nition H has a basis {vi }ni=1 , meaning that any element of H can be written
X
(3.179) v= c i vi
i

and there is no dependence relation between the vi ’s – the presentation of v = 0 in


the form (3.179) is unique. Show that H has an orthonormal basis, {ei }ni=1 satis-
fying (ei , ej ) = δij (= 1 if i = j and 0 otherwise). Check that for the orthonormal
basis the coefficients in (3.179) are ci = (v, ei ) and that the map
(3.180) T : H 3 v 7−→ ((v, ei )) ∈ Cn
is a linear isomorphism with the properties
X
(3.181) (u, v) = (T u)i (T v)i , kukH = kT ukCn ∀ u, v ∈ H.
i

Why is a finite dimensional preHilbert space a Hilbert space?


Problem 3.3. : Prove (3.148). The important step is actually the fact that
Spec(A) ⊂ [−kAk, kAk] if A is self-adjoint, which is proved somewhere above. Now,
if f is a real polynomial, we can assume the leading constant, c, in (3.147) is 1.
If λ ∈/ f ([−kAk, kAk]) then f (A) is self-adjoint and λ − f (A) is invertible – it is
114 3. HILBERT SPACES

enough to check this for each factor in (3.147). Thus Spec(f (A)) ⊂ f ([−kAk, kAk])
which means that
(3.182) kf (A)k ≤ sup{z ∈ f ([−kAk, kAk])}
which is in fact (3.147).
Problem 3.4. Let H be a separable Hilbert space. Show that K ⊂ H is
compact if and only if it is closed, bounded and has the property that any sequence
in K which is weakly convergent sequence in H is (strongly) convergent.
Hint (Problem ??) In one direction use the result from class that any bounded
sequence has a weakly convergent subsequence.
Problem 3.5. Show that, in a separable Hilbert space, a weakly convergent
sequence {vn }, is (strongly) convergent if and only if the weak limit, v satisfies
(3.183) kvkH = lim kvn kH .
n→∞

Hint (Problem 3.5) To show that this condition is sufficient, expand


(3.184) (vn − v, vn − v) = kvn k2 − 2 Re(vn , v) + kvk2 .
Problem 3.6. Show that a subset of a separable Hilbert space is compact if
and only if it is closed and bounded and has the property of ‘finite dimensional
approximation’ meaning that for any  > 0 there exists a linear subspace DN ⊂ H
of finite dimension such that
(3.185) d(K, DN ) = sup inf {d(u, v)} ≤ .
u∈K v∈DN

See Hint 22
Hint (Problem ??) To prove necessity of this condition use the ‘equi-small
tails’ property of compact sets with respect to an orthonormal basis. To use the
finite dimensional approximation condition to show that any weakly convergent
sequence in K is strongly convergent, use the convexity result from class to define
the sequence {vn0 } in DN where vn0 is the closest point in DN to vn . Show that vn0
is weakly, hence strongly, convergent and hence deduce that {vn } is Cauchy.
Problem 3.7. Suppose that A : H −→ H is a bounded linear operator with
the property that A(H) ⊂ H is finite dimensional. Show that if vn is weakly
convergent in H then Avn is strongly convergent in H.
Problem 3.8. Suppose that H1 and H2 are two different Hilbert spaces and
A : H1 −→ H2 is a bounded linear operator. Show that there is a unique bounded
linear operator (the adjoint) A∗ : H2 −→ H1 with the property
(3.186) (Au1 , u2 )H2 = (u1 , A∗ u2 )H1 ∀ u1 ∈ H1 , u2 ∈ H2 .
Problem 3.9. Question:- Is it possible to show the completeness of the Fourier
basis

exp(ikx)/ 2π
by computation? Maybe, see what you think. These questions are also intended to
get you to say things clearly.
22. PROBLEMS 115

(1) Work out the Fourier coefficients ck (t) = (0,2π) ft e−ikx of the step func-
R

tion
(
1 0≤x<t
(3.187) ft (x) =
0 t ≤ x ≤ 2π
for each fixed t ∈ (0, 2π).
(2) Explain why this Fourier series converges to ft in L2 (0, 2π) if and only if
X
(3.188) 2 |ck (t)|2 = 2πt − t2 , t ∈ (0, 2π).
k>0

(3) Write this condition out as a Fourier series and apply the argument again
to show that the completeness of the Fourier basis implies identities for
the sum of k −2 and k −4 .
(4) Can you explain how reversing the argument, that knowledge of the sums
of these two series should imply the completeness of the Fourier basis?
There is a serious subtlety in this argument, and you get full marks for
spotting it, without going ahead a using it to prove completeness.
Problem 3.10. Prove that for appropriate constants dk , the functions dk sin(kx/2),
k ∈ N, form an orthonormal basis for L2 (0, 2π).
See Hint 22
Hint (Problem 3.17 The usual method is to use the basic result from class plus
translation and rescaling to show that d0k exp(ikx/2) k ∈ Z form an orthonormal
basis of L2 (−2π, 2π). Then extend functions as odd from (0, 2π) to (−2π, 2π).
Problem 3.11. Let ek , k ∈ N, be an orthonormal basis in a separable Hilbert
space, H. Show that there is a uniquely defined bounded linear operator S : H −→
H, satisfying
(3.189) Sej = ej+1 ∀ j ∈ N.
Show that if B : H −→ H is a bounded linear operator then S +B is not invertible
if  < 0 for some 0 > 0.
Hint (Problem 3.18)- Consider the linear functional L : H −→ C, Lu =
(Bu, e1 ). Show that B 0 u = Bu − (Lu)e1 is a bounded linear operator from H
to the Hilbert space H1 = {u ∈ H; (u, e1 ) = 0}. Conclude that S + B 0 is invertible
as a linear map from H to H1 for small . Use this to argue that S + B cannot be
an isomorphism from H to H by showing that either e1 is not in the range or else
there is a non-trivial element in the null space.
Problem 3.12. Show that the product of bounded operators on a Hilbert space
is strong continuous, in the sense that if An and Bn are strong convergent sequences
of bounded operators on H with limits A and B then the product An Bn is strongly
convergent with limit AB.
Hint (Problem 3.19) Be careful! Use the result in class which was deduced from
the Uniform Boundedness Theorem.
Problem 3.13. Show that a continuous function K : [0, 1] −→ L2 (0, 2π) has
the property that the Fourier series of K(x) ∈ L2 (0, 2π), for x ∈ [0, 1], converges
116 3. HILBERT SPACES

uniformly in the sense that if Kn (x) is the sum of the Fourier series over |k| ≤ n
then Kn : [0, 1] −→ L2 (0, 2π) is also continuous and
(3.190) sup kK(x) − Kn (x)kL2 (0,2π) → 0.
x∈[0,1]

Hint (Problem 3.13) Use one of the properties of compactness in a Hilbert space
that you proved earlier.
Problem 3.14. Consider an integral operator acting on L2 (0, 1) with a kernel
which is continuous – K ∈ C([0, 1]2 ). Thus, the operator is
Z
(3.191) T u(x) = K(x, y)u(y).
(0,1)
2
Show that T is bounded on L (I think we did this before) and that it is in the
norm closure of the finite rank operators.
Hint (Problem 3.13) Use the previous problem! Show that a continuous function
such as K in this Problem defines a continuous map [0, 1] 3 x 7−→ K(x, ·) ∈ C([0, 1])
and hence a continuous function K : [0, 1] −→ L2 (0, 1) then apply the previous
problem with the interval rescaled.
Here is an even more expanded version of the hint: You can think of K(x, y) as
a continuous function of x with values in L2 (0, 1). Let Kn (x, y) be the continuous
function of x and y given by the previous problem, by truncating the Fourier series
(in y) at some point n. Check that this defines a finite rank operator on L2 (0, 1)
– yes it maps into continuous functions but that is fine, they are Lebesgue square
integrable. Now, the idea is the difference K − Kn defines a bounded operator with
small norm as n becomes large. It might actually be clearer to do this the other
way round, exchanging the roles of x and y.
Problem 3.15. Although we have concentrated on the Lebesgue integral in
one variable, you proved at some point the covering lemma in dimension 2 and
that is pretty much all that was needed to extend the discussion to 2 dimensions.
Let’s just assume you have assiduously checked everything and so you know that
L2 ((0, 2π)2 ) is a Hilbert space. Sketch a proof – noting anything that you are not
sure of – that the functions exp(ikx+ily)/2π, k, l ∈ Z, form a complete orthonormal
basis.
Problem 3.16. Question:- Is it possible to show the completeness of the Fourier
basis √
exp(ikx)/ 2π
by computation? Maybe, see what you think. These questions are also intended to
get you to say things clearly.
(1) Work out the Fourier coefficients ck (t) = (0,2π) ft e−ikx of the step func-
R

tion
(
1 0≤x<t
(3.192) ft (x) =
0 t ≤ x ≤ 2π
for each fixed t ∈ (0, 2π).
(2) Explain why this Fourier series converges to ft in L2 (0, 2π) if and only if
X
(3.193) 2 |ck (t)|2 = 2πt − t2 , t ∈ (0, 2π).
k>0
22. PROBLEMS 117

(3) Write this condition out as a Fourier series and apply the argument again
to show that the completeness of the Fourier basis implies identities for
the sum of k −2 and k −4 .
(4) Can you explain how reversing the argument, that knowledge of the sums
of these two series should imply the completeness of the Fourier basis?
There is a serious subtlety in this argument, and you get full marks for
spotting it, without going ahead a using it to prove completeness.
Problem 3.17. Prove that for appropriate constants dk , the functions dk sin(kx/2),
k ∈ N, form an orthonormal basis for L2 (0, 2π).
Hint: The usual method is to use the basic result from class plus translation
and rescaling to show that d0k exp(ikx/2) k ∈ Z form an orthonormal basis of
L2 (−2π, 2π). Then extend functions as odd from (0, 2π) to (−2π, 2π).
Problem 3.18. Let ek , k ∈ N, be an orthonormal basis in a separable Hilbert
space, H. Show that there is a uniquely defined bounded linear operator S : H −→
H, satisfying
(3.194) Sej = ej+1 ∀ j ∈ N.
Show that if B : H −→ H is a bounded linear operator then S +B is not invertible
if  < 0 for some 0 > 0.
Hint:- Consider the linear functional L : H −→ C, Lu = (Bu, e1 ). Show that
B 0 u = Bu − (Lu)e1 is a bounded linear operator from H to the Hilbert space
H1 = {u ∈ H; (u, e1 ) = 0}. Conclude that S + B 0 is invertible as a linear map from
H to H1 for small . Use this to argue that S + B cannot be an isomorphism from
H to H by showing that either e1 is not in the range or else there is a non-trivial
element in the null space.
Problem 3.19. Show that the product of bounded operators on a Hilbert space
is strong continuous, in the sense that if An and Bn are strong convergent sequences
of bounded operators on H with limits A and B then the product An Bn is strongly
convergent with limit AB.
Hint: Be careful! Use the result in class which was deduced from the Uniform
Boundedness Theorem.
Problem 3.20. Let H be a separable (partly because that is mostly what I
have been talking about) Hilbert space with inner product (·, ·) and norm k · k. Say
that a sequence un in H converges weakly if (un , v) is Cauchy in C for each v ∈ H.
(1) Explain why the sequence kun kH is bounded.
Solution: Each un defines a continuous linear functional on H by
(3.195) Tn (v) = (v, un ), kTn k = kun k, Tn : H −→ C.
For fixed v the sequence Tn (v) is Cauchy, and hence bounded, in C so by
the ‘Uniform Boundedness Principle’ the kTn k are bounded, hence kun k
is bounded in R.
(2) Show that there exists an element u ∈ H such that (un , v) → (u, v) for
each v ∈ H.
Solution: Since (v, un ) is Cauchy in C for each fixed v ∈ H it is
convergent. Set
(3.196) T v = lim (v, un ) in C.
n→∞
118 3. HILBERT SPACES

This is a linear map, since


(3.197) T (c1 v1 + c2 v2 ) = lim c1 (v1 , un ) + c2 (v2 , u) = c1 T v1 + c2 T v2
n→∞

and is bounded since |T v| ≤ Ckvk, C = supn kun k. Thus, by Riesz’ the-


orem there exists u ∈ H such that T v = (v, u). Then, by definition of
T,
(3.198) (un , v) → (u, v) ∀ v ∈ H.
(3) If ei , i ∈ N, is an orthonormal sequence, give, with justification, an ex-
ample of a sequence un which is not weakly convergent in H but is such
that (un , ej ) converges for each j.
Solution: One such example is un = nen . Certainly (un , ei ) = 0 for all
i > n, so converges to 0. However, kun k is not bounded, so the sequence
cannot be weakly convergent by the first part above.
(4) Show that if the ei form an orthonormal basis, kun k is bounded and
(un , ej ) converges for each j then un converges weakly.
Solution: By the assumption that (un , ej ) converges for all j it follows
that (un , v) converges as n → ∞ for all v which is a finite linear combi-
nation of the ei . For general v ∈ H the convergence of the Fourier-Bessell
series for v with respect to the orthonormal basis ej
X
(3.199) v= (v, ek )ek
k
shows that there is a sequence vk → v where each vk is in the finite span
of the ej . Now, by Cauchy’s inequality
(3.200) |(un , v) − (um , v)| ≤ |(un vk ) − (um , vk )| + |(un , v − vk )| + |(um , v − vk )|.
Given  > 0 the boundedness of kun k means that the last two terms can be
arranged to be each less than /4 by choosing k sufficiently large. Having
chosen k the first term is less than /4 if n, m > N by the fact that (un , vk )
converges as n → ∞. Thus the sequence (un , v) is Cauchy in C and hence
convergent.
Problem 3.21. Consider the two spaces of sequences

X
h±2 = {c : N 7−→ C; j ±4 |cj |2 < ∞}.
j=1

Show that both h±2 are Hilbert spaces and that any linear functional satisfying
T : h2 −→ C, |T c| ≤ Ckckh2
for some constant C is of the form

X
Tc = ci di
j=1

where d : N −→ C is an element of h−2 .


Solution: Many of you hammered this out by parallel with l2 . This is fine, but
to prove that h±2 are Hilbert spaces we can actually use l2 itself. Thus, consider
the maps on complex sequences
(3.201) (T ± c)j = cj j ±2 .
22. PROBLEMS 119

Without knowing anything about h±2 this is a bijection between the sequences in
h±2 and those in l2 which takes the norm
(3.202) kckh±2 = kT ckl2 .
It is also a linear map, so it follows that h± are linear, and that they are indeed
Hilbert spaces with T ± isometric isomorphisms onto l2 ; The inner products on h±2
are then

X
(3.203) (c, d)h±2 = j ±4 cj dj .
j=1

Don’t feel bad if you wrote it all out, it is good for you!
Now, once we know that h2 is a Hilbert space we can apply Riesz’ theorem to
see that any continuous linear functional T : h2 −→ C, |T c| ≤ Ckckh2 is of the form

X
0
(3.204) T c = (c, d )h2 = j 4 cj d0j , d0 ∈ h2 .
j=1

Now, if d0 ∈ h2 then dj = j 4 d0j defines a sequence in h−2 . Namely,


X X
(3.205) j −4 |dj |2 = j 4 |d0j |2 < ∞.
j j

Inserting this in (3.204) we find that



X
(3.206) Tc = cj dj , d ∈ h−2 .
j=1

(1) In P9.2 (2), and elsewhere, C ∞ (S) should be C 0 (S), the space of continuous
functions on the circle – with supremum norm.
(2) In (3.219) it should be u = F v, not u = Sv.
(3) Similarly, before (3.220) it should be u = F v.
(4) Discussion around (3.222) clarified.
(5) Last part of P10.2 clarified.
This week I want you to go through the invertibility theory for the operator
d2
(3.207) Qu = (− + V (x))u(x)
dx2
acting on periodic functions. Since we have not developed the theory to handle this
directly we need to approach it through integral operators.
Problem 3.22. Let S be the circle of radius 1 in the complex plane, centered
at the origin, S = {z; |z| = 1}.
(1) Show that there is a 1-1 correspondence

(3.208) C 0 (S) = {u : S −→ C, continuous} −→


{u : R −→ C; continuous and satisfying u(x + 2π) = u(x) ∀ x ∈ R}.
(2) Show that there is a 1-1 correspondence

(3.209) L2 (0, 2π) ←→ {u ∈ L1loc (R); u (0,2π) ∈ L2 (0, 2π)


and u(x + 2π) = u(x) ∀ x ∈ R}/NP


120 3. HILBERT SPACES

where NP is the space of null functions on R satisfying u(x + 2π) = u(x)


for all x ∈ R.
(3) If we denote by L2 (S) the space on the left in (3.209) show that there is
a dense inclusion
(3.210) C 0 (S) −→ L2 (S).
So, the idea is that we can think of functions on S as 2π-periodic functions on
R.
Next are some problems dealing with Schrödinger’s equation, or at least it is
an example thereof:
d2 u(x)
(3.211) − + V (x)u(x) = f (x), x ∈ R,
dx2
(1) First we will consider the special case V = 1. Why not V = 0? – Don’t
try to answer this until the end!
(2) Recall how to solve the differential equation
d2 u(x)
(3.212) − + u(x) = f (x), x ∈ R,
dx2
where f (x) ∈ C 0 (S) is a continuous, 2π-periodic function on the line. Show
that there is a unique 2π-periodic and twice continuously differentiable
function, u, on R satisfying (3.212) and that this solution can be written
in the form
Z
(3.213) u(x) = (Sf )(x) = A(x, y)f (y)
0,2π

where A(x, y) ∈ C 0 (R2 ) satisfies A(x+2π, y +2π) = A(x, y) for all (x, y) ∈
R.
Extended hint: In case you managed to avoid a course on differential
equations! First try to find a solution, igonoring the periodicity issue. To
do so one can (for example, there are other ways) factorize the differential
operator involved, checking that
d2 u(x) dv du
(3.214) − 2
+ u(x) = −( + v) if v = −u
dx dx dx
since the cross terms cancel. Then recall the idea of integrating factors to
see that
du dφ
− u = ex , φ = e−x u,
(3.215) dx dx
dv dψ
+ v = e−x , ψ = ex v.
dx dx
Now, solve the problem by integrating twice from the origin (say) and
hence get a solution to the differential equation (3.212). Write this out
explicitly as a double integral, and then change the order of integration
to write the solution as
Z
0
(3.216) u (x) = A0 (x, y)f (y)dy
0,2π

where A is continuous on R×[0, 2π]. Compute the difference u0 (2π)−u0 (0)


0
0
du0 0
and du
dx (2π) − dx (0) as integrals involving f. Now, add to u as solution
22. PROBLEMS 121

to the homogeneous equation, for f = 0, namely c1 ex + c2 e−x , so that the


new solution to (3.212) satisfies u(2π) = u(0) and du du
dx (2π) = dx (0). Now,
check that u is given by an integral of the form (3.213) with A as stated.
(3) Check, either directly or indirectly, that A(y, x) = A(x, y) and that A is
real.
(4) Conclude that the operator S extends by continuity to a bounded operator
on L2 (S).
(5) Check, probably indirectly rather than directly, that
(3.217) S(eikx ) = (k 2 + 1)−1 eikx , k ∈ Z.
(6) Conclude, either from the previous result or otherwise that S is a compact
self-adjoint operator on L2 (S).
(7) Show that if g ∈ C 0 (S)) then Sg is twice continuously differentiable. Hint:
Proceed directly by differentiating the integral.
(8) From (3.217) conclude that S = F 2 where F is also a compact self-adjoint
1
operator on L2 (S) with eigenvalues (k 2 + 1)− 2 .
(9) Show that F : L2 (S) −→ C 0 (S).
(10) Now, going back to the real equation (3.211), we assume that V is contin-
uous, real-valued and 2π-periodic. Show that if u is a twice-differentiable
2π-periodic function satisfying (3.211) for a given f ∈ C 0 (S) then
(3.218) u + S((V − 1)u) = Sf and hence u = −F 2 ((V − 1)u) + F 2 f
and hence conclude that
(3.219) u = F v where v ∈ L2 (S) satisfies v + (F (V − 1)F )v = F f
where V − 1 is the operator defined by multiplication by V − 1.
(11) Show the converse, that if v ∈ L2 (S) satisfies
(3.220) v + (F (V − 1)F )v = F f, f ∈ C 0 (S)
then u = F v is 2π-periodic and twice-differentiable on R and satisfies
(3.211).
(12) Apply the Spectral theorem to F (V − 1)F (including why it applies) and
show that there is a sequence λj in R \ {0} with |λj | → 0 such that for all
λ ∈ C \ {0}, the equation
(3.221) λv + (F (V − 1)F )v = g, g ∈ L2 (S)
has a unique solution for every g ∈ L2 (S) if and only if λ 6= λj for any j.
(13) Show that for the λj the solutions of
(3.222) λj v + (F (V − 1)F )v = 0, v ∈ L2 (S),
are all continuous 2π-periodic functions on R.
(14) Show that the corresponding functions u = F v where v satisfies (3.222) are
all twice continuously differentiable, 2π-periodic functions on R satisfying
d2 u
(3.223) − + (1 − sj + sj V (x))u(x) = 0, sj = 1/λj .
dx2
(15) Conversely, show that if u is a twice continuously differentiable, 2π-periodic
function satisfying
d2 u
(3.224) − + (1 − s + sV (x))u(x) = 0, s ∈ C,
dx2
122 3. HILBERT SPACES

and u is not identically 0 then s = sj for some j.


(16) Finally, conclude that Fredholm’s alternative holds for the equation (3.211)
Theorem 16. For a given real-valued, continuous 2π-periodic func-
tion V on R, either (3.211) has a unique twice continuously differentiable,
2π-periodic, solution for each f which is continuous and 2π-periodic or
else there exists a finite, but positive, dimensional space of twice continu-
ously differentiable 2π-periodic solutions to the homogeneous equation
d2 w(x)
(3.225) − + V (x)w(x) = 0, x ∈ R,
dx2
R
and (3.211) has a solution if and only if (0,2π) f w = 0 for every 2π-
periodic solution, w, to (3.225).
Problem 3.23. Check that we really can understand all the 2π periodic eigen-
functions of the Schrödinger operator using the discussion above. First of all, there
was nothing sacred about the addition of 1 to −d2 /dx2 , we could add any positive
number and get a similar result – the problem with 0 is that the constants satisfy
the homogeneous equation d2 u/dx2 = 0. What we have shown is that the operator
d2 u
(3.226) u 7−→ Qu = − u+Vu
dx2
applied to twice continuously differentiable functions has at least a left inverse
unless there is a non-trivial solution of
d2 u
(3.227) − 2 u + V u = 0.
dx
Namely, the left inverse is R = F (Id +F (V −1)F )−1 F. This is a compact self-adjoint
operator. Show – and there is still a bit of work to do – that (twice continuously
differentiable) eigenfunctions of Q, meaning solutions of Qu = τ u are precisely the
non-trivial solutions of Ru = τ −1 u.
What to do in case (3.227) does have a non-trivial solution? Show that the
space of these is finite dimensional and conclude that essentially the same result
holds by working on the orthocomplement in L2 (S).
By now you should have become reasonably comfortable with a separable
Hilbert space such as l2 . However, it is worthwhile checking once again that it
is rather large – if you like, let me try to make you uncomfortable for one last time.
An important result in this direction is Kuiper’s theorem, which I will not ask you
to prove1. However, I want you to go through the closely related result sometimes
known as Eilenberg’s swindle. Perhaps you will appreciate the little bit of trickery.
First some preliminary results. Note that everything below is a closed curve in the
x ∈ [0, 1] variable – you might want to identify this with a circle instead, I just did
it the primitive way.
Problem 3.24. Let H be a separable, infinite dimensional Hilbert space. Show
that the direct sum of two copies of H is a Hilbert space with the norm
1
(3.228) H ⊕ H 3 (u1 , u2 ) 7−→ (ku1 k2H + ku2 k2H ) 2
1Kuiper’s theorem says that for any (norm) continuous map, say from any compact metric
space, g : M −→ GL(H) with values in the invertible operators on a separable infinite-dimensional
Hilbert space there exists a continuous map, an homotopy, h : M × [0, 1] −→ GL(H) such that
h(m, 0) = g(m) and h(m, 1) = IdH for all m ∈ M.
22. PROBLEMS 123

either by constructing an isometric isomorphism


(3.229) T : H −→ H ⊕ H, 1-1 and onto, kukH = kT ukH⊕H
or otherwise. In any case, construct a map as in (3.229).
Problem 3.25. One can repeat the preceding construction any finite number
of times. Show that it can be done ‘countably often’ in the sense that if H is a
separable, infinite dimensional, Hilbert space then
X
(3.230) l2 (H) = {u : N −→ H; kuk2l2 (H) = kui k2H < ∞}
i

has a Hilbert space structure and construct an explicit isometric isomorphism from
l2 (H) to H.
Problem 3.26. Recall, or perhaps learn about, the winding number of a closed
curve with values in C∗ = C \ {0}. We take as given the following fact:2 If Q =
[0, 1]N and f : Q −→ C∗ is continuous then for each choice of b ∈ C satisfying
exp(2πib) = f (0), there exists a unique continuous function F : Q −→ C satisfying
(3.231) exp(2πiF (q)) = f (q), ∀ q ∈ Q and F (0) = b.
Of course, you are free to change b to b + n for any n ∈ Z but then F changes to
F + n, just shifting by the same integer.
(1) Now, suppose c : [0, 1] −→ C∗ is a closed curve – meaning it is continuous
and c(1) = c(0). Let C : [0, 1] −→ C be a choice of F for N = 1 and
f = c. Show that the winding number of the closed curve c may be defined
unambiguously as
(3.232) wn(c) = C(1) − C(0) ∈ Z.
(2) Show that wn(c) is constant under homotopy. That is if ci : [0, 1] −→ C∗ ,
i = 1, 2, are two closed curves so ci (1) = ci (0), i = 1, 2, which are homo-
topic through closed curves in the sense that there exists f : [0, 1]2 −→ C∗
continuous and such that f (0, x) = c1 (x), f (1, x) = c2 (x) for all x ∈ [0, 1]
and f (y, 0) = f (y, 1) for all y ∈ [0, 1], then wn(c1 ) = wn(c2 ).
(3) Consider the closed curve Ln : [0, 1] 3 x 7−→ e2πix Idn×n of n×n matrices.
Using the standard properties of the determinant, show that this curve
is not homotopic to the identity through closed curves in the sense that
there does not exist a continuous map G : [0, 1]2 −→ GL(n), with values in
the invertible n × n matrices, such that G(0, x) = Ln (x), G(1, x) ≡ Idn×n
for all x ∈ [0, 1], G(y, 0) = G(y, 1) for all y ∈ [0, 1].
Problem 3.27. Consider the closed curve corresponding to Ln above in the
case of a separable but now infinite dimensional Hilbert space:
(3.233) L : [0, 1] 3 x 7−→ e2πix IdH ∈ GL(H) ⊂ B(H)
taking values in the invertible operators on H. Show that after identifying H with
H ⊕ H as above, there is a continuous map
(3.234) M : [0, 1]2 −→ GL(H ⊕ H)

2Of course, you are free to give a proof – it is not hard.


124 3. HILBERT SPACES

with values in the invertible operators and satisfying


(3.235)
M (0, x) = L(x), M (1, x)(u1 , u2 ) = (e4πix u1 , u2 ), M (y, 0) = M (y, 1), ∀ x, y ∈ [0, 1].
Hint: So, think of H ⊕ H as being 2-vectors (u1 , u2 ) with entries in H. This allows
one to think of ‘rotation’ between the two factors. Indeed, show that
(3.236) U (y)(u1 , u2 ) = (cos(πy/2)u1 + sin(πy/2)u2 , − sin(πy/2)u1 + cos(πy/2)u2 )
defines a continuous map [0, 1] 3 y 7−→ U (y) ∈ GL(H ⊕ H) such that U (0) = Id,
U (1)(u1 , u2 ) = (u2 , −u1 ). Now, consider the 2-parameter family of maps
(3.237) U −1 (y)V2 (x)U (y)V1 (x)
where V1 (x) and V2 (x) are defined on H ⊕ H as multiplication by exp(2πix) on the
first and the second component respectively, leaving the other fixed.
Problem 3.28. Using a rotation similar to the one in the preceeding problem
(or otherwise) show that there is a continuous map
(3.238) G : [0, 1]2 −→ GL(H ⊕ H)
such that
(3.239) G(0, x)(u1 , u2 ) = (e2πix u1 , e−2πix u2 ),
G(1, x)(u1 , u2 ) = (u1 , u2 ), G(y, 0) = G(y, 1) ∀ x, y ∈ [0, 1].
Problem 3.29. Now, think about combining the various constructions above
in the following way. Show that on l2 (H) there is an homotopy like (3.238), G̃ :
[0, 1]2 −→ GL(l2 (H)), (very like in fact) such that
∞ ∞
(3.240) G̃(0, x) {uk }k=1 = exp((−1)k 2πix)uk k=1 ,


G̃(1, x) = Id, G̃(y, 0) = G̃(y, 1) ∀ x, y ∈ [0, 1].


Problem 3.30. “Eilenberg’s swindle” For an infinite dimenisonal separable
Hilbert space, construct an homotopy – meaning a continuous map G : [0, 1]2 −→
GL(H) – with G(0, x) = L(x) in (3.233) and G(1, x) = Id and of course G(y, 0) =
G(y, 1) for all x, y ∈ [0, 1].
Hint: Just put things together – of course you can rescale the interval at the end
to make it all happen over [0, 1]. First ‘divide H into 2 copies of itself’ and deform
from L to M (1, x) in (3.235). Now, ‘divide the second H up into l2 (H)’ and apply
an argument just like the preceding problem to turn the identity on this factor into
alternating terms multiplying by exp(±4πix) – starting with −. Now, you are on
H ⊕ l2 (H), ‘renumbering’ allows you to regard this as l2 (H) again and when you
do so your curve has become alternate multiplication by exp(±4πix) (with + first).
Finally then, apply the preceding problem again, to deform to the identity (always
of course through closed curves). Presto, Eilenberg’s swindle!
Problem 3.31. Check that we really can understand all the 2π periodic eigen-
functions of the Schrödinger operator using the discussion above. First of all, there
was nothing sacred about the addition of 1 to −d2 /dx2 , we could add any positive
number and get a similar result – the problem with 0 is that the constants satisfy
the homogeneous equation d2 u/dx2 = 0. What we have shown is that the operator
d2 u
(3.241) u 7−→ Qu = − u+Vu
dx2
23. SOLUTIONS TO PROBLEMS 125

applied to twice continuously differentiable functions has at least a left inverse


unless there is a non-trivial solution of
d2 u
(3.242) − 2 u + V u = 0.
dx
Namely, the left inverse is R = F (Id +F (V −1)F )−1 F. This is a compact self-adjoint
operator. Show – and there is still a bit of work to do – that (twice continuously
differentiable) eigenfunctions of Q, meaning solutions of Qu = τ u are precisely the
non-trivial solutions of Ru = τ −1 u.
What to do in case (3.242) does have a non-trivial solution? Show that the
space of these is finite dimensional and conclude that essentially the same result
holds by working on the orthocomplement in L2 (S).

23. Solutions to problems


CHAPTER 4

Applications

The last part of the course includes some applications of Hilbert space – the
completeness of the Fourier basis, some spectral theory for operators on an interval
or the circle and enough of a treatment of the eigenfunctions for the harmonic
oscillator to show that the Fourier transform in an isomorphism on L2 (R). Once
one has all this, one can do a lot more, but there is no time left. Such is life.

1. Fourier series and L2 (0, 2π).


Let us now try applying our knowledge of Hilbert space to a concrete Hilbert
space such as L2 (a, b) for a finite interval (a, b) ⊂ R. You showed that this is indeed a
Hilbert space. One of the reasons for developing Hilbert space techniques originally
was precisely the following result.
Theorem 17. If u ∈ L2 (0, 2π) then the Fourier series of u,
Z
1 X
(4.1) ikx
ck e , ck = u(x)e−ikx dx
2π (0,2π)
k∈Z

converges in L2 (0, 2π) to u.


Notice that this does not say the series converges pointwise, or pointwise almost
everywhere since this need not be true – depending on u. We are just claiming that
Z
1 X
(4.2) lim |u(x) − ck eikx |2 = 0
n→∞ 2π
|k|≤n

for any u ∈ L2 (0, 2π).


Our abstract Hilbert space theory has put us quite close to proving this. First
observe that if e0k (x) = exp(ikx) then these elements of L2 (0, 2π) satisfy
Z 2π (
if k 6= j
Z
0 0 0
(4.3) ek ej = exp(i(k − j)x) =
0 2π if k = j.
Thus the functions
e0k 1
(4.4) ek = = √ eikx
ke0k k 2π
form an orthonormal set in L2 (0, 2π). It follows that (4.1) is just the Fourier-Bessel
series for u with respect to this orthonormal set:-
√ 1
(4.5) ck = 2π(u, ek ) =⇒ ck eikx = (u, ek )ek .

So, we already know that this series converges in L2 (0, 2π) thanks to Bessel’s iden-
tity. So ‘all’ we need to show is
127
128 4. APPLICATIONS

Proposition 43. The ek , k ∈ Z, form an orthonormal basis of L2 (0, 2π), i.e.


are complete:
Z
(4.6) ueikx = 0 ∀ k =⇒ u = 0 in L2 (0, 2π).

This however, is not so trivial to prove. An equivalent statement is that the fi-
nite linear span of the ek is dense in L2 (0, 2π). I will prove this using Fejér’s method.
In this approach, we check that any continuous function on [0, 2π] satisfying the
additional condition that u(0) = u(2π) is the uniform limit on [0, 2π] of a sequence
in the finite span of the ek . Since uniform convergence of continuous functions cer-
tainly implies convergence in L2 (0, 2π) and we already know that the continuous
functions which vanish near 0 and 2π are dense in L2 (0, 2π) this is enough to prove
Proposition 43. However the proof is a serious piece of analysis, at least it is to
me! There are other approaches, for instance we could use the Stone-Weierstrass
Theorem.
So, the problem is to find the sequence in the span of the ek . The trick is to use
the Fourier expansion that we want to check. The idea of Cesàro is to make this
Fourier expansion ‘converge faster’, or maybe better. For the moment we can work
with a general function u ∈ L2 (0, 2π) – or think of it as continuous if you prefer.
So the truncated Fourier series is
Z
1 X
(4.7) Un (x) = ( u(t)e−ikt dt)eikx
2π (0,2π)
|k|≤n

where I have just inserted the definition of the ck ’s into the sum. This is just a
finite sum so we can treat x as a parameter and use the linearity of the integral to
write this as
Z
1 X iks
(4.8) Un (x) = Dn (x − t)u(t), Dn (s) = e .
(0,2π) 2π
|k|≤n

Now this sum can be written as an explicit quotient, since, by telescoping,


1 1
(4.9) 2πDn (s)(eis/2 − e−is/2 ) = ei(n+ 2 )s − e−i(n+ 2 )s .
So in fact, at least where s 6= 0,
1 1
ei(n+ 2 )s − e−i(n+ 2 )s
(4.10) Dn (s) =
2π(eis/2 − e−is/2 )
and of course the limit as s → 0 exists just fine.
As I said, Cesàro’s idea is to speed up the convergence by replacing Un by its
average
n
1 X
(4.11) Vn (x) = Ul .
n+1
l=0

Again plugging in the definitions of the Ul ’s and using the linearity of the integral
we see that
Z n
1 X
(4.12) Vn (x) = Sn (x − t)u(t), Sn (s) = Dl (s).
(0,2π) n+1
l=0
1. FOURIER SERIES AND L2 (0, 2π). 129

So again we want to compute a more useful form for Sn (s) – which is the Fejér
kernel. Since the denominators in (4.10) are all the same,
n n
1 1
X X
(4.13) 2π(n + 1)(eis/2 − e−is/2 )Sn (s) = ei(n+ 2 )s − e−i(n+ 2 )s .
l=0 l=0

Using the same trick again,


n
1
X
(4.14) (eis/2 − e−is/2 ) ei(n+ 2 )s = ei(n+1)s − 1
l=0
so
2π(n + 1)(eis/2 − e−is/2 )2 Sn (s) = ei(n+1)s + e−i(n+1)s − 2
(n+1)
(4.15) 1 sin2 ( 2 s)
=⇒ Sn (s) = .
n + 1 2π sin2 ( 2s )
Now, what can we say about this function? One thing we know immediately is
that if we plug u = 1 into the disucssion above, we get Un = 1 for n ≥ 0 and hence
Vn = 1 as well. Thus in fact
Z
(4.16) Sn (x − ·) = 1.
(0,2π)

Looking directly at (4.15) the first thing to notice is that Sn (s) ≥ 0. Also, we
can see that the denominator only vanishes when s = 0 or s = 2π in [0, 2π]. Thus
if we stay away from there, say s ∈ (δ, 2π − δ) for some δ > 0 then – since sin(t) is
a bounded function
(4.17) |Sn (s)| ≤ (n + 1)−1 Cδ on (δ, 2π − δ).
We are interested in how close Vn (x) is to the given u(x) in supremum norm,
where now we will take u to be continuous. Because of (4.16) we can write
Z
(4.18) u(x) = Sn (x − t)u(x)
(0,2π)

where t denotes the variable of integration (and x is fixed in [0, 2π]). This ‘trick’
means that the difference is
Z
(4.19) Vn (x) − u(x) = Sx (x − t)(u(t) − u(x)).
(0,2π)

For each x we split this integral into two parts, the set Γ(x) where x − t ∈ [0, δ] or
x − t ∈ [2π − δ, 2π] and the remainder. So
(4.20) Z Z
|Vn (x) − u(x)| ≤ Sx (x − t)|u(t) − u(x)| + Sx (x − t)|u(t) − u(x)|.
Γ(x) (0,2π)\Γ(x)

Now on Γ(x) either |t − x| ≤ δ – the points are close together – or t is close to 0 and
x to 2π so 2π − x + t ≤ δ or conversely, x is close to 0 and t to 2π so 2π − t + x ≤ δ.
In any case, by assuming that u(0) = u(2π) and using the uniform continuity of a
continuous function on [0, 2π], given  > 0 we can choose δ so small that
(4.21) |u(x) − u(t)| ≤ /2 on Γ(x).
130 4. APPLICATIONS

On the complement of Γ(x) we have (4.17) and since u is bounded we get the
estimate
Z
(4.22) |Vn (x)−u(x)| ≤ /2 Sn (x−t)+(n+1)−1 C 0 (δ) ≤ /2+(n+1)−1 C 0 (δ).
Γ(x)

Here the fact that Sn is non-negative and has integral one has been used again to
estimate the integral of Sn (x − t) over Γ(x) by 1. Having chosen δ to make the first
term small, we can choose n large to make the second term small and it follows
that
(4.23) Vn (x) → u(x) uniformly on [0, 2π] as n → ∞
under the assumption that u ∈ C([0, 2π]) satisfies u(0) = u(2π).
So this proves Proposition 43 subject to the density in L2 (0, 2π) of the contin-
uous functions which vanish near (but not of course in a fixed neighbourhood of)
the ends. In fact we know that the L2 functions which vanish near the ends are
dense since we can chop off and use the fact that
Z Z !
(4.24) lim |f |2 + |f |2 = 0.
δ→0 (0,δ) (2π−δ,2π)

The L2 functions which vanish near the ends are in the closure of the span of
the step functions which vanish near the ends. Each such step function can be
approximated in L2 ((0, 2π)) by a continuous function which vanishes near the ends
so we are done as far as density is concerned. This proves Theorem 17.

2. Dirichlet problem on an interval


I want to do a couple more ‘serious’ applications of what we have done so
far. There are many to choose from, and I will mention some more, but let me
first consider the Diriclet problem on an interval. I will choose the interval [0, 2π]
because we looked at it before. So, we supposed to be trying to solve
d2 u(x)
(4.25) − + V (x)u(x) = f (x) on (0, 2π), u(0) = u(2π) = 0
dx2
where the last part are the Dirichlet boundary conditions. I will assume that
(4.26) V : [0, 2π] −→ R is continuous and real-valued.
Now, it certainly makes sense to try to solve the equation (4.25) for say a given
f ∈ C 0 ([0, 2π]), looking for a solution which is twice continuously differentiable on
the interval. It may not exist, depending on V but one thing we can shoot for,
which has the virtue of being explicit, is the following:
Proposition 44. If V ≥ 0 as in (4.26) then for each f ∈ C 0 ([0, 2π]) there
exists a unique twice continuously differentiable solution, u, to (4.25).
You will see that it is a bit hard to approach this directly – especially if you
have some ODE theory at your fingertips. There are in fact various approaches
to this but we want to go through L2 theory – not surprisingly of course. How to
start?
Well, we do know how to solve (4.25) if V ≡ 0 since we can use (Riemann)
integration. Thus, ignoring the boundary conditions for the moment, we can find
2. DIRICHLET PROBLEM ON AN INTERVAL 131

a solution to −d2 v/dx2 = f on the interval by integrationg twice:


Z xZ y
(4.27) v(x) = − f (t)dtdy satifies − d2 v/dx2 = f on (0, 2π).
0 0
Moroever v really is twice continuously differentiable if f is continuous. So, what
has this got to do with operators? Well, we can change the order of integration in
(4.27) to write v as
Z xZ x Z 2π
(4.28) v(x) = − f (t)dydt = a(x, t)f (t)dt, a(x, t) = (t − x)H(x − t)
0 t 0

where the Heaviside function H(y) is 1 when y ≥ 0 and 0 when y < 0. Thus a(x, t)
is actually continuous on [0, 2π] × [0, 2π] since the t − x factor vanishes at the jump
in H(t − x). So (4.28) shows that v is given by applying an integral operator, with
continuous kernel on the square, to f.
Before thinking more seriously about this, recall that there is also the matter
of the conditions. Clearly, v(0) = 0 since we integrated from there. However, there
is no particular reason why
Z 2π
(4.29) v(2π) = (t − 2π)f (t)dt
0
should vanish. However, we can always add to v any linear function and still satify
the differential equation. Since we do not want to spoil the vanishing at x = 0 we
can only afford to add cx but if we choose c correctly, namely consider
Z 2π
1
(4.30) c= (2π − t)f (t)dt, then (v + cx)(2π) = 0.
2π 0
So, finally the solution we want is
Z 2π
tx
(4.31) w(x) = b(x, t)f (t)dt, b(x, t) = min(t, x) − ∈ C([0, 2π]2 )
0 2π
with the formula for b following by simpleo manipulation from
tx
(4.32) b(x, t) = a(x, t) + x −

Thus there is the unique, twice continuously differentiable, solution of −d2 w/dx2 =
f in (0, 2π) which vanishes at both end points and it is given by the integral operator
(4.31).
Lemma 38. The integral operator (4.31) extends by continuity from C 0 ([0, 2π])
to a compact, self-adjoint operator on L2 (0, 2π).
Proof. Since w is given by an integral operator with a continuous real-valued
kernel which is even in the sense that (check it)
(4.33) b(t, x) = b(x, t)
we might as well give a more general result. 

Proposition 45. If b ∈ C 0 ([0, 2π]2 ) then


Z 2π
(4.34) Bf (x) = b(x, t)f (t)dt
0
132 4. APPLICATIONS

defines a compact operator on L2 (0, 2π) if in addition b satisfies


(4.35) b(t, x) = b(x, t)
then B is self-adjoint.
Proof. If f ∈ L2 ((0, 2π)) and v ∈ C([0, 2π]) then the product vf ∈ L2 ((0, 2π))
and kvf kL2 ≤ kvk∞ kf kL2 . This can be seen for instance by taking an absolutely
summable approcimation to f, which gives a sequence of continuous functions con-
verging a.e. to f and bounded by a fixed L2 function and observing that vfn → vf
a.e. with bound a constant multiple, sup |v|, of that function. It follows that for
b ∈ C([0, 2π]) the product
(4.36) b(x, y)f (y) ∈ L2 ((0, 2π))
for each x ∈ [0, 2π]. Thus Bf (x) is well-defined by (4.35) since L2 ((0, 2π) ⊂
L2 ((0, 2π)).
Not only that, but Bf ∈ C([0, 2π]) as can be seen from Schwarz inequality,
(4.37) Z
1
|Bf (x0 ) − Bf (x)| = | (b(x0 , y) − b(x, y))f (y)| ≤ sup |b(x0 , y − b(x, y)|(2π) 2 kf kL2 .
y

Essentially the same estimate shows that


1
(4.38) sup kBf (x)k ≤ (2π) 2 sup |b|kf kL2
x (x,y)
2
so indeed, B : L ((0, 2π)) −→ C([0, 2π]) is a bounded linear operator.
When b satisfies (4.35) and f and g are continuous
Z Z
(4.39) BF (x)g(x) = f (x)Bg(x)

and the general case follows by approximation by continuous functions.


So, we need to see the compactness. If we fix x then b(x, y) ∈ C([0, 2π]) and
then if we let x vary,
(4.40) [0, 2π] 3 x 7−→ b(x, ·) ∈ C([0, 2π])
is continuous as a map into this Banach space. Again this is the uniform continuity
of a continuous function on a compact set, which shows that
(4.41) sup kb(x0 , y) − b(x, y)| → 0 as x0 → x.
y

Since the inclusion map C([0, 2π]) −→ L2 (90, 2π)) is bounded, i.e continuous, it
follows that the map (I have reversed the variables)
(4.42) [0, 2π] 3 y 7−→ b(·, y) ∈ L2 ((0, 2π))
is continuous and so has a compact range.
Take the Fourier basis ek for [0, 2π] and expand b in the first variable. Given
 > 0 the compactness of the image of (4.42) implies that for some N
X
(4.43) |(b(x, y), ek (x))|2 <  ∀ y ∈ [0, 2π].
|k|>N

The finite part of the Fourier series is continuous as a function of both arguments
X
(4.44) bN (x, y) = ek (x)ck (y), ck (y) = (b(x, y), ek (x))
|k|≤N
2. DIRICHLET PROBLEM ON AN INTERVAL 133

and so defines another bounded linear operator BN as before. This operator can
be written out as
X Z
(4.45) BN f (x) = ek (x) ck (y)f (y)dy
|k|≤N

and so is of finite rank – it always takes values in the span of the first 2N + 1
trigonometric functions. On the other hand the remained is given by a similar
operator with corresponding to qN = b − bN and this satisfies
(4.46) sup kqN (·, y)kL2 ((0,2π)) → 0 as N → ∞.
y

Thus, qN has small norm as a bounded operator on L2 ((0, 2π)) so B is compact –


it is the norm limit of finite rank operators. 
Now, recall from one of the Problem sets that uk = c sin(kx/2), k ∈ N, is
also an orthonormal basis for L2 (0, 2π) (it is not the Fouier basis!) Moreover,
differentiating we find straight away that
d2 uk k2
(4.47) − = uk .
dx2 4
Since of course uk (0) = 0 = uk (2π) as well, from the uniqueness above we conclude
that
4
(4.48) Buk = 2 uk ∀ k.
k
Thus, in this case we know the orthonormal basis of eigenfunctions for B. They
are the uk , each eigenspace is 1 dimensional and the eigenvalues are 4k −2 . So, this
happenstance allows us to decompose B as the square of another operator defined
directly on the othornormal basis. Namely
2
(4.49) Auk = uk =⇒ B = A2 .
k
Here again it is immediate that A is a compact self-adjoint operator on L2 (0, 2π)
since its eigenvalues tend to 0. In fact we can see quite a lot more than this.
Lemma 39. The operator A maps L2 (0, 2π) into C 0 ([0, 2π]) and Af (0) =
Af (2π) = 0 for all f ∈ L2 (0, 2π).
Proof. If f ∈ L2 (0, 2π) we may expand it in Fourier-Bessel series in terms of
the uk and find
X
(4.50) f= ck uk , {ck } ∈ l2 .
k
Then of course, by definition,
X 2ck
(4.51) Af = uk .
k
k
Here each uk is a bounded continuous function, with the bound on uk being in-
dependent of k. So in fact (4.51) converges uniformly and absolutely since it is
uniformly Cauchy, for any q > p,
  12
q q q
X 2ck X X
(4.52) | uk | ≤ 2|c| |ck |k −1 ≤ 2|c|  k −2  kf kL2
k
k=p k=p k=p
134 4. APPLICATIONS

where Cauchy-Schwartz has been used. This proves that

A : L2 (0, 2π) −→ C 0 ([0, 2π])


is bounded and by the uniform convergence uk (0) = uk (2π) = 0 for all k implies
that Af (0) = Af (2π) = 0. 

So, going back to our original problem we try to solve (4.25) by moving the V u
term to the right side of the equation (don’t worry about regularity yet) and hope
to use the observation that
(4.53) u = −A2 (V u) + A2 f
should satisfy the equation and boundary conditions. In fact, let’s hope that u =
Av, which has to be true if (4.53) holds with v = −AV u + Af, and look instead for
(4.54) v = −AV Av + Af =⇒ (Id +AV A)v = Af.
So, we know that multiplication by V, which is real and continuous, is a bounded
self-adjoint operator on L2 (0, 2π). Thus AV A is a self-adjoint compact operator so
we can apply our spectral theory to Id +AV A. It has a complete orthonormal basis
of eigenfunctions and is invertible if and only if it has trivial null space.
An element of the null space would have to satisfy u = −AV Au. On the other
hand we know that AV A is positive since
Z Z
(4.55) (AV Aw, w) = (V Av, Av) = V (x)|Av|2 ≥ 0 =⇒ |u|2 = 0,
(0,2π) (0,2π)

using the non-negativity of V. So, there can be no null space – all the eigenvalues
of AV A are at least non-negative so −1 is not amongst them.
Thus Id +AV A is invertible on L2 (0, 2π) with inverse of the form Id +Q, Q
again compact and self-adjoint. So, to solve (4.54) we just need to take
(4.56) v = (Id +Q)Af ⇐⇒ v + AV Av = Af in L2 (0, 2π).
Then indeed
(4.57) u = Av satisfies u + A2 V u = A2 f.
In fact since v ∈ L2 (0, 2π) from (4.56) we already know that u ∈ C 0 ([0, 2π]) vanishes
at the end points.
Moreover if f ∈ C 0 ([0, 2π]) we know that Bf = A2 f is twice continuously
differentiable, since it is given by integrations – that is where B came from. Now,
we know that u is L2 satisfies u = −A2 (V u) + A2 f. Since V u ∈ L2 ((0, 2π) so is
A(V u) and then, as seen above, A(A(V u) is continuous. So combining this with
the result about A2 f we see that u itself is continuous and hence so is V u. But
then, going through the routine again
(4.58) u = −A2 (V u) + A2 f
is the sum of two twice continuously differentiable functions. Thus is so itself. In
fact from the properties of B = A2 it satisifes
d2 u
(4.59) − = −V u + f
dx2
2. DIRICHLET PROBLEM ON AN INTERVAL 135

which is what the result claims. So, we have proved the existence part of Proposi-
tion 44. The uniqueness follows pretty much the same way. If there were two twice
continuously differentiable solutions then the difference w would satisfy
d2 w
(4.60) − + V w = 0, w(0) = w(2π) = 0 =⇒ w = −Bw = −A2 V w.
dx2
Thus w = Aφ, φ = −AV w ∈ L2 (0, 2π). Thus φ in turn satisfies φ = AV Aφ and
hence is a solution of (Id +AV A)φ = 0 which we know has none (assuming V ≥ 0).
Since φ = 0, w = 0.
This completes the proof of Proposition 44. To summarize what we have shown
is that Id +AV A is an invertible bounded operator on L2 (0, 2π) (if V ≥ 0) and then
the solution to (4.25) is precisely
(4.61) u = A(Id +AV A)−1 Af
which is twice continuously differentiable and satisfies the Dirichlet conditions for
each f ∈ C 0 ([0, 2π]).
Now, even if we do not assume that V ≥ 0 we pretty much know what is
happening.
Proposition 46. For any V ∈ C 0 ([0, 2π]) real-valued, there is an orthonormal
basis wk of L2 (0, 2π) consisting of twice-continuously differentiable functions on
2
[0, 2π], vanishing at the end-points and satisfying − ddxw2k + V wk = Tk wk where
Tk → ∞ as k → ∞. The equation (4.25) has a (twice continuously differentiable)
solution for given f ∈ C 0 ([0, 2π]) if and only if
Z
(4.62) TK = 0 =⇒ f wk = 0,
(0,2π)

i.e. f is orthogonal to the null space of Id +A2 V, which is the image under A of
the null space of Id +AV A, in L2 (0, 2π).
Proof. Notice the form of the solution in case V ≥ 0 in (4.61). In general, we
can choose a constant c such that V + c ≥ 0. Then the equation
d2 w d2 w
(4.63) − 2
+ V w = T wk ⇐⇒ − 2 + (V + c)w = (T + c)w.
dx dx
Thus, if w satisfies this eigen-equation then it also satisfies

(4.64) w = (T + c)A(Id +A(V + c)A)−1 Aw ⇐⇒


Sw = (T + c)−1 w, S = A(Id +A(V + c)A)−1 A.

Now, we have shown that S is a compact self-adjoint operator on L2 (0, 2π) so we


know that it has a complete set of eigenfunctions, ek , with eigenvalues τk 6= 0. From
the discussion above we then know that each ek is actually continuous – since it is
Aw0 with w0 ∈ L2 (0, 2π) and hence also twice continuously differentiable. So indeed,
these ek satisfy the eigenvalue problem (with Dirichlet boundary conditions) with
eigenvalues
(4.65) Tk = τk−1 + c → ∞ as k → ∞.
The solvability part also follows in much the same way. 
136 4. APPLICATIONS

3. Harmonic oscillator
As a second ‘serious’ application of our Hilbert space theory – but not this time
the spectral theorem, at least not immediately, I want to discuss the Hermite basis
for L2 (R). Note that so far we have not found an explicit orthonormal basis on the
whole real line, even though we know L2 (R) to be separable, so we certainly know
that such a basis exists. How to construct one explicitly and with some handy
properties? One way is to simply orthonormalize – using Gram-Schmidt – some
countable set with dense span. For instance consider the basic Gaussian function
x2
(4.66) exp(−) ∈ L2 (R).
2
This is so rapidly decreasing at infinity that the product with any polynomial is
also square integrable:
x2
(4.67) xk exp(−) ∈ L2 (R) ∀ k ∈ N0 = {0, 1, 2, . . . }.
2
Orthonormalizing this sequence gives an orthonormal basis, where completeness
can be shown by an appropriate approximation technique but as usual is not so
simple.
Rather than proceed directly we will work up to this by discussing the eigen-
functions of the harmonic oscillator
d2
(4.68) H = − 2 + x2
dx
which we want to think of as an operator – although for the moment I will leave
vague the question of what it operates on.
The first thing to observe is that the Gaussian is an eigenfunction of H
2 d 2 2
(4.69) He−x /2
=− (−xe−x /2 + x2 e−x /2
dx
2 2 2
− (x2 − 1)e−x /2 + x2 e−x /2 = e−x /2
with eigenvalue 1 – for the moment this is only in a formal sense.
In this special case there is an essentially algebraic way to generate a whole
sequence of eigenfunctions from the Gaussian. To do this, write
d d
(4.70) Hu = (− + x)( + x)u + u = (CA + 1)u,
dx dx
d d
C = (− + x), A = ( + x)
dx dx
again formally as operators. Then note that
2
(4.71) Ae−x /2
=0
which again proves (4.69). The two operators in (4.70) are the ‘creation’ operator
and the ‘annihilation’ operator. They almost commute in the sense that
(4.72) (AC − CA)u = 2u
for say any twice continuously differentiable function u.
2
Now, set u0 = e−x /2 which is the ‘ground state’ and consider u1 = Cu0 . From
(4.72), (4.71) and (4.70),
(4.73) Hu1 = (CAC + C)u0 = C 2 Au0 + 3Cu0 = 3u1 .
3. HARMONIC OSCILLATOR 137

Thus, u1 is an eigenfunction with eigenvalue 3.


Lemma 40. For j ∈ N0 = {0, 1, 2, . . . } the function uj = C j u0 satisfies Huj =
(2j + 1)uj .
Proof. This follows by induction on j, where we know the result for j = 0
and j = 1. Then
(4.74) HCuj = (CA + 1)Cuj = C(H − 1)uj + 3Cuj = (2j + 3)uj .

j j −x2 /2
Again by induction we can check that uj = (2 x + qj (x))e where qj is a
polynomial of degree at most j − 2. Indeed this is true for j = 0 and j = 1 (where
q0 = q1 ≡ 0) and then
2
(4.75) Cuj = (2j+1 xj+1 + Cqj )e−x /2
.
and qj+1 = Cqj is a polynomial of degree at most j − 1 – one degree higher than
qj .
From this it follows in fact that the finite span of the uj consists of all the
2
products p(x)e−x /2 where p(x) is any polynomial.
Now, all these functions are in L2 (R) and we want to compute their norms.
First, a standard integral computation1 shows that

Z Z
2 2
(4.76) (e−x /2 )2 = e−x = π
R R
For j > 0, integration by parts (easily justified by taking the integral over [−R, R]
and then letting R → ∞) gives
Z Z Z
(4.77) (C j u0 )2 = C j u0 (x)C j u0 (x)dx = u0 Aj C j u0 .
R R R
Now, from (4.72), we can move one factor of A through the j factors of C until it
emerges and ‘kills’ u0
(4.78) AC j u0 = 2C j−1 u+ CAC j−1 u0 = 2C j−1 u0 + C 2 AC j−2 u0 = 2jC j−1 u0 .
So in fact,

Z Z
(4.79) (C j u0 )2 = 2j (C j−1 u0 )2 = 2j j! π.
R R
A similar argument shows that
Z Z
(4.80) uk uj = u0 Ak C j u0 = 0 if k 6= j.
R R
Thus the functions
1 1 2
(4.81) ej = 2−j/2 (j!)− 2 π − 4 C j e−x /2

form an orthonormal sequence in L2 (R).

1To compute the Gaussian integral, square it and write as a double integral then introduct
polar coordinates
Z Z Z ∞ Z 2π
2 2 2 2 2 ∞
( e−x dx)2 = e−x −y dxdy = e−r rdrdθ = π − e−r 0 = π.

R R2 0 0
138 4. APPLICATIONS

We would like to show this orthonormal sequence is complete. Rather than


argue through approximation, we can guess that in some sense the operator
d d d2
(4.82) AC = ( + x)(− + x) = − 2 + x2 + 1
dx dx dx
should be invertible, so one approach ist to try to construct its ‘inverse’ and show
this really is a compact, self-adjoint operator on L2 (R) and that its only eigenfunc-
tions are the ei in (4.81). Rather than do this I will proceed more indirectly.

4. Completeness of Hermite basis


Starting from the ground state for the harmonic oscillator
d2 2
(4.83) H=− + x2 , Hu0 = u0 , u0 = e−x /2
dx2
and using the creation and annihilation operators
d d
(4.84) A= + x, C = − + x, AC − CA = 2 Id, H = CA + Id
dx dx
we have constructed the higher eigenfunctions:
(4.85) uj = C j u0 = pj (x)u0 (c), p(x) = 2j xj + l.o.ts, Huj = (2j + 1)uj
and shown that these are orthogonal, uj ⊥ uk , j 6= k, and so when normalized give
an orthonormal system in L2 (R) :
uj
(4.86) ej = 1 1 .
j/2
2 (j!) 2 π 4
Now, what we want, is that these ej form an orthonormal basis of L2 (R),
meaning they are complete as an orthonormal sequence. There are various proofs
of this, but the only ‘simple’ ones I know involve the Fourier inversion formula and
I want to use the completeness to prove the Fourier inversion formula, so that will
not do. Instead I want to use a version of Mehler’s formula.
To show the completeness of the ej ’s it is enough to find a compact self-adjoint
operator with these as eigenfunctions and no null space. It is the last part which
is tricky. The first part is easy. Remembering that all the ej are real, we can find
an operator with the ej ;s as eigenfunctions with corresponding eigenvalues λj > 0
(say) by just defining
X∞ ∞
X Z
(4.87) Au(x) = λj (u, ej )ej (x) = λj ej (x) ej (y)u(y).
j=0 j=0

For this to be a compact operator we need λj → 0 as j → ∞, although for bound-


edness we just need the λj to be bounded. So, the problem with this is to show
that A has no null space – which of course is just the completeness of the e0j since
(assuming all the λj are positive)
(4.88) Au = 0 ⇐⇒ u ⊥ ej ∀ j.
Nevertheless, this is essentially what we will do. The idea is to write A as an
integral operator and then work with that. I will take the λj = wj where w ∈ (0, 1).
The point is that we can find an explicit formula for
X∞
(4.89) Aw u = wj ej (x)ej (y) = A(w, x, y).
j=0
4. COMPLETENESS OF HERMITE BASIS 139

To find A(w, x, y) we will need to compute the Fourier transforms of the ej .


Recall that
F : L1 (R) −→ C∞
0
(R), F(u) = û,
(4.90)
Z
û(ξ) = e−ixξ u(x), sup |û| ≤ kukL1 .

Lemma 41. The Fourier transform of u0 is



(4.91) (Fu0 )(ξ) = 2πu0 (ξ).
Proof. The Fourier transform is the limit of a Riemann integral
Z R
(4.92) û0 (ξ) = lim eiξx u0 (x)
R→∞ −R

since u0 (x) is continuous and rapidly decreasing at infinity. Now, for such a Rie-
mann integral we can differentiate under the integral sign and see that
Z R
d
û0 (ξ) = lim ixeiξx u0 (x)
dξ R→∞ −R
Z R
d
= lim i eiξx (− u0 (x)
(4.93) R→∞ −R dx
Z R Z R
d iξx
eiξx u0 (x)

= lim −i e u0 (x) − ξ lim
R→∞ −R dx R→∞ −R

= −ξ û0 (ξ).
Maybe this deserves a little more discussion! Still, this means that û0 is annihilated
by A :
d
(4.94) ( + ξ)û0 (ξ) = 0.

Thus, it follows that û0 (ξ) = c exp(−ξ 2 /2) since these are the only functions in
annihilated by A. The constant is easy to compute, since
Z
2 √
(4.95) û0 (0) = e−x /2 dx = 2π

proving (4.91). 

We can use this formula, of if you like the argument to prove it, to show that
2 √ 2
(4.96) v = e−x /4 =⇒ v̂ = πe−ξ .
Changing the names of the variables this just says
Z
2 1 2
(4.97) e−x = √ eixs−s /4 ds.
2 π R
The definition of the uj ’s can be rewritten
d 2 2 d 2
(4.98) uj (x) = (− + x)j e−x /2 = ex /2 (− )j e−x
dx dx
2
as is easy to see inductively – the point being that ex /2 is an integrating factor for
the creation operator. Plugging this into (4.97) and carrying out the derivatives –
140 4. APPLICATIONS

which is legitimate since the integral is so strongly convergent – gives


2
ex /2
Z
2
(4.99) uj (x) = √ (−is)j eixs−s /4 ds.
2 π R
Now we can use this formula twice on the sum on the left in (4.89) and insert
the normalizations in (4.86) to find that
∞ ∞ 2 2
ex /2+y /2 (−1)j wj sj tj isx+ity−s2 /4−t2 /4
X X Z
(4.100) wj ej (x)ej (y) = e dsdt.
j=0 j=0
4π 3/2 R2 2j j!

The crucial thing here is that we can sum the series to get an exponential, this
allows us to finally conclude:
Lemma 42. The identity (4.89) holds with
 
1 1−w 2 1+w 2
(4.101) A(w, x, y) = √ √ exp − (x + y) − (x − y)
π 1 − w2 4(1 + w) 4(1 − w)
Proof. Summing the series in (4.100) we find that
2 2
ex /2+y /2
Z
1
(4.102) A(w, x, y) = 3/2
exp(− wst + isx + ity − s2 /4 − t2 /4)dsdt.
4π R2 2
Now, we can use the same formula as before for the Fourier transform of u0 to
evaluate these integrals explicitly. I think the clever way, better than what I did in
lecture, is to change variables by setting
√ √
(4.103) s = (S + T )/ 2, t = (S − T )/ 2 =⇒ dsdt = dSdT,
1 x+y 1 x−y 1
− wst + isx + ity − s2 /4 − t2 /4 = iS √ − (1 + w)S 2 iT √ − (1 − w)T 2 .
2 2 4 2 4
The formula for the Fourier transform of exp(−x2 ) can be used, after a change of
variable, to conclude that

(x + y)2
Z
x+y 1 2 2 π
exp(iS √ − (1 + w)S )dS = p exp(− )
R 2 4 (1 + w) 2(1 + w)
(4.104) √
x−y 1 (x − y)2
Z
2 π
exp(iT √ − (1 − w)T 2 )dT = p exp(− ).
R 2 4 (1 − w) 2(1 − w)
Inserting these formulæ back into (4.102) gives
(x + y)2 (x − y)2 x2 y2
 
1
(4.105) A(w, x, y) = √ √ exp − − + +
π 1 − w2 2(1 + w) 2(1 − w) 2 2
which after a little adjustment gives (4.101). 

Now, this explicit representation of Aw as an integral operator allows us to


show
Proposition 47. For all real-valued f ∈ L2 (R),

X
(4.106) |(u, ej )|2 = kf k2L2 .
j=1
4. COMPLETENESS OF HERMITE BASIS 141

Proof. By definition of Aw

X
(4.107) |(u, ej )|2 = lim(f, Aw f )
w↑1
j=1

so (4.106) reduces to
(4.108) lim(f, Aw f ) = kf k2L2 .
w↑1

To prove (4.108) we will make our work on the integral operators rather simpler
by assuming first that f ∈ C 0 (R) is continuous and vanishes outside some bounded
interval, f (x) = 0 in |x| > R. Then we can write out the L2 inner product as a
doulbe integral, which is a genuine (iterated) Riemann integral:
Z Z
(4.109) (f, Aw f ) = A(w, x, y)f (x)f (y)dydx.

Here I have used the fact that f and A are real-valued.


Look at the formula for A in (4.101). The first thing to notice is the factor
(1 − w2 )− 21 which blows up as w → 1. On the other hand, the argument of the
exponential has two terms, the first tends to 0 as w → 1 and the second blows up,
at least when x − y 6= 0. Given the signs, we see that
if  > 0, X = {(x, y); |x| ≤ R, |y| ≤ R, |x − y| ≥ } then
(4.110) sup |A(w, x, y)| → 0 as w → 1.
X

So, the part of the integral in (4.109) over |x − y| ≥  tends to zero as w → 1.


So, look at the other part, where |x − y| ≤ . By the (uniform) continuity of f,
given δ > 0 there exits  > 0 such that
(4.111) |x − y| ≤  =⇒ |f (x) − f (y)| ≤ δ.
Now we can divide (4.109) up into three pieces:-
Z
(4.112) (f, Aw f ) = A(w, x, y)f (x)f (y)dydx
S∩{|x−y|≥}
Z
+ A(w, x, y)(f (x) − f (y))f (y)dydx
S∩{|x−y|≤}
Z
+ A(w, x, y)f (y)2 dydx
S∩{|x−y|≤}
2
where S = [−R, R] .
Look now at the third integral in (4.112)q since it is the important one. We can
1+w
change variable of integration from x to t = 1−w (x − y) and then this becomes
r
1−w
Z
A(w, y + t , y)f (y)2 dydt,
S∩{|x−y|≤} 1+w
r
(4.113) A(w, y+t 1 − w , y)
1+w

   2
1 1−w 2 t
=√ exp − (2y + t 1 − w) exp − .
π(1 + w) 4(1 + w) 4
142 4. APPLICATIONS

Here y is bounded; the first exponential factor tends to 1 so it is straightforward to


show that for any  > 0 the third term in (4.112) tends to

Z
2
(4.114) kf k2L2 as w → 1 since e−t /4 = 2 π.

Noting that A ≥ 0 the same sort of argument shows that the second term is
bounded by a constant multiple of δ. So this proves (4.108) (first choose δ then )
and hence (4.106) under the assumption that f is continuous and vanishes outside
some interval [−R, R].
However, the general case follows by continuity since such continuous functions
vanishing outside compact sets are dense in L2 (R) and both sides of (4.106) are
continuous in f ∈ L2 (R). 
Now, (4.108) certainly implies that the ej form an orthonormal basis, which is
what we wanted to show – but hard work! I did it really to remind you of how we
did the Fourier series computation of the same sort and to suggest that you might
like to compare the two arguments.

5. Fourier transform
Last time I showed the completeness of the orthonormal sequence formed by
the eigenfunctions of the harmonic oscillator. This allows us to prove some basic
facts about the Fourier transform, which we already know is a linear operator
Z
(4.115) L1 (R) −→ C∞0
(R), û(ξ) = eixξ u(x)dx.

Namely we have already shown the effect of the Fourier transform on the ‘ground
state’:

(4.116) F(u0 )(ξ) = 2πe0 (ξ).
By a similar argument we can check that

(4.117) F(uj )(ξ) = 2πij uj (ξ) ∀ j ∈ N.
As usual we can proceed by induction using the fact that uj = Cuj−1 . The integrals
involved here are very rapidly convergent at infinity, so there is no problem with
the integration by parts in
(4.118)
Z T
d duj−1
F( uj−1 ) = lim e−ixξ dx
dx T →∞ −T dx
Z T !
−ixξ
 −ixξ T
= lim (iξ)e uj−1 dx + e uj−1 (x) −T = (iξ)F(uj−1 ),
T →∞ −T

de−ixξ
Z
d
F(xuj−1 ) = i uj−1 dx = i F(uj−1 ).
dξ dξ
Taken together these identities imply the validity of the inductive step:
d d √
(4.119) F(uj ) = F((− + x)uj−1 ) = (i(− + ξ)F(uj−1 ) = iC( 2πij−1 uj−1 )
dx dξ
so proving (4.117).
So, we have found an orthonormal basis for L2 (R) with elements which are all
in L1 (R) and which are also eigenfunctions for F.
5. FOURIER TRANSFORM 143

Theorem 18. The Fourier transform maps L1 (R) ∩ L2 (R) into L2 (R) and
extends by continuity to an isomorphism of L2 (R) such that √12π F is unitary with
the inverse of F the continuous extension from L1 (R) ∩ L2 (R) of
Z
1
(4.120) F(f )(x) = eixξ f (ξ).

Proof. This really is what we have already proved. The elements of the
Hermite basis ej are all in both L1 (R) and L2 (R) so if u ∈ L1 (R) ∩ L2 (R) its image
under F because we can compute the L2 inner products and see that
Z Z √
(4.121) (F(u), ej ) = ej (ξ)e u(x)dxdξ = F(ej )(x)u(x) = 2πij (u, ej ).
ixξ
R2
Now Bessel’s inequality shows that F(u) ∈ L2 (R) (it is of course locally integrable)
since it is continuous).
Everything else now follows easily. 
Notic in particular that we have also proved Parseval’s and Plancherel’s identities:-

(4.122) kF(u)kL2 = 2πkukL2 , (F(u), F(v)) = 2π(u, v), ∀ u, v ∈ L2 (R).
Since we are at the end of the course, more or less, I do not have any time to
remind you of the many applications of the Fourier transform. However, let me
just indicate the definitions of Sobolev spaces and Schwartz space and how they
are related to the Fourier transform.
First Sobolev spaces. We now see that F maps L2 (R) isomorphically onto
2
L (R) and we can see from (4.118) for instance that it ‘turns differentiations by
x into multiplication by ξ’. Of course we do not know how to differentiate L2
functions so we have some problems making sense of this. One way, the usual
mathematicians trick, is to turn what we want into a definition.
Definition 21. The the Sobolev spaces of order s, for any s ∈ (0, ∞), are
defined as subspaces of L2 (R) :
s
(4.123) H s (R) = {u ∈ L2 (R); (1 + |ξ|2 ) 2 û ∈ L2 (R)}.
It is natural to identify H 0 (R) = L2 (R).
Exercise 1. Show that the Sobolev spaces of positive order are Hilbert spaces
with the norm
s
(4.124) kuks = k(1 + |ξ|2 ) 2 ûkL2 .
Having defined these spaces, which get smaller as s increases it can be shown
for instances that if n ≥ s is an integer then the set of n times continuously
differentiabel functions on R which vanish outside a compact set are dense in H s .
This allows us to justify, by continuity, the following statement:-
Proposition 48. The bounded linear map
d du
(4.125) : H s (R) −→ H s−1 (R), s ≥ 1, v(x) = ⇐⇒ v̂(ξ) = iξ û(ξ)
dx dx
is consistent with differentiation on n times continuously differentiable functions of
compact support, for any integer n ≥ s.
One of the more important results about Sobolev spaces – of which there are
many – is the relationship between these ‘L2 derivatives’ and ‘true derivatives’.
144 4. APPLICATIONS

1
Theorem 19 (Sobolev embedding). If n is an integer and s > n + 2 then
s n
(4.126) H (R) ⊂ C∞ (R)
consists of n times continuosly differentiable functions with bounded derivatives to
order n.
This is actually not so hard to prove.
These are not the only sort of spaces with ‘more regularity’ one can define
and use. For instance one can try to treat x and ξ more symmetrically and define
smaller spaces than the H s above by setting
s s
s
(4.127) Hiso (R) = {u ∈ L2 (R); (1 + |ξ|2 ) 2 û ∈ L2 (R), (1 + |x|2 ) 2 u ∈ L2 (R)}.
Exercise 2. Find a norm with respect to which these (‘isotropic Sobolev
s
spaces’) Hiso (R) are Hilbert spaces.
Exercise 3. (Probably too hard) Show that the harmonic oscillator extends
by continuity to an isomorphism
s+2 s
(4.128) H : Hiso (R) −→ Hiso (R) ∀ s ≥ 2.
Finally in this general vein, I wanted to point out that Hilbert, and even Ba-
nach, spaces are not the end of the road! One very important space in relation to
a direct treatment of the Fourier transform, is the Schwartz space. The definition
is reasonably simple. Namely we denote Schwartz space by S(R) and say
u ∈ S(R) ⇐⇒ u : R −→ C
is continuously differentiable of all orders and for every n and ,
(4.129) X dp u
kukn = sup(1 + |x|)k | p | < ∞.
x∈R dx
k+p≤n

All these inequalities just mean that all the derivatives of u are ‘rapidly decreasing
at ∞’ in the sense that they stay bounded when multiplied by any polynomial.
So in fact we know already that S(R) is not empty since the elements of the
Hermite basis, ej ∈ S(R) for all j.
As you can see from the definition in (4.129) this is not likely to be a Banach
space. Each of the k · kn is a norm. However, S(R) is pretty clearly not going to be
complete with respect to any one of these. However it is complete with respect to
all, countably many, norms. What does this mean? In fact S(R) is a metric space
with the metric
X ku − vkn
(4.130) d(u, v) = 2−n
n
1 + ku − vkn
as you can check, with some thought perhaps required. So the claim is that S(R)
is comlete as a metric space – such a thing is called a Fréchet space.
What has this got to do with the Fourier transform? The point is that
(4.131)
du dF(u)
F : S(R) −→ S(R) is an isomorphism and F( ) = iξF(u), F(xu) = −i
dx dξ
where this now makes sense. The Sobolev embedding theorem implies that
Z
s
(4.132) S(R) = Hiso (R).
n
6. DIRICHLET PROBLEM 145

The dual space of S(R) is the space, denoted S 0 (R), of tempered distributions on
R.

6. Dirichlet problem
As a final application, which I do not have time to do in full detail in lectures,
I want to consider the Dirichlet problem again, but now in higher dimensions. Of
course this is a small issue, since I have not really gone through the treatment of
the Lebesgue integral etc in higher dimensions – still I hope it is clear that with a
little more application we could do it and for the moment I will just pretend that
we have.
So, what is the issue? Consider Laplace’s equation on an open set in Rn . That
is, we want to find a solution of
∂ 2 u(x) ∂ 2 u(x) ∂ 2 u(x)
(4.133) −( + + · · · + ) = f (x) in Ω ⊂ Rn .
∂x21 ∂x22 ∂x2n
Now, maybe some of you have not had a rigorous treatment of partical deriva-
tives either. Just add that to the heap of unresolved issues. In any case, partial
derivatives are just one-dimensional derivatives in the variable concerned with the
other variables held fixed. So, we are looking for a function u which has all partial
derivatives up to order 2 existing everywhere and continous. So, f will have to be
continuous too. Unfortunately this is not enough to guarantee the existence of a
twice continuously differentiable solution – later we will just suppose that f itself
is once continuously differentiable.
Now, we want a solution of (4.133) which satisfies the Dirichlet condition. For
this we need to have a reasonable domain, which has a decent boundary. To short
cut the work involved, let’s just suppose that 0 ∈ Ω and that it is given by an
inequality of the sort
(4.134) Ω = {z ∈ Rn ; |z| < ρ(z/|z|)
where ρ is another once continuously differentiable, and strictly positive, function
on Rn (although we only care about its values on the unit vectors). So, this is no
worse than what we are already dealing with.
Now, the Dirichlet condition can be stated as
u ∈ C 0 (Ω), u z| = ρ(z/|z|) = 0.

(4.135)
Here we need the first condition to make much sense of the second.
So, what I want to approach is the following result – which can be improved a
lot and which I will not quite manage to prove anyway.
Theorem 20. If 0 < ρ ∈ C 1 (Rn ), and f ∈ C 1 (Rn ) then there exists a unique
u ∈ C 2 (Ω) ∩ C 0 (Ω) satisfying (4.133) and (4.135).
CHAPTER

Appendix: Exam Preparation Problems

EP.1 Let H be a Hilbert space with inner product (·, ·) and suppose that
(.1) B : H × H ←→ C
is a(nother) sesquilinear form – so for all c1 , c2 ∈ C, u, u1 , u2 and v ∈ H,
(.2) B(c1 u1 + c2 u2 , v) = c1 B(u1 , v) + c2 B(u2 , v), B(u, v) = B(v, u).
Show that B is continuous, with respect to the norm k(u, v)k = kukH + kvkH on
H × H if and only if it is bounded, in the sense that for some C > 0,
(.3) |B(u, v)| ≤ CkukH kvkH .
EP.2 A continuous linear map T : H1 −→ H2 between two, possibly different,
Hilbert spaces is said to be compact if the image of the unit ball in H1 under T is
precompact in H2 . Suppose A : H1 −→ H2 is a continuous linear operator which
is injective and surjective and T : H1 −→ H2 is compact. Show that there is a
compact operator K : H2 −→ H2 such that T = KA.
EP.3 Suppose P ⊂ H is a (non-trivial, i.e. not {0}) closed linear subspace of
a Hilbert space. Deduce from a result done in class that each u in H has a unique
decomposition
(.4) u = v + v 0 , v ∈ P, v 0 ⊥ P
and that the map πP : H 3 u 7−→ v ∈ P has the properties
(.5) (πP )∗ = πP , (πP )2 = πP , kπP kB(H) = 1.
EP.4 Show that for a sequence of non-negative
PR step functionsPfj , defined on
R, which is absolutely summable, meaning fj < ∞, the series fj (x) cannot
j j
diverge for all x ∈ (a, b), for any a < b.
EP.5 Let Aj ⊂ [−N, N ] ⊂ R (for N fixed) be a sequence of subsets with the
property that the characteristic function, χj of Aj , is integrable for each j. Show
that the characteristic function of
[
(.6) A= Aj
j

is integrable.
2
EP.6 Let ej = cj C j e−x /2 , cj > 0, C = − dx d
+ x the creation operator, be
2
the orthonormal basis of L (R) consisting of the eigenfunctions of the harmonic
oscillator discussed in class. Define an operator on L2 (R) by

1
X
(.7) Au = (2j + 1)− 2 (u, ej )L2 ej .
j=0

(1) Show that A is compact as an operator on L2 (R).


147
148 APPENDIX: EXAM PREPARATION PROBLEMS

0
(2) Suppose that V ∈ C∞ (R) is a bounded, real-valued, continuous function
on R. What can you say about the eigenvalues τj , and eigenfunctions vj ,
of K = AV A, where V is acting by multiplication on L2 (R)?
(3) Show that for C > 0 a large enough constant, Id +A(V + C)A is invertible
(with bounded inverse on L2 (R)).
(4) Show that L2 (R) has an orthonormal basis of eigenfunctions of J =
A(Id +A(V + C)A)−1 A.
(5) What would you need to show to conclude that these eigenfunctions of J
satisfy
d2 vj (x)
(.8) − + x2 vj (x) + V (x)vj (x) = λj vj ?
dx2
(6) What would you need to show to check that all the square-integrable,
twice continuously differentiable, solutions of (.8), for some λj ∈ C, are
eigenfunctions of K?
EP.7 Test 1 from last year (N.B. There may be some confusion between L1
and L1 here, just choose the correct interpretation!):-
Q1. Recall Lebesgue’s Dominated Convergence Theorem and use it to show
that if u ∈ L2 (R) and v ∈ L1 (R) then
Z Z
2
lim |u| = 0, lim |CN u − u|2 = 0,
N →∞ |x|>N N →∞
(Eq1) Z Z
lim |v| = 0 and lim |CN v − v| = 0.
N →∞ |x|>N N →∞

where

N
 if f (x) > N
(Eq2) CN f (x) = −N if f (x) < −N

f (x) otherwise.

Q2. Show that step functions are dense in L1 (R) and in L2 (R) (Hint:- Look at
Q1 above and think about f −fN , fN = CN f χ[−N,N ] and its square. So it
suffices to show that fN is the limit in L2 of a sequence of step functions.
Show that if gn is a sequence of step functions converging to fN in L1
then CN gn χ[−N,N ] is converges to fN in L2 .) and that if f ∈ L1 (R) then
there is a sequence of step functions un and an element g ∈ L1 (R) such
that un → f a.e. and |un | ≤ g.
Q3. Show that L1 (R) and L2 (R) are separable, meaning that each has a count-
able dense subset.
Q4. Show that the minimum and the maximum of two locally integrable func-
tions is locally integrable.
Q5. A subset of R is said to be (Lebesgue) measurable if its characteristic
function is locally integrable. Show that a countable union of measurable
sets is measurable. Hint: Start with two!
Q6. Define L∞ (R) as consisting of the locally integrable functions which are
bounded, supR |u| < ∞. If N∞ ⊂ L∞ (R) consists of the bounded functions
which vanish outside a set of measure zero show that
(Eq3) ku + N∞ kL∞ = inf sup |u(x) + h(x)|
h∈N∞ x∈R
APPENDIX: EXAM PREPARATION PROBLEMS 149

is a norm on L∞ (R) = L∞ (R)/N∞ .


Q7. Show that if u ∈ L∞ (R) and v ∈ L1 (R) then uv ∈ L1 (R) and that
Z
(Eq4) | uv| ≤ kukL∞ kvkL1 .

Q8. Show that each u ∈ L2 (R) is continuous in the mean in the sense that
Tz u(x) = u(x − z) ∈ L2 (R) for all z ∈ R and that
Z
(Eq5) lim |Tz u − u|2 = 0.
|z|→0

Q9. If {uj } is a Cauchy sequence in L2 (R) show that both (Eq5) and (Eq1)
are uniform in j, so given  > 0 there exists δ > 0 such that
Z Z
2
(Eq6) |Tz uj − uj | < , |uj |2 <  ∀ |z| < δ and all j.
|x|>1/δ

Q10. Construct a sequence in L2 (R) for which the uniformity in (Eq6) does not
hold.
EP.8 Test 2 from last year.
(1) Recall the discussion of the Dirichlet problem for d2 /dx2 from class and
carry out an analogous discussion for the Neumann problem to arrive at
a complete orthonormal basis of L2 ([0, 1]) consisting of ψn ∈ C 2 functions
which are all eigenfunctions in the sense that
d2 ψn (x) dψn dψn
(NeuEig) 2
= γn ψn (x) ∀ x ∈ [0, 1], (0) = (1) = 0.
dx dx dx
This is actually a little harder than the Dirichlet problem which I did in
class, because there is an eigenfunction of norm 1 with γ = 0. Here are
some individual steps which may help you along the way!
What is the eigenfunction with eigenvalue 0 for (NeuEig)?
What is the operator of orthogonal projection onto this function?
What is the operator of orthogonal projection onto the orthocom-
plement of this function?
The crucual part. Find an integral operator AN = B − BN , where
B is the operator from class,
Z x
(B-Def) (Bf )(x) = (x − s)f (s)ds
0

and BN is of finite rank, such that if f is R 1continuous then u = AN f is


twice continuously differentiable, satisfies 0 u(x)dx = 0, AN 1 = 0 (where
1 is the constant function) and
Z 1
f (x)dx = 0 =⇒
(GI) 0
d2 u du du
= f (x) ∀ x ∈ [0, 1], (0) = (1) = 0.
dx2 dx dx
Show that AN is compact and self-adjoint.
Work out what the spectrum of AN is, including its null space.
Deduce the desired conclusion.
150 APPENDIX: EXAM PREPARATION PROBLEMS

(2) Show that these two orthonormal bases of L2 ([0, 1]) (the one above and the
one from class) can each be turned into an orthonormal basis of L2 ([0, π])
by change of variable.
(3) Construct an orthonormal basis of L2 ([−π, π]) by dividing each element
into its odd and even parts, resticting these to [0, π] and using the Neu-
mann basis above on the even part and the Dirichlet basis from class on
the odd part.
(4) Prove the basic theorem of Fourier series, namely that for any function
u ∈ L2 ([−π, π]) there exist unique constants ck ∈ C, k ∈ Z such that
X
(FS) u(x) = ck eikx converges in L2 ([−π, π])
k∈Z
and give an integral formula for the constants.
EP.9 Let B ∈ C([0, 1]2 ) be a continuous function of two variables.
Explain why the integral operator
Z
T u(x) = B(x, y)u(y)
[0,1]

defines a bounded linear map L1 ([0, 1]) −→ C([0, 1]) and hence a bounded
operator on L2 ([0, 1]).
(a) Explain why T is not surjective as a bounded operator on L2 ([0, 1]).
(b) Explain why Id −T has finite dimensional null space N ⊂ L2 ([0, 1])
as an operator on L2 ([0, 1])
(c) Show that N ⊂ C([0, 1]).
(d) Show that Id −T has closed range R ⊂ L2 ([0, 1])) as a bounded op-
erator on L2 ([0, 1]).
(e) Show that the orthocomplement of R is a subspace of C([0, 1]).
EP.10 Let c : N2 −→ C be an ‘infinite matrix’ of complex numbers
satisfying

X
(.9) |cij |2 < ∞.
i,j=1

If {ei }∞
i=1 is an orthornomal basis of a (separable of course) Hilbert space
H, show that
X∞
(.10) Au = cij (u, ej )ei
i,j=1

defines a compact operator on H.


CHAPTER

Solutions to problems

Solution .1 (Problem 1.1). Write out a proof (you can steal it from one of
many places but at least write it out in your own hand) either for p = 2 or for each
p with 1 ≤ p < ∞ that
X∞
lp = {a : N −→ C; |aj |p < ∞, aj = a(j)}
j=1

is a normed space with the norm


  p1
X∞
kakp =  |aj |p  .
j=1

This means writing out the proof that this is a linear space and that the three
conditions required of a norm hold.
Solution:- We know that the functions from any set with values in a linear space
form a linear space – under addition of values (don’t feel bad if you wrote this out,
it is a good thing to do once). So, to see that lp is a linear space it suffices to see
that it is closed under addition and scalar multiplication. For scalar multiples this
is clear:-
(.1) |tai | = |t||ai | so ktakp = |t|kakp
which is part of what is needed for the proof that k · kp is a norm anyway. The fact
that a, b ∈ lp imples a + b ∈ lp follows once we show the triangle inequality or we
can be a little cruder and observe that
|ai + bi |p ≤ (2 max(|a|i , |bi |))p = 2p max(|a|pi , |bi |p ) ≤ 2p (|ai | + |bi |)
(.2)
X
ka + bkp = p |a + b |p ≤ 2p (kakp + kbkp ),
i i
j

where we use the fact that tp is an increasing function of t ≥ 0.


Now, to see that lp is a normed space we need to check that kakp is indeed a
norm. It is non-negative and kakp = 0 implies ai = 0 for all i which is to say a = 0.
So, only the triangle inequality remains. For p = 1 this is a direct consequence of
the usual triangle inequality:
X X
(.3) ka + bk1 = |ai + bi | ≤ (|ai | + |bi |) = kak1 + kbk1 .
i i
For 1 < p < ∞ it is known as Minkowski’s inequality. This in turn is deduced
from Hölder’s inequality – which follows from Young’s inequality! The latter says
if 1/p + 1/q = 1, so q = p/(p − 1), then
αp βq
(.4) αβ ≤ + ∀ α, β ≥ 0.
p q
151
152 SOLUTIONS TO PROBLEMS

To check it, observe that as a function of α = x,


xp βq
(.5) f (x) = − xβ +
p q
if non-negative at x = 0 and clearly positive when x >> 0, since xp grows faster
than xβ. Moreover, it is differentiable and the derivative only vanishes at xp−1 =
β, where it must have a global minimum in x > 0. At this point f (x) = 0 so
Young’s inequality follows. Now, applying this with α = |ai |/kakp and β = |bi |/kbkq
(assuming both are non-zero) and summing over i gives Hölder’s inequality
X X X  |ai |p |bi |q

| ai bi |/kakp kbkq ≤ |ai ||bi |/kakp kbkq ≤ + =1
i i i
kakpp p kbkqq q
(.6) X
=⇒ | ai bi | ≤ kakp kbkq .
i

Of course, if either kakp = 0 or kbkq = 0 this inequality holds anyway.

Now, from this Minkowski’s inequality follows. Namely from the ordinary tri-
angle inequality and then Minkowski’s inequality (with q power in the first factor)
X X
(.7) |ai + bi |p = |ai + bi |(p−1) |ai + bi |
i i
X X
≤ |ai + bi |(p−1) |ai | + |ai + bi |(p−1) |bi |
i i
!1/q
X
p
≤ |ai + bi | (kakp + kbkq )
i

gives after division by the first factor on the right

(.8) ka + bkp ≤ kakp + kbkp .

Thus, lp is indeed a normed space.


I did not necessarily expect you to go through the proof of Young-Hölder-
Minkowksi, but I think you should do so at some point since I will not do it in
class.

Solution .2. The ‘tricky’ part in Problem 1.1 is the triangle inequality. Sup-
pose you knew – meaning I tell you – that for each N
  p1
N
X
 |aj |p  is a norm on CN
j=1

would that help?

Solution. Yes indeed it helps. If we know that for each N


  p1   p1   p1
N
X N
X XN
(.9)  |aj + bj |p  ≤  |aj |p  +  |bj |p 
j=1 j=1 j=1
SOLUTIONS TO PROBLEMS 153

then for elements of lp the norms always bounds the right side from above, meaning
  p1
XN
(.10)  |aj + bj |p  ≤ kakp + kbkp .
j=1

Since the left side is increasing with N it must converge and be bounded by the
right, which is independent of N. That is, the triangle inequality follows. Really
this just means it is enough to go through the discussion in the first problem for
finite, but arbitrary, N. 

Solution .3. Prove directly that each lp as defined in Problem 1.1 – or just
2
l – is complete, i.e. it is a Banach space. At the risk of offending some, let me
say that this means showing that each Cauchy sequence converges. The problem
here is to find the limit of a given Cauchy sequence. Show that for each N the
sequence in CN obtained by truncating each of the elements at point N is Cauchy
with respect to the norm in Problem 1.2 on CN . Show that this is the same as being
Cauchy in CN in the usual sense (if you are doing p = 2 it is already the usual
sense) and hence, this cut-off sequence converges. Use this to find a putative limit
of the Cauchy sequence and then check that it works.
Solution. So, suppose we are given a Cauchy sequence a(n) in lp . Thus, each
(n)
element is a sequence {aj }∞ p
j=1 in l . From the continuity of the norm in Problem
(n)
1.5 below, ka k must be Cauchy in R and so converges. In particular the sequence
is norm bounded, there exists A such that ka(n) kp ≤ A for all n. The Cauchy
condition itself is that given  > 0 there exists M such that for all m, n > M,
! p1
X (n) (m)
(.11) ka(n) − a(m) kp = |ai − ai |p < /2.
i
(n) (m) (n)
Now for each i, |ai − ai | ≤ ka(n) − a(m) kp so each of the sequences ai must
be Cauchy in C. Since C is complete
(n)
(.12) lim a = ai exists for each i = 1, 2, . . . .
n→∞ i

So, our putative limit is a, the sequence {ai }∞


i=1 . The boundedness of the norms
shows that
N
(n)
X
(.13) |ai |p ≤ Ap
i=1

and we can pass to the limit here as n → ∞ since there are only finitely many
terms. Thus
N
X
(.14) |ai |p ≤ Ap ∀ N =⇒ kakp ≤ A.
i=1
p
Thus, a ∈ l as we hoped. Similarly, we can pass to the limit as m → ∞ in the
finite inequality which follows from the Cauchy conditions
N
X (n) (m) 1
(.15) ( |ai − ai |p ) p < /2
i=1
154 SOLUTIONS TO PROBLEMS

to see that for each N


N
X (n) 1
(.16) ( |ai − ai |p ) p ≤ /2
i=1
and hence
(.17) ka(n) − ak <  ∀ n > M.
Thus indeed, a(n) → a in lp as we were trying to show.
Notice that the trick is to ‘back off’ to finite sums to avoid any issues of inter-
changing limits. 
Solution .4. Consider the ‘unit sphere’ in lp – where if you want you can set
p = 2. This is the set of vectors of length 1 :
S = {a ∈ lp ; kakp = 1}.
(1) Show that S is closed.
(2) Recall the sequential (so not the open covering definition) characterization
of compactness of a set in a metric space (e .g . by checking in Rudin).
(3) Show that S is not compact by considering the sequence in lp with kth
element the sequence which is all zeros except for a 1 in the kth slot. Note
that the main problem is not to get yourself confused about sequences of
sequences!
Solution. By the next problem, the norm is continuous as a function, so
(.18) S = {a; kak = 1}
is the inverse image of the closed subset {1}, hence closed.
Now, the standard result on metric spaces is that a subset is compact if and
only if every sequence with values in the subset has a convergent subsequence with
limit in the subset (if you drop the last condition then the closure is compact).
In this case we consider the sequence (of sequences)
(
(n) 0 i 6= n
(.19) ai = .
1 i=n
1
This has the property that ka(n) − a(m) kp = 2 p whenever n 6= m. Thus, it cannot
have any Cauchy subsequence, and hence cannot have a convergent subsequence,
so S is not compact.
This is important. In fact it is a major difference between finite-dimensional
and infinite-dimensional normed spaces. In the latter case the unit sphere cannot
be compact whereas in the former it is. 
Solution .5. Show that the norm on any normed space is continuous. Solution:-
Right, so I should have put this problem earlier!
The triangle inequality shows that for any u, v in a normed space
(.20) kuk ≤ ku − vk + kvk, kvk ≤ ku − vk + kuk
which implies that
(.21) |kuk − kvk| ≤ ku − vk.
This shows that k · k is continuous, indeed it is Lipschitz continuous.
SOLUTIONS TO PROBLEMS 155

Solution .6. Finish the proof of the completeness of the space B constructed
in lecture on February 10. The description of that construction can be found in the
notes to Lecture 3 as well as an indication of one way to proceed.

Solution. The proof could be shorter than this, I have tried to be fairly
complete.
To recap. We start of with a normed space V. From this normed space we
construct the new linear space Ṽ with points the absolutely summable series in V.
Then we consider the subspace S ⊂ Ṽ of those absolutely summable series which
converge to 0 in V. We are interested in the quotient space
(.22) B = Ṽ /S.
What we know already is that this is a normed space where the norm of b = {vn }+S
– where {vn } is an absolutely summable series in V is
N
X
(.23) kbkB = lim k vn kV .
N →∞
n=1

This is independent of which series is used to represent b – i.e. is the same if an


element of S is added to the series.
Now, what is an absolutely summable series in B? It is a sequence {bn }, thought
of a series, with the property that
X
(.24) kbn kB < ∞.
n

We have to show that it converges in B. The first task is to guess what the limit
should be. The idea is that it should be a series which adds up to ‘the sum of the
(n)
bn ’s’. Each bn is represented by an absolutely summable series vk in V. So, we
can just look for the usual diagonal sum of the double series and set
X (n)
(.25) wj = vk .
n+k=j

The problem is that this will not in generall be absolutely summable as a series in
V. What we want is the estimate
X X X (n)
(.26) kwj k = k vk k < ∞.
j j j=n+k

The only way we can really estimate this is to use the triangle inequality and
conclude that

(n)
X X
(.27) kwj k ≤ kvk kV .
j=1 k,n

Each of the sums over k on the right is finite, but we do not know that the sum
over k is then finite. This is where the first suggestion comes in:-
(n)
We can choose the absolutely summable series vk representing bn such that
X (n)
(.28) kvk k ≤ kbn kB + 2−n .
k
156 SOLUTIONS TO PROBLEMS

Suppose an initial choice of absolutely summable series representing bn is uk , so


PN P
kbn k = limN →∞ k uk k and kuk kV < ∞. Choosing M large it follows that
k=1 k
X
(.29) kuk kV ≤ 2−n−1 .
k>M

M
(n) P (n)
With this choice of M set v1 = uk and vk = uM +k−1 for all k ≥ 2. This does
k=1
still represent bn since the difference of the sums,
N
X (n)
N
X X−1
N +M
(.30) vk − uk = − uk
k=1 k=1 k=N

for all N. The sum on the right tends to 0 in V (since it is a fixed number of terms).
Moreover, because of (.29),
(.31)
X (n) XM X XN X N
X
kvk kV = k uj kV + kuk k ≤ k uj k + 2 kuk k ≤ k uj k + 2−n
k j=1 k>M j=1 k>M j=1

for all N. Passing to the limit as N → ∞ gives (.28).


Once we have chosen these ‘nice’ representatives of each of the bn ’s if we define
the wj ’s by (.25) then (.26) means that
X X X
(.32) kwj kV ≤ kbn kB + 2−n < ∞
j n n

because the series bn is absolutely summable. Thus {wj } defines an element of Ṽ


and hence b ∈ B. P
Finally then we want to show that bn = b in B. This just means that we
n
need to show
N
X
(.33) lim kb − bn kB = 0.
N →∞
n=1

N
P
The norm here is itself a limit – b − bn is represented by the summable series
n=1
with nth term
N
(n)
X
(.34) wk − vk
n=1

and the norm is then


p N
(n)
X X
(.35) lim k (wk − vk )kV .
p→∞
k=1 n=1

Then we need to understand what happens as N → ∞! Now, wk is the diagonal


(n) (n)
sum of the vj ’s so sum over k gives the difference of the sum of the vj over the
first p anti-diagonals minus the sum over a square with height N (in n) and width
SOLUTIONS TO PROBLEMS 157

p. So, using the triangle inequality the norm of the difference can be estimated by
the sum of the norms of all the ‘missing terms’ and then some so
p N
(n) (m)
X X X
(.36) k (wk − vk )kV ≤ kvl kV
k=1 n=1 l+m≥L

where L = min(p, N ). This sum is finite and letting p → ∞ is replaced by the sum
over l + m ≥ N. Then letting N → ∞ it tends to zero by the absolute (double)
summability. Thus
N
X
(.37) lim kb − bn kB = 0
N →∞
n=1
P
which is the statelent we wanted, that bn = b. 
n

Problem .1. Let’s consider an example of an absolutely summable sequence


of step functions. For the interval [0, 1) (remember there is a strong preference
for left-closed but right-open intervals for the moment) consider a variant of the
construction of the standard Cantor subset based on 3 proceeding in steps. Thus,
remove the ‘central interval [1/3, 2/3). This leave C1 = [0, 1/3) ∪ [2/3, 1). Then
remove the central interval from each of the remaining two intervals to get C2 =
[0, 1/9) ∪ [2/9, 1/3) ∪ [2/3, 7/9) ∪ [8/9, 1). Carry on in this way to define successive
sets Ck ⊂ Ck−1 , each consisting of a finite union of semi-open intervals. Now,
consider the series of step functions fk where fk (x) = 1 on Ck and 0 otherwise.
(1) Check that this is an absolutely
P summable series.
(2) For which x ∈ [0, 1) does |fk (x)| converge?
k
(3) Describe a function on [0, 1) which is shown to be Lebesgue integrable
(as defined in Lecture 4) by the existence of this series and compute its
Lebesgue integral.
(4) Is this function Riemann integrable (this is easy, not hard, if you check
the definition of Riemann integrability)?
(5) Finally consider the function g which is equal to one on the union of all
the subintervals of [0, 1) which are removed in the construction and zero
elsewhere. Show that g is Lebesgue integrable and compute its integral.
Solution. (1) The total length of the intervals is being reduced by a
k
factor of 1/3 each time. Thus l(Ck ) = 32k . Thus the integral of f, which
is non-negative, is actually

2k 2k
Z XZ X
(.38) fk = k =⇒ |fk | = =2
3 3k
k k=1

Thus the series is absolutely summable.


(2) Since the Ck are decreasing, Ck ⊃ Ck+1 , only if
\
(.39) x∈E= Ck
k
P
does the series |fk (x)| diverge (to +∞) otherwise it converges.
k
158 SOLUTIONS TO PROBLEMS

(3) The function defined as the sum of the series where it converges and zero
otherwise
P
 fk (x) x ∈ R \ E
(.40) f (x) = k
0 x∈E
is integrable by definition. Its integral is by definition
Z XZ
(.41) f= fk = 2
k

from the discussion above.


(4) The function f is not Riemann integrable since it is not bounded – and
this is part of the definition. In particular for x ∈ Ck \ Ck+1 , which is not
an empty set, f (x) = k.
(5) The set F, which is the union of the intervals removed is [0, 1) \ E. Taking
step functions equal to 1 on each of the intervals removed gives an abso-
lutely summable series, since they are non-negative and the kth one has
integral 1/3 × (2/3)k−1 for k = 1, . . . . This series converges to g on F so
g is Lebesgue integrable and hence
Z
(.42) g = 1.

Problem .2. The covering lemma for R2 . By a rectangle we will mean a set
of the form [a1 , b1 ) × [a2 , b2 ) in R2 . The area of a rectangle is (b1 − a1 ) × (b2 − a2 ).
(1) We may subdivide a rectangle by subdividing either of the intervals –
replacing [a1 , b1 ) by [a1 , c1 ) ∪ [c1 , b1 ). Show that the sum of the areas of
rectangles made by any repeated subdivision is always the same as that
of the original.
(2) Suppose that a finite collection of disjoint rectangles has union a rectangle
(always in this same half-open sense). Show, and I really mean prove, that
the sum of the areas is the area of the whole rectange. Hint:- proceed by
subdivision.
(3) Now show that for any countable collection of disjoint rectangles contained
in a given rectange the sum of the areas is less than or equal to that of
the containing rectangle.
(4) Show that if a finite collection of rectangles has union containing a given
rectange then the sum of the areas of the rectangles is at least as large of
that of the rectangle contained in the union.
(5) Prove the extension of the preceeding result to a countable collection of
rectangles with union containing a given rectangle.
Solution. (1) For the subdivision of one rectangle this is clear enough.
Namely we either divide the first side in two or the second side in two at
an intermediate point c. After subdivision the area of the two rectanges
is either
(c − a1 )(b2 − a2 ) + (b1 − c)(b2 − a2 ) = (b1 − c1 )(b2 − a2 ) or
(.43)
(b1 − a1 )(c − a2 ) + (b1 − a1 )(b2 − c) = (b1 − c1 )(b2 − a2 ).
SOLUTIONS TO PROBLEMS 159

this shows by induction that the sum of the areas of any the rectangles
made by repeated subdivision is always the same as the original.
(2) If a finite collection of disjoint rectangles has union a rectangle, say
[a1 , b2 ) × [a2 , b2 ) then the same is true after any subdivision of any of
the rectangles. Moreover, by the preceeding result, after such subdivi-
sion the sum of the areas is always the same. Look at all the points
C1 ⊂ [a1 , b1 ) which occur as an endpoint of the first interval of one of
the rectangles. Similarly let C2 be the corresponding set of end-points of
the second intervals of the rectangles. Now divide each of the rectangles
repeatedly using the finite number of points in C1 and the finite number
of points in C2 . The total area remains the same and now the rectangles
covering [a1 , b1 ) × [A2 , b2 ) are precisely the Ai × Bj where the Ai are a set
of disjoint intervals covering [a1 , b1 ) and the Bj are a similar set covering
[a2 , b2 ). Applying the one-dimensional result from class we see that the
sum of the areas of the rectangles with first interval Ai is the product

(.44) length of Ai × (b2 − a2 ).

Then we can sum over i and use the same result again to prove what we
want.
(3) For any finite collection of disjoint rectangles contained in [a1 , b1 )×[a2 , b2 )
we can use the same division process to show that we can add more disjoint
rectangles to cover the whole big rectangle. Thus, from the preceeding
result the sum of the areas must be less than or equal to (b1 − a1 )(b2 − a2 ).
For a countable collection of disjoint rectangles the sum of the areas is
therefore bounded above by this constant.
(4) Let the rectangles be Di , i = 1, . . . , N the union of which contains the
rectangle D. Subdivide D1 using all the endpoints of the intervals of D.
Each of the resulting rectangles is either contained in D or is disjoint from
it. Replace D1 by the (one in fact) subrectangle contained in D. Proceed-
ing by induction we can suppose that the first N − k of the rectangles are
disjoint and all contained in D and together all the rectangles cover D.
Now look at the next one, DN −k+1 . Subdivide it using all the endpoints
of the intervals for the earlier rectangles D1 , . . . , Dk and D. After subdi-
vision of DN −k+1 each resulting rectangle is either contained in one of the
Dj , j ≤ N − k or is not contained in D. All these can be discarded and
the result is to decrease k by 1 (maybe increasing N but that is okay). So,
by induction we can decompose and throw away rectangles until what is
left are disjoint and individually contained in D but still cover. The sum
of the areas of the remaining rectangles is precisely the area of D by the
previous result, so the sum of the areas must originally have been at least
this large.
(5) Now, for a countable collection of rectangles covering D = [a1 , b1 )×[a2 , b2 )
we proceed as in the one-dimensional case. First, we can assume that
there is a fixed upper bound C on the lengths of the sides. Make the
kth rectangle a little larger by extending both the upper limits by 2−k δ
where δ > 0. The area increases, but by no more than 2C2−k . After
extension the interiors of the countable collection cover the compact set
[a1 , b1 − δ] × [a2 , b1 − δ]. By compactness, a finite number of these open
160 SOLUTIONS TO PROBLEMS

rectangles cover, and hence there semi-closed version, with the same end-
points, covers [a1 , b1 −δ)×[a2 , b1 −δ). Applying the preceeding finite result
we see that
(.45) Sum of areas + 2Cδ ≥ Area D − 2Cδ.
Since this is true for all δ > 0 the result follows.

I encourage you to go through the discussion of integrals of step functions – now
based on rectangles instead of intervals – and see that everything we have done can
be extended to the case of two dimensions. In fact if you want you can go ahead
and see that everything works in Rn !
Problem 2.4
(1) Show that any continuous function on [0, 1] is the uniform limit on [0, 1)
of a sequence of step functions. Hint:- Reduce to the real case, divide
the interval into 2n equal pieces and define the step functions to take
infimim of the continuous function on the corresponding interval. Then
use uniform convergence.
(2) By using the ‘telescoping trick’ show that any continuous function on [0, 1)
can be written as the sum
X
(.46) fj (x) ∀ x ∈ [0, 1)
i
P
where the fj are step functions and |fj (x)| < ∞ for all x ∈ [0, 1).
j
(3) Conclude that any continuous function on [0, 1], extended to be 0 outside
this interval, is a Lebesgue integrable function on R.
Solution. (1) Since the real and imaginary parts of a continuous func-
tion are continuous, it suffices to consider a real continous function f and
then add afterwards. By the uniform continuity of a continuous function
on a compact set, in this case [0, 1], given n there exists N such that
|x − y| ≤ 2−N =⇒ |f (x) − f (y)| ≤ 2−n . So, if we divide into 2N equal
intervals, where N depends on n and we insist that it be non-decreasing
as a function of n and take the step function fn on each interval which is
equal to min f = inf f on the closure of the interval then
(.47) |f (x) − Fn (x)| ≤ 2−n ∀ x ∈ [0, 1)
since this even works at the endpoints. Thus Fn → f uniformly on [0, 1).
(2) Now just define f1 = F1 and fk = Fk − Fk−1 for all k > 1. It follows that
these are step functions and that
X n
(.48) = fn .
k=1
Moreover, each interval for Fn+1 is a subinterval for Fn . Since f can
varying by no more than 2−n on each of the intervals for Fn it follows
that
(.49) |fn (x)| = |Fn+1 (x) − Fn (x)| ≤ 2−n ∀ n > 1.
Thus |fn | ≤ 2−n and so the series is absolutely summable. Moreover, it
R

actually converges everywhere on [0, 1) and uniformly to f by (.47).


SOLUTIONS TO PROBLEMS 161

(3) Hence f is Lebesgue integrable.


(4) For some reason I did not ask you to check that
Z Z 1
(.50) f= f (x)dx
0
where on the right is the Riemann integral. However this follows from the
fact that
Z Z
(.51) f = lim Fn
n→∞

and the integral of the step function is between the Riemann upper and
lower sums for the corresponding partition of [0, 1].


Solution .7. If f and g ∈ L1 (R) are Lebesgue integrable functions on the line
show that
R
(1) If f (x) ≥ 0 a.e. then f R≥ 0. R
(2) If f (x) ≤ g(x) a.e. then f ≤ g.
(3) IfR f is complex
R valued then its real part, Re f, is Lebesgue integrable and
| Re f | ≤ |f |.
(4) For a general complex-valued Lebesgue integrable function
Z Z
(.52) | f | ≤ |f |.

Hint: You can look up a proof of this


R easilyR enough, but the usual trick
is to choose θ ∈ [0, 2π) so that eiθ f = (eiθ f ) ≥ 0. Then apply the
preceeding estimate to g = eiθ f.
(5) Show that the integral is a continuous linear functional
Z
(.53) : L1 (R) −→ C.

Solution. (1) If f is real and fn is a real-valued absolutely summable


series of step functions converging to f where it is absolutely convergent
(if we only have a complex-valued sequence use part (3)). Then we know
that
(.54) g1 = |f1 |, gj = |fj | − |fj−1 |, f ≥ 1
is an absolutely convergent sequence converging to |f | almost everywhere.
It follows that f+ = 21 (|f |+f ) = f, if f ≥ 0, is the limit almost everywhere
of the series obtained by interlacing 12 gj and 21 fj :
(
1
gk n = 2k − 1
(.55) hn = 2
fk n = 2k.
Thus f+ is Lebesgue integrable. Moreover we know that
 
Z X Z Z Xk k
X
(.56) f+ = lim hk = lim | fj | + fj 
k→∞ k→∞
n≤2k j=1 j=1
R
where each term is a non-negative step function, so f+ ≥ 0.
162 SOLUTIONS TO PROBLEMS

(2) Apply the preceeding result to g − f which is integrable and satisfies


Z Z Z
(.57) g − f = (g − f ) ≥ 0.

(3) Arguing from first principles again, if fn is now complex valued and an
absolutely summable series of step functions converging a .e . to f then
define

Re fk
 n = 3k − 2
(.58) hn = Im fk n = 3k − 1

− Im fk n = 3k.

This series of step functions is absolutely summable and


X X X
(.59) |hn (x)| < ∞ ⇐⇒ |fn (x)| < ∞ =⇒ hn (x) = Re f.
n n n

Thus Re f is integrable. Since ± Re f ≤ |f |


Z Z Z Z
(.60) ± Re f ≤ |f | =⇒ | Re f | ≤ |f |.

(4) For a complex-valued


R f proceed as suggested. Choose z ∈ C with |z| = 1
such that z f ∈ [0, ∞) which is possible by the properties of complex
numbers. Then by the linearity of the integral
(.61)Z Z Z Z Z Z Z Z
z f = (zf ) = Re(zf ) ≤ |z Re f | ≤ |f | =⇒ | f | = z f ≤ |f |.

(where the second equality follows from the fact that the integral is equal
to its real part).
(5) We know that the integral defines a linear map
Z
1
(.62) I : L (R) 3 [f ] 7−→ f ∈ C
R R
since f = g if f = g a.e. are two representatives of the same class in
L1 (R). To say this is continuous is equivalent to it being bounded, which
follows from the preceeding estimate
Z Z
(.63) |I([f ])| = | f | ≤ |f | = k[f ]kL1

(Note that writing [f ] instead of f ∈ L1 (R) is correct but would normally


be considered pedantic – at least after you are used to it!)
(6) I should have asked – and might do on the test: What is the norm of I
as an element of the dual space of L1 (R). It is 1 – better make sure that
you can prove this.

Problem 3.2 If I ⊂ R is an interval, including possibly (−∞, a) or (a, ∞),
we define Lebesgue integrability of a function f : I −→ C to mean the Lebesgue
integrability of
(
˜ ˜ f (x) x ∈ I
(.64) f : R −→ C, f (x) =
0 x ∈ R \ I.
SOLUTIONS TO PROBLEMS 163

The integral of f on I is then defined to be


Z Z
(.65) f = f˜.
I
(1) Show that the space of such integrable functions on I is linear, denote it
L1 (I).
(2) Show that is f is integrable on I then so R is |f |.
(3) Show that if f is integrable on I and I |f | = 0 then f = 0 a.e. in the
sense that f (x) = 0 for all x ∈ I \ E where E ⊂ I is of measure zero as a
subset of R.
(4) Show that the set of null functions as in the preceeding question is a linear
R it N (I).
space, denote
(5) Show that I |f | defines a norm on L1 (I) = L1 (I)/N (I).
(6) Show that if f ∈ L1 (R) then
(
f (x) x ∈ I
(.66) g : I −→ C, g(x) =
0 x∈R\I
is in L1 (R) an hence that f is integrable on I.
(7) Show that the preceeding construction gives a surjective and continuous
linear map ‘restriction to I’
(.67) L1 (R) −→ L1 (I).
(Notice that these are the quotient spaces of integrable functions modulo
equality a.e.)
Solution:
(1) If f and g are both integrable on I then setting h = f + g, h̃ = f˜ + g̃,
directly from the definitions, so f + g is integrable on I if f and g are by
the linearity of L1 (R). Similarly if h = cf then h̃ = cf˜ is integrable for
any constant c if f˜ is integrable. Thus L1 (I) is linear.
(2) Again from the definition, |f˜| = h̃ if h = |f |. Thus f integrable on I
implies f˜ ∈ L1 (R), which, as we know, implies that |f˜| ∈ L1 (R). So in
turn h̃ ∈ L1 (R) where h = |f |, so R|f | ∈ L1 (I).
(3) If f ∈ L1 (I) and I |f | = 0 then R |f˜| = 0 which implies that f˜ = 0 on
R

R \ E where E ⊂ R is of measure zero. Now, EI = E ∩ I ⊂ E is also of


measure zero (as a subset of a set of measure zero) and f vanishes outside
EI .
(4) If f, g : I −→ C are both of measure zero in this sense then f + g vanishes
on I \ (Ef ∪ Eg ) where Ef ⊂ I and Ef ⊂ I are of measure zero. The
union of two sets of measure zero (in R) is of measure zero so this shows
f + g is null. The same is true of cf + dg for constant c and d, so N (I)
is a linear space.
(5) If f ∈ L1 (I) and g ∈ N (I) then |f + g| − |f | ∈ N (I), since it vanishes
where g vanishes. Thus
Z Z
(.68) |f + g| = |f | ∀ f ∈ L1 (I), g ∈ N (I).
I I
Thus
Z
(.69) k[f ]kI = |f |
I
164 SOLUTIONS TO PROBLEMS

is a well-defined function on L1 (I) = L1 (R)/N (I) since it is constant


on equivalence classes. Now, the norm properties follow from the same
properties on the whole of R.
(6) Suppose f ∈ L1 (R) and g is defined in (.66) above by restriction to I.
We need to show that g ∈ L1 (R). If fn is an absolutely summable series
of step functions converging to f wherever, on R, it converges absolutely
consider
(
fn (x) on I˜
(.70) gn (x) =
0 on R \ I˜

where I˜ is I made half-open if it isn’t already – by adding the lower


end-point (if there is one) and removing the upper end-point (if there is
one). Then ˜
R R gn is a step function (which is why we need I). Moreover,
|gn | ≤ |fn | so the series gn is absolutely summable and converges
to gn outside I and at all points inside I where the series is absolutely
convergent (since it is then the same as fn ). Thus g is integrable, and since
f˜ differs from g by its values at two points, at most, it too is integrable
so f is integrable on I by definition.
(7) First we check we do have a map. Namely if f ∈ N (R) then g in (.66) is
certainly an element of N (I). We have already seen that ‘restriction to I’
maps L1 (R) into L1 (I) and since this is clearly a linear map it defines (.67)
– the image only depends on the equivalence class of f. It is clearly linear
and to see that it is surjective observe that if g ∈ L1 (I) then extending it
as zero outside I gives an element of L1 (R) and the class of this function
maps to [g] under (.67).
Problem 3.3 Really continuing the previous one.
(1) Show that if I = [a, b) and f ∈ L1 (I) then the restriction of f to Ix = [x, b)
is an element of L1 (Ix ) for all a ≤ x < b.
(2) Show that the function
Z
(.71) F (x) = f : [a, b) −→ C
Ix

is continuous.
(3) Prove that the function x−1 cos(1/x) is not Lebesgue integrable on the
interval (0, 1]. Hint: Think about it a bit and use what you have shown
above.
Solution:
(1) This follows from the previous question. If f ∈ L1 ([a, b)) with f 0 a repre-
sentative then extending f 0 as zero outside the interval gives an element
of L1 (R), by defintion. As an element of L1 (R) this does not depend on
the choice of f 0 and then (.67) gives the restriction to [x, b) as an element
of L1 ([x, b)). This is a linear map.
(2) Using the discussion in the preceeding question, we now that if fn is an
absolutely summable series converging to f 0 (a representative of f ) where
it converges absolutely, then for any a ≤ x ≤ b, we can define
(.72) fn0 = χ([a, x))fn , fn00 = χ([x, b))fn
SOLUTIONS TO PROBLEMS 165

where χ([a, b)) is the characteristic function of the interval. It follows


that fn0 converges to f χ([a, x)) and fn00 to f χ([x, b)) where they converge
absolutely. Thus
Z Z XZ Z Z XZ
(.73) f = f χ([x, b)) = fn00 , f = f χ([a, x)) = fn0 .
[x,b) n [a,x) n

fn0 fn00
R R R
Now, for step functions, we know that fn = + so
Z Z Z
(.74) f= f+ f
[a,b) [a,x) [x,b)

as we have every right to expect. Thus it suffices to show (by moving the
end point from a to a general point) that
Z
(.75) lim f =0
x→a [a,x)

for any f integrable on [a, b). Thus can be seen in terms of a defining
absolutely summable sequence of step functions using the usual estimate
that
Z Z X XZ
(.76) | f| ≤ | fn | + |fn |.
[a,x) [a,x) n≤N n>N [a,x)

The last sum can be made small, independent of x, by choosing N large


enough. On the other hand as x → a the first integral, for fixed N, tends
to zero by the definition for step functions. This proves (.76) and hence
the continuity of F.
(3) If the function x−1 cos(1/x) were Lebesgue integrable on the interval (0, 1]
(on which it is defined) then it would be integrable on [0, 1) if we define
it arbitrarily, say to be 0, at 0. The same would be true of the absolute
value and Riemann integration shows us easily that
Z 1
(.77) lim x| cos(1/x)|dx = ∞.
t↓0 t
This is contrary to the continuity of the integral as a function of the limits
just shown.
Problem 3.4 [Harder but still doable] Suppose f ∈ L1 (R).
(1) Show that for each t ∈ R the translates
(.78) ft (x) = f (x − t) : R −→ C
are elements of L1 (R).
(2) Show that
Z
(.79) lim |ft − f | = 0.
t→0

This is called ‘Continuity in the mean for integrable functions’. Hint: I


will add one!
(3) Conclude that for each f ∈ L1 (R) the map (it is a ‘curve’)
(.80) R 3 t 7−→ [ft ] ∈ L1 (R)
is continuous.
Solution:
166 SOLUTIONS TO PROBLEMS

(1) If fn is an absolutely summable series of step functions converging to f


where it converges absolutely then fn (· − t) is such a series converging to
f (· − t) for each t ∈ R. Thus, each of the f (x − t) is Lebesgue integrable,
i.e. are elements of L1 (R)
(2) Now, we know that if fn is a series converging to f as above then
Z XZ
(.81) |f | ≤ |fn |.
n

We can sum the first terms and then start the series again and so it follows
that for any N,
Z Z X XZ
(.82) |f | ≤ | fn | + |fn |.
n≤N n>N

Applying this to the series fn (· − t) − fn (·) we find that


Z Z X XZ
(.83) |ft − f | ≤ | fn (· − t) − fn (·)| + |fn (· − t) − fn (·)|
n≤N n>N
P R
The second sum here is bounded by 2 |fn |. Given δ > 0 we can
n>N
choose N so large that this sum is bounded by δ/2, by the absolute con-
vergence. So the result is reduce to proving that if |t| is small enough
then
Z X
(.84) | fn (· − t) − fn (·)| ≤ δ/2.
n≤N

This however is a finite sum of step functions. So it suffices to show that


Z
(.85) | g(· − t) − g(·)| → 0 as t → 0

for each component, i.e. a constant, c, times the characteristic function


of an interval [a, b) where it is bounded by 2|c||t|.
(3) For the ‘curve’ ft which is a map
(.86) R 3 t 7−→ ft ∈ L1 (R)
it follows that ft+s = (ft )s so we can apply the argument above to show
that for each s,
Z
(.87) lim |ft − fs | = 0 =⇒ lim k[ft ] − [fs ]kL1 = 0
t→s t→s

which proves continuity of the map (.86).


Problem 3.5 In the last problem set you showed that a continuous function
on a compact interval, extended to be zero outside, is Lebesgue integrable. Using
this, and the fact that step functions are dense in L1 (R) show that the linear space
of continuous functions on R each of which vanishes outside a compact set (which
depends on the function) form a dense subset of L1 (R).
Solution: Since we know that step functions (really of course the equivalence
classes of step functions) are dense in L1 (R) we only need to show that any step
function is the limit of a sequence of continuous functions each vanishing outside a
SOLUTIONS TO PROBLEMS 167

compact set, with respect to L1 . So, it suffices to prove this for the charactertistic
function of an interval [a, b) and then multiply by constants and add. The sequence



 0 x < a − 1/n
n(x − a + 1/n) a − 1/n ≤ x ≤ a



(.88) gn (x) = 0 a<x<b

n(b + 1/n − x) b ≤ x ≤ b + 1/n





0 x > b + 1/n
is clearly continuous and vanishes outside a compact set. Since
Z Z 1 Z b+1/n
(.89) |gn − χ([a, b))| = gn + gn ≤ 2/n
a−1/n b

it follows that [gn ] → [χ([a, b))] in L1 (R). This proves the density of continuous
functions with compact support in L1 (R).
Problem 3.6
(1) If g : R −→ C is bounded and continuous and f ∈ L1 (R) show that
gf ∈ L1 (R) and that
Z Z
(.90) |gf | ≤ sup |g| · |f |.
R

(2) Suppose now that G ∈ C([0, 1]×[0, 1]) is a continuous function (I use C(K)
to denote the continuous functions on a compact metric space). Recall
from the preceeding discussion that we have defined L1 ([0, 1]). Now, using
the first part show that if f ∈ L1 ([0, 1]) then
Z
(.91) F (x) = G(x, ·)f (·) ∈ C
[0,1]

(where · is the variable in which the integral is taken) is well-defined for


each x ∈ [0, 1].
(3) Show that for each f ∈ L1 ([0, 1]), F is a continuous function on [0, 1].
(4) Show that
(.92) L1 ([0, 1]) 3 f 7−→ F ∈ C([0, 1])
is a bounded (i.e. continuous) linear map into the Banach space of con-
tinuous functions, with supremum norm, on [0, 1].
Solution:
(1) Let’s first assume that f = 0 outside [−1, 1]. Applying a result form Prob-
lem set there exists a sequence of step functions gn such that for any R,
gn → g uniformly on [0, 1). By passing to a subsequence we can arrange
that sup[−1,1] |gn (x) − gn−1 (x)| < 2−n . If fn is an absolutly summable se-
ries of step functions converging a .e . to f we can replace it by fn χ([−1, 1])
as discussed above, and still have the same conclusion. Thus, from the
uniform convergence of gn ,
n
X
(.93) gn (x) fk (x) → g(x)f (x) a.e. on R.
k=1
168 SOLUTIONS TO PROBLEMS

n
P n−1
P
So define h1 = g1 f1 , hn = gn (x) fk (x) − gn−1 (x) fk (x). This series
k=1 k=1
of step functions converges to gf (x) almost everywhere and since
(.94)
X XZ XZ XZ
|hn | ≤ A|fn (x)| + 2−n |fk (x)|, |hn | ≤ A |fn | + 2 |fn | < ∞
k<n n n n

it is absolutely summable. Here A is a bound for |gn | independent of n.


Thus gf ∈ L1 (R) under the assumption that f = 0 outside [0, 1) and
Z Z
(.95) |gf | ≤ sup |g| |f |

follows from the limiting argument. Now we can apply this argument to
fp which is the restriction of p to the interval [p, p + 1), for each p ∈ Z.
Then we get gf as the limit a .e . of the absolutely summable series gfp
where (.95) provides the absolute summablitly since
XZ XZ
(.96) |gfp | ≤ sup |g| |f | < ∞.
p p [p,p+1)

Thus, gf ∈ L1 (R) by a theorem in class and


Z Z
(.97) |gf | ≤ sup |g| |f |.

(2) If f ∈ L1 [(0, 1]) has a representative f 0 then G(x, ·)f 0 (·) ∈ L1 ([0, 1)) so
Z
(.98) F (x) = G(x, ·)f (·) ∈ C
[0,1]

is well-defined, since it is indpendent of the choice of f 0 , changing by a


null function if f 0 is changed by a null function.
(3) Now by the uniform continuity of continuous functions on a compact met-
ric space such as S = [0, 1] × [0, 1] given δ > 0 there exist  > 0 such that
(.99) sup |G(x, y) − G(x0 , y)| < δ if |x − x0 | < .
y∈[0,1]

Then if |x − x0 | < ,
Z Z
(.100) |F (x) − F (x0 )| = | (G(x, ·) − G(x0 , ·))f (·)| ≤ δ |f |.
[0,1]

Thus F ∈ C([0, 1]) is a continuous function on [0, 1]. Moreover the map
f 7−→ F is linear and
Z
(.101) sup |F | ≤ sup |G| ||f |
[0,1] S [0,1]

which is the desired boundedness, or continuity, of the map


Z
(.102) I : L1 ([0, 1]) −→ C([0, 1]), F (f )(x) = G(x, ·)f (·),

kI(f )ksup ≤ sup |G|kf kL1 .


SOLUTIONS TO PROBLEMS 169

You should be thinking about using Lebesgue’s dominated convergence at sev-


eral points below.
Problem 5.1
Let f : R −→ C be an element of L1 (R). Define
(
f (x) x ∈ [−L, L]
(.103) fL (x) =
0 otherwise.

Show that fL ∈ L1 (R) and that |fL − f | → 0 as L → ∞.


R

Solution. If χL is the characteristic function of [−N, N ] then fL = f χL . If


fn is an absolutely summable seriesR of step functionsR converging a.e. to f then
fn χRL is absolutely summable, since |fn χL | ≤ |fn | and converges a.e. to fL , so
fL L1 (R). Certainly |fL (x) − f (x)| → 0 for each x as L → ∞ and |fRL (x) − f (x)| ≤
|fl (x)| + |f (x)| ≤ 2|f (x)| so by Lebesgue’s dominated convergence, |f − fL | → 0.
Problem 5.2 Consider a real-valued function f : R −→ R which is locally
integrable in the sense that
(
f (x) x ∈ [−L, L]
(.104) gL (x) =
0 x ∈ R \ [−L, L]
is Lebesgue integrable of each L ∈ N.
(1) Show that for each fixed L the function

gL (x) if gL (x) ∈ [−N, N ]

(N )
(.105) gL (x) = N if gL (x) > N

−N if gL (x) < −N

is Lebesgue integrable.
R (N )
(2) Show that |gL − gL | → 0 as N → ∞.
(3) Show that there is a sequence, hn , of step functions such that
(.106) hn (x) → f (x) a.e. in R.
(4) Defining


0 x 6∈ [−L, L]

h (x) if h (x) ∈ [−N, N ], x ∈ [−L, L]
(N ) n n
(.107) hn,L = .


N if hn (x) > N, x ∈ [−L, L]
−N if hn (x) < −N, x ∈ [−L, L]

R (N ) (N )
Show that |hn,L − gL | → 0 as n → ∞.
Solution:
(N )
(1) By definition gL = max(−N χL , min(N χL , gL )) where χL is the charac-
teristic funciton of −[L, L], thus it is in L1 (R).
(N ) (N )
(2) Clearly gL (x) → gL (x) for every x and |gL (x)| ≤ |gL (x)| so by Dom-
(N ) (N )
inated Convergence, gL → gL in L1 , i.e. |gL − gL | → 0 as N → ∞
R

since the sequence converges to 0 pointwise and is bounded by 2|g(x)|.


(3) Let SL,n be a sequence of step functions converging a.e. to gL – for ex-
ample the sequence of partial sums of an absolutely summable series of
step functions converging to gL which exists by the assumed integrability.
170 SOLUTIONS TO PROBLEMS

Then replacing SL,n by SL,n χL we can assume that the elements all van-
ish outside [−N, N ] but still have convergence a.e. to gL . Now take the
sequence
(
Sk,n−k on [k, −k] \ [(k − 1), −(k − 1)], 1 ≤ k ≤ n,
(.108) hn (x) =
0 on R \ [−n, n].
This is certainly a sequence of step functions – since it is a finite sum of
step functions for each n – and on [−L, L] \ [−(L − 1), (L − 1)] for large
integral L is just SL,n−L → gL . Thus hn (x) → f (x) outside a countable
union of sets of measure zero, so also almost everywhere.
(N ) (N )
(4) This is repetition of the first problem, hn,L (x) → gL almost everywhere
(N ) (N ) R (N ) (N )
and |hn,L | ≤ N χL so gL ∈ L1 (R) and |hn,L − gL | → 0 as n → ∞.
Problem 5.3 Show that L2 (R) is a Hilbert space – since it is rather central to
the course I wanted you to go through the details carefully!
First working with real functions, define L2 (R) as the set of functions f : R −→
R which are locally integrable and such that |f |2 is integrable.
(N ) (N )
(1) For such f choose hn and define gL , gL and hn by (.104), (.105) and
(.107).
(N ) (N ) (N )
(2) Show using the sequence hn,L for fixed N and L that gL and (gL )2
(N ) (N )
are in L1 (R) and that |(hn,L )2 − (gL )2 | → 0 as n → ∞.
R
(N )
(3) Show that R(gL )2 ∈ L1 (R) and that |(gL )2 − (gL )2 | → 0 as N → ∞.
R
2 2
(4) Show that |(gL ) − f | → 0 as L → ∞.
(5) Show that f, g ∈ L2 (R) then f g ∈ L1 (R) and that
Z Z Z
(.109) | f g| ≤ |f g| ≤ kf kL2 kgkL2 , kf k2L2 = |f |2 .

(6) Use these constructions to show that L2 (R) is a linear space.


(7) Conclude that the quotient space L2 (R) = L2 (R)/N , where N is the space
of null functions, is a real Hilbert space.
(8) Extend the arguments to the case of complex-valued functions.
Solution:
(N )
(1) Done. I think it should have been hn,L .
(N )
(2) We already checked that gL ∈ L1 (R) and the same argument applies to
(N ) (N ) (N )
(gL ), namely (hn,L )2 → gL almost everywhere and both are bounded
by N 2 χL so by dominated convergence
(N ) (N ) (N )
(hn,L )2 → gL )2 ≤ N 2 χL a.e. =⇒ gL )2 ∈ L1 (R) and
(N ) (N )
(.110) |hn,L )2 − gL )2 | → 0 a.e. ,
Z
(N ) (N ) (N ) (N )
|hn,L )2 − gL )2 | ≤ 2N 2 χL =⇒ |hn,L )2 − gL )2 | → 0.
(N ) (N )
(3) Now, as N → ∞, (gL )2 → (gL )2 a .e . and (gL )2 → (gL )2 ≤ f 2 so
(N )
by dominated convergence, (gL )2 ∈ L1 and |(gL )2 − (gL )2 | → 0 as
R

N → ∞.
2
(4) The Rsame argument of dominated convergence shows now that gL → f2
2 2 2 1
and |gL − f | → 0 using the bound by f ∈ L (R).
SOLUTIONS TO PROBLEMS 171

(5) What this is all for is to show that f g ∈ L1 (R) if f, F = g ∈ L2 (R) (for
easier notation). Approximate each of them by sequences of step functions
(N ) (N )
as above, hn,L for f and Hn,L for g. Then the product sequence is in L1
– being a sequence of step functions – and
(N ) (N ) (N ) (N )
(.111) hn,L (x)Hn,L (x) → gL (x)GL (x)
almost everywhere and with absolute value bounded by N 2 χL . Thus
(N ) (N )
by dominated convergence gL GL ∈ L1 (R). Now, let N → ∞; this
sequence converges almost everywhere to gL (x)GL (x) and we have the
bound
(N ) (N ) 1
(.112) |gL (x)GL (x)| ≤ |f (x)F (x)| (f 2 + F 2 )
2
so as always by dominated convergence, the limit gL GL ∈ L1 . Finally,
letting L → ∞ the same argument shows that f F ∈ L1 (R). Moreover,
|f F | ∈ L1 (R) and
Z Z
(.113) | f F | ≤ |f F | ≤ kf kL2 kF kL2

where the last inequality follows from Cauchy’s inequality – if you wish,
first for the approximating sequences and then taking limits.
(6) So if f, g ∈ L2 (R) are real-value, f + g is certainly locally integrable and
(.114) (f + g)2 = f 2 + 2f g + g 2 ∈ L1 (R)
by the discussion above. For constants f ∈ L2 (R) implies cf ∈ L2 (R) is
directly true.
(7) The argument is the same as for L1 versus L1 . Namely f 2 = 0 implies
R

that f 2 = 0 almost everywhere which is equivalent to f = 0 a@ė. Then


the norm is the same for all f + h where h is a null function since f h and
h2 are null so (f + h)2 = f 2 + 2f h + h2 . The same is true for the inner
product so it follows that the quotient by null functions
(.115) L2 (R) = L2 (R)/N
is a preHilbert space.
P {[fn ]} is an ab-
However, it remains to show completeness. Suppose
solutely summable series in L2 (R) which means that kfn kL2 < ∞. It
n
follows that the cut-off series fn χL is absolutely summable in the L1 sense
since
Z Z
1 1
(.116) |fn χL | ≤ L 2 ( fn2 ) 2
n
P
by Cauchy’s inequality. Thus if we set Fn = fk then Fn (x)χL con-
k−1
verges almost everywhere for each L so in fact
(.117) Fn (x) → f (x) converges almost everywhere.
We want to show that f ∈ L2 (R) where it follows already that f is locally
integrable by the completeness of L1 . Now consider the series
(.118) g1 = F12 , gn = Fn2 − Fn−1
2
.
172 SOLUTIONS TO PROBLEMS

The elements are in L1 (R) and by Cauchy’s inequality for n > 1,


Z Z
|gn | = |Fn2 − Fn−1 |2 ≤ kFn − Fn−1 kL2 kFn + Fn−1 kL2
(.119) X
≤ kfn kL2 2 kfk kL2
k
where the triangle inequality has been used. Thus in fact the series gn is
absolutely summable in L1
XZ X
(.120) |gn | ≤ 2( kfn kL2 )2 .
n n

So indeed the sequence of partial sums, the Fn2 converge to f 2 ∈ L1 (R).


Thus f ∈ L2 (R) and moroever
Z Z Z Z
(.121) (Fn − f )2 = Fn2 + f 2 − 2 Fn f → 0 as n → ∞.

Indeed the first term converges to f 2 and, by Cauchys inequality, the


R

series of products fn f is absulutely summable in L1 with limit f 2 so the


third term converges to −2 f . Thus in fact [Fn ] → [f ] in L2 (R) and we
R 2

have proved completeness.


(8) For the complex case we need to check linearity, assuming f is locally
integrable and |f |2 ∈ L1 (R). The real part of f is locally integrable and the
(N ) (N )
approximation FL discussed above is square integrable with (FL )2 ≤
|f |2 so by dominated convergence, letting first N → ∞ and then L → ∞
the real part is in L2 (R). Now linearity and completeness follow from the
real case.
Problem 5.4
Consider the sequence space
 
 X 
(.122) h2,1 = c : N 3 j 7−→ cj ∈ C; (1 + j 2 )|cj |2 < ∞ .
 
j

(1) Show that


X
(.123) h2,1 × h2,1 3 (c, d) 7−→ hc, di = (1 + j 2 )cj dj
j

is an Hermitian inner form which turns h2,1 into a Hilbert space.


(2) Denoting the norm on this space by k · k2,1 and the norm on l2 by k · k2 ,
show that
(.124) h2,1 ⊂ l2 , kck2 ≤ kck2,1 ∀ c ∈ h2,1 .
Solution:
(1) The inner product is well defined since the series defining it converges
absolutely by Cauchy’s inequality:
1
X 1
hc, di = (1 + j 2 ) 2 cj (1 + j 2 ) 2 dj ,
j
(.125) X 1 1
X 1
X 1
|(1 + j ) cj (1 + j 2 ) dj | ≤ ( (1 + j 2 )|cj |2 ) 2 ( (1 + j 2 )|dj |2 ) 2 .
2 2 2

j j j
SOLUTIONS TO PROBLEMS 173

It is sesquilinear and positive definite since


1
X
(.126) kck2,1 = ( (1 + j 2 )|cj |2 ) 2
j

only vanishes if all cj vanish. Completeness follows as for l2 – if c(n) is a


(n) 1 (n)
Cauchy sequence then each component cj converges, since (1 + j) 2 cj
is Cauchy. The limits cj define an element of h2,1 since the sequence is
bounded and
N N
1 (n)
X X
(.127) (1 + j 2 ) 2 |cj |2 = lim (1 + j 2 )|cj |2 ≤ A
n→∞
j=1 j=1

where A is a bound on the norms. Then from the Cauchy condition


c(n) → c in h2,1 by passing to the limit as m → ∞ in kc(n) − c(m) k2,1 ≤ .
(2) Clearly h2,2 ⊂ l2 since for any finite N
N
X N
X
(.128) |cj |2 (1 + j)2 |cj |2 ≤ kck22,1
j=1 j=1

and we may pass to the limit as N → ∞ to see that


(.129) kckl2 ≤ kck2,1 .
Problem 5.5 In the separable case, prove Riesz Representation Theorem di-
rectly.
Choose an orthonormal basis {ei } of the separable Hilbert space H. Suppose
T : H −→ C is a bounded linear functional. Define a sequence
(.130) wi = T (ei ), i ∈ N.
(1) Now, recall that |T u| ≤ CkukH for some constant C. Show that for every
finite N,
N
X
(.131) |wi |2 ≤ C 2 .
j=1
2
(2) Conclude that {wi } ∈ l and that
X
(.132) w= wi ei ∈ H.
i
(3) Show that
(.133) T (u) = hu, wiH ∀ u ∈ H and kT k = kwkH .
Solution:
N
P
(1) The finite sum wN = wi ei is an element of the Hilbert space with norm
i=1
N
kwN k2N = |wi |2 by Bessel’s identity. Expanding out
P
i=1
N
X n
X N
X
(.134) T (wN ) = T ( w i ei ) = wi T (ei ) = |wi |2
i=1 i=1 i=1
and from the continuity of T,
(.135) |T (wN )| ≤ CkwN kH =⇒ kwN k2H ≤ CkwN kH =⇒ kwN k2 ≤ C 2
174 SOLUTIONS TO PROBLEMS

which is the desired inequality.


(2) Letting N → ∞ it follows that the infinite sum converges and
X X
(.136) |wi |2 ≤ C 2 =⇒ w = w i ei ∈ H
i i
2
P
since kwN − wk ≤ |wi | tends to zero with N.
j>N
N
P
(3) For any u ∈ H uN = hu, ei iei by the completness of the {ei } so from
i=1
the continuity of T
N
X
(.137) T (u) = lim T (uN ) = lim hu, ei iT (ei )
N →∞ N →∞
i=1
N
X
= lim hu, wi ei i = lim hu, wN i = hu, wi
N →∞ N →∞
i=1

where the continuity of the inner product has been used. From this and
Cauchy’s inequality it follows that kT k = supkukH =1 |T (u)| ≤ kwk. The
converse follows from the fact that T (w) = kwk2H .
Solution .8. If f ∈ L1 (Rk × Rp ) show that there exists a set of measure zero
E ⊂ Rk such that
(.138) x ∈ Rk \ E =⇒ gx (y) = f (x, y) defines gx ∈ L1 (Rp ),
gx defines an element F ∈ L1 (Rk ) and that
R
that F (x) =
Z Z
(.139) F = f.
Rk Rk ×Rp

Note: These identities are usually written out as an equality of an iterated


integral and a ‘regular’ integral:
Z Z Z
(.140) f (x, y) = f.
Rk Rp
It is often used to ‘exchange the order of integration’ since the hypotheses are
the same if we exchange the variables.
Solution. This is not hard but is a little tricky (I believe Fubini never under-
stood what the fuss was about).
Certainly this result holds for step functions, since ultimately it reduces to the
case of the characterisitic function for a ‘rectrangle’.
In the general case we can take an absolutely summable sequence fj of step
functions summing to f
X X
(.141) f (x, y) = fj (x, y) whenever |fj (x, y)| < ∞.
j j

This, after all, is our definition of integrability.


Now, consider the functions
Z
(.142) hj (x) = |fj (x, ·)|
Rp
SOLUTIONS TO PROBLEMS 175

which are step functions. Moreover this series is absolutely summable since
XZ XZ
(.143) |hj | = |fj |.
j Rk j Rk ×Rp

hj (x) converges (absolutely) on the complement of a set E ⊂ Rk


P
Thus the series
j
of measure zero. It follows that the series of step functions
Z
(.144) Fj (x) = fj (x, ·)
Rp
k
converges absolutely on R \ E since |fj (x)| ≤ hj (x). Thus,
X
(.145) F (x) = Fj (x) converges absolutely on Rk \ E
j
1 k
defines F ∈ L (R ) with
Z XZ XZ Z
(.146) F = Fj = fj = f.
Rk j Rk j Rk ×Rp Rk ×Rp
P
The absolute convergence of hj (x) for a given x is precisely the absolutely
j
summability of fk (x, y) as a series of functions of y,
XZ X
(.147) |fj (x, ·)| = hj (x).
j Rp j

fk (x, y) must converge absolutely for y ∈ (Rp \Ex )


P
Thus for each x ∈
/ E the series
j
where Ex is a set of measure zero. But (.141) shows that the sum is gx (y) = f (x, y)
/ E, f (x, ·) ∈ L1 (Rp ) (as the limit of an absolutely
at all such points, so for x ∈
summable series) and
Z
(.148) F (x) = gx .
Rp
With (.146) this is what we wanted to show. 
Problem 4.1
Let H be a normed space in which the norm satisfies the parallelogram law:
(.149) ku + vk2 + ku − vk2 = 2(kuk2 + kvk2 ) ∀ u, v ∈ H.
Show that the norm comes from a positive definite sesquilinear (i.e. Hermitian)
inner product. Big Hint:- Try
1
ku + vk2 − ku − vk2 + iku + ivk2 − iku − ivk2 !

(.150) (u, v) =
4
Solution: Setting u = v, even without the parallelogram law,
1
(u, u) = 2uk2 + ik(1 + i)uk2 − ik(1 − i)uk2 = kuk2 .

(.151)
4
So the point is that the parallelogram law shows that (u, v) is indeed an Hermitian
inner product. Taking complex conjugates and using properties of the norm, ku +
ivk = kv − iuk etc
1
kv + uk2 − kv − uk2 − ikv − iuk2 + ikv + iuk2 = (v, u).

(.152) (u, v) =
4
176 SOLUTIONS TO PROBLEMS

Thus we only need check the linearity in the first variable. This is a little tricky!
First compute away. Directly from the identity (u, −v) = −(u, v) so (−u, v) =
−(u, v) using (.152). Now,
(.153)
1
(2u, v) = ku + (u + v)k2 − ku + (u − v)k2
4
+ iku + (u + iv)k2 − iku + (u − iv)k2


1
= ku + vk2 + kuk2 − ku − vk2 − kuk2
2
+ ik(u + iv)k2 + ikuk2 − iku − ivk2 − ikuk2


1
ku − (u + v)k2 − ku − (u − v)k2 + iku − (u + iv)k2 − iku − (u − iv)k2


4
=2(u, v).
Using this and (.152), for any u, u0 and v,
1
(u + u0 , v) = (u + u0 , 2v)
2
11
= k(u + v) + (u0 + v)k2 − k(u − v) + (u0 − v)k2
24
+ ik(u + iv) + (u − iv)k2 − ik(u − iv) + (u0 − iv)k2


1
(.154) = ku + vk + ku0 + vk2 − ku − vk − ku0 − vk2
4
+ ik(u + iv)k2 + iku − ivk2 − iku − ivk − iku0 − ivk2


11
− k(u + v) − (u0 + v)k2 − k(u − v) − (u0 − v)k2
24
+ ik(u + iv) − (u − iv)k2 − ik(u − iv) = (u0 − iv)k2


= (u, v) + (u0 , v).


Using the second identity to iterate the first it follows that (ku, v) = k(u, v) for any
u and v and any positive integer k. Then setting nu0 = u for any other positive
integer and r = k/n, it follows that
(.155) (ru, v) = (ku0 , v) = k(u0 , v) = rn(u0 , v) = r(u, v)
where the identity is reversed. Thus it follows that (ru, v) = r(u, v) for any rational
r. Now, from the definition both sides are continuous in the first element, with
respect to the norm, so we can pass to the limit as r → x in R. Also directly from
the definition,
1
kiu + vk2 − kiu − vk2 + ikiu + ivk2 − ikiu − ivk2 = i(u, v)

(.156) (iu, v) =
4
so now full linearity in the first variable follows and that is all we need.
Problem 4.2
Let H be a finite dimensional (pre)Hilbert space. So, by definition H has a
basis {vi }ni=1 , meaning that any element of H can be written
X
(.157) v= c i vi
i
and there is no dependence relation between the vi ’s – the presentation of v = 0 in
the form (.157) is unique. Show that H has an orthonormal basis, {ei }ni=1 satisfying
SOLUTIONS TO PROBLEMS 177

(ei , ej ) = δij (= 1 if i = j and 0 otherwise). Check that for the orthonormal basis
the coefficients in (.157) are ci = (v, ei ) and that the map
(.158) T : H 3 v 7−→ ((v, ei )) ∈ Cn
is a linear isomorphism with the properties
X
(.159) (u, v) = (T u)i (T v)i , kukH = kT ukCn ∀ u, v ∈ H.
i

Why is a finite dimensional preHilbert space a Hilbert space?


Solution: Since H is assumed to be finite dimensional, it has a basis vi , i =
1, . . . , n. This basis can be replaced by an orthonormal basis in n steps. First
replace v1 by e1 = v1 /kv1 k where kv1 k =6 0 by the linear indepedence of the basis.
Then replace v2 by
(.160) e2 = w2 /kw2 k, w2 = v2 − (v2 , e1 )e1 .
Here w2 ⊥ e1 as follows by taking inner products; w2 cannot vanish since v2 and e1
must be linearly independent. Proceeding by finite induction we may assume that
we have replaced v1 , v2 , . . . , vk , k < n, by e1 , e2 , . . . , ek which are orthonormal
and span the same subspace as the vi ’s i = 1, . . . , k. Then replace vk+1 by
k
X
(.161) ek+1 = wk+1 /kwk+1 k, wk+1 = vk+1 − (vk+1 , ei )ei .
i=1

By taking inner products, wk+1 ⊥ ei , i = 1, . . . , k and wk+1 6= 0 by the linear


independence of the vi ’s. Thus the orthonormal set has been increased by one
element preserving the same properties and hence the basis can be orthonormalized.
Now, for each u ∈ H set
(.162) ci = (u, ei ).
n
P
It follows that U = u − ci ei is orthogonal to all the ei since
i=1
X
(.163) (u, ej ) = (u, ej ) − ci (ei , ej ) = (u.ej ) − cj = 0.
i
P
This implies that U = 0 since writing U = di ei it follows that di = (U, ei ) = 0.
i
Now, consider the map (.158). We have just shown that this map is injective,
since T u = 0 implies ci = 0 for all i and hence u = 0. It is linear since the ci depend
linearly on u by the linearity of the innerP product in the first variable. Moreover
it is surjective, since for any ci ∈ C, u = ci ei reproduces the ci through (.162).
i
Thus T is a linear isomorphism and the first identity in (.159) follows by direct
computation:-
n
X X
(T u)i (T v)i = (u, ei )
i=1 i
(.164) X
= (u, (v, ei )ei )
i
= (u, v).
Setting u = v shows that kT ukCn = kukH .
178 SOLUTIONS TO PROBLEMS

Now, we know that Cn is complete with its standard norm. Since T is an


isomorphism, it carries Cauchy sequences in H to Cauchy sequences in Cn and T −1
carries convergent sequences in Cn to convergent sequences in H, so every Cauchy
sequence in H is convergent. Thus H is complete.
Hint: Don’t pay too much attention to my hints, sometimes they are a little off-
the-cuff and may not be very helpfult. An example being the old hint for Problem
6.2!
Problem 6.1 Let H be a separable Hilbert space. Show that K ⊂ H is compact
if and only if it is closed, bounded and has the property that any sequence in K
which is weakly convergent sequence in H is (strongly) convergent.
Hint:- In one direction use the result from class that any bounded sequence has
a weakly convergent subsequence.
Problem 6.2 Show that, in a separable Hilbert space, a weakly convergent
sequence {vn }, is (strongly) convergent if and only if the weak limit, v satisfies
(.165) kvkH = lim kvn kH .
n→∞

Hint:- To show that this condition is sufficient, expand


(.166) (vn − v, vn − v) = kvn k2 − 2 Re(vn , v) + kvk2 .
Problem 6.3 Show that a subset of a separable Hilbert space is compact if
and only if it is closed and bounded and has the property of ‘finite dimensional
approximation’ meaning that for any  > 0 there exists a linear subspace DN ⊂ H
of finite dimension such that
(.167) d(K, DN ) = sup inf {d(u, v)} ≤ .
u∈K v∈DN

Hint:- To prove necessity of this condition use the ‘equi-small tails’ property of
compact sets with respect to an orthonormal basis. To use the finite dimensional
approximation condition to show that any weakly convergent sequence in K is
strongly convergent, use the convexity result from class to define the sequence {vn0 }
in DN where vn0 is the closest point in DN to vn . Show that vn0 is weakly, hence
strongly, convergent and hence deduce that {vn } is Cauchy.
Problem 6.4 Suppose that A : H −→ H is a bounded linear operator with the
property that A(H) ⊂ H is finite dimensional. Show that if vn is weakly convergent
in H then Avn is strongly convergent in H.
Problem 6.5 Suppose that H1 and H2 are two different Hilbert spaces and
A : H1 −→ H2 is a bounded linear operator. Show that there is a unique bounded
linear operator (the adjoint) A∗ : H2 −→ H1 with the property
(.168) (Au1 , u2 )H2 = (u1 , A∗ u2 )H1 ∀ u1 ∈ H1 , u2 ∈ H2 .
Problem 8.1 Show that a continuous function K : [0, 1] −→ L2 (0, 2π) has
the property that the Fourier series of K(x) ∈ L2 (0, 2π), for x ∈ [0, 1], converges
uniformly in the sense that if Kn (x) is the sum of the Fourier series over |k| ≤ n
then Kn : [0, 1] −→ L2 (0, 2π) is also continuous and
(.169) sup kK(x) − Kn (x)kL2 (0,2π) → 0.
x∈[0,1]

Hint. Use one of the properties of compactness in a Hilbert space that you
proved earlier.
Problem 8.2
SOLUTIONS TO PROBLEMS 179

Consider an integral operator acting on L2 (0, 1) with a kernel which is contin-


uous – K ∈ C([0, 1]2 ). Thus, the operator is
Z
(.170) T u(x) = K(x, y)u(y).
(0,1)
2
Show that T is bounded on L (I think we did this before) and that it is in the
norm closure of the finite rank operators.
Hint. Use the previous problem! Show that a continuous function such as K in
this Problem defines a continuous map [0, 1] 3 x 7−→ K(x, ·) ∈ C([0, 1]) and hence
a continuous function K : [0, 1] −→ L2 (0, 1) then apply the previous problem with
the interval rescaled.
Here is an even more expanded version of the hint: You can think of K(x, y) as
a continuous function of x with values in L2 (0, 1). Let Kn (x, y) be the continuous
function of x and y given by the previous problem, by truncating the Fourier series
(in y) at some point n. Check that this defines a finite rank operator on L2 (0, 1)
– yes it maps into continuous functions but that is fine, they are Lebesgue square
integrable. Now, the idea is the difference K − Kn defines a bounded operator with
small norm as n becomes large. It might actually be clearer to do this the other
way round, exchanging the roles of x and y.
Problem 8.3 Although we have concentrated on the Lebesgue integral in one
variable, you proved at some point the covering lemma in dimension 2 and that is
pretty much all that was needed to extend the discussion to 2 dimensions. Let’s just
assume you have assiduously checked everything and so you know that L2 ((0, 2π)2 )
is a Hilbert space. Sketch a proof – noting anything that you are not sure of – that
the functions exp(ikx + ily)/2π, k, l ∈ Z, form a complete orthonormal basis.
P9.1: Periodic functions
Let S be the circle of radius 1 in the complex plane, centered at the origin,
S = {z; |z| = 1}.
(1) Show that there is a 1-1 correspondence

(.171) C 0 (S) = {u : S −→ C, continuous} −→


{u : R −→ C; continuous and satisfying u(x + 2π) = u(x) ∀ x ∈ R}.
Solution: The map E : R 3 θ 7−→ e2πiθ ∈ S is continuous, surjective
and 2π-periodic and the inverse image of any point of the circle is precisly
of the form θ + 2πZ for some θ ∈ R. Thus composition defines a map
(.172) E ∗ : C 0 (S) −→ C 0 (R), E ∗ f = f ◦ E.
This map is a linear bijection.
(2) Show that there is a 1-1 correspondence

(.173) L2 (0, 2π) ←→ {u ∈ L1loc (R); u (0,2π) ∈ L2 (0, 2π)


and u(x + 2π) = u(x) ∀ x ∈ R}/NP


where NP is the space of null functions on R satisfying u(x + 2π) = u(x)
for all x ∈ R.
Solution: Our original definition of L2 (0, 2π) is as functions on R
which are square-integrable and vanish outside (0, 2π). Given such a func-
tion u we can define an element of the right side of (.173) by assigning a
180 SOLUTIONS TO PROBLEMS

value at 0 and then extending by periodicity

(.174) ũ(x) = u(x − 2nπ), n ∈ Z

where for each x ∈ R there is a unique integer n so that x − 2nπ ∈ [0, 2π).
Null functions are mapped to null functions his way and changing the
choice of value at 0 changes ũ by a null function. This gives a map as in
(.173) and restriction to (0, 2π) is a 2-sided invese.
(3) If we denote by L2 (S) the space on the left in (.173) show that there is a
dense inclusion

(.175) C 0 (S) −→ L2 (S).

Solution: Combining the first map and the inverse of the second gives
an inclusion. We know that continuous functions vanishing near the end-
points of (0, 2π) are dense in L2 (0, 2π) so density follows.
So, the idea is that we can freely think of functions on S as 2π-periodic functions
on R and conversely.
P9.2: Schrödinger’s operator
Since that is what it is, or at least it is an example thereof:

d2 u(x)
(.176) − + V (x)u(x) = f (x), x ∈ R,
dx2
(1) First we will consider the special case V = 1. Why not V = 0? – Don’t
try to answer this until the end!
Solution: The reason we take V = 1, or at least some other positive
constant is that there is 1-d space of periodic solutions to d2 u/dx2 = 0,
namely the constants.
(2) Recall how to solve the differential equation

d2 u(x)
(.177) − + u(x) = f (x), x ∈ R,
dx2
where f (x) ∈ C 0 (S) is a continuous, 2π-periodic function on the line. Show
that there is a unique 2π-periodic and twice continuously differentiable
function, u, on R satisfying (.177) and that this solution can be written
in the form
Z
(.178) u(x) = (Sf )(x) = A(x, y)f (y)
0,2π

where A(x, y) ∈ C 0 (R2 ) satisfies A(x+2π, y +2π) = A(x, y) for all (x, y) ∈
R.
Extended hint: In case you managed to avoid a course on differential
equations! First try to find a solution, igonoring the periodicity issue. To
do so one can (for example, there are other ways) factorize the differential
operator involved, checking that

d2 u(x) dv du
(.179) − + u(x) = −( + v) if v = −u
dx2 dx dx
SOLUTIONS TO PROBLEMS 181

since the cross terms cancel. Then recall the idea of integrating factors to
see that
du dφ
− u = ex , φ = e−x u,
(.180) dx dx
dv −x dψ
+v =e , ψ = ex v.
dx dx
Now, solve the problem by integrating twice from the origin (say) and
hence get a solution to the differential equation (.177). Write this out
explicitly as a double integral, and then change the order of integration
to write the solution as
Z
(.181) u0 (x) = A0 (x, y)f (y)dy
0,2π

where A is continuous on R×[0, 2π]. Compute the difference u0 (2π)−u0 (0)


0
0
du0 0
and du
dx (2π) − dx (0) as integrals involving f. Now, add to u as solution
x −x
to the homogeneous equation, for f = 0, namely c1 e + c2 e , so that the
new solution to (.177) satisfies u(2π) = u(0) and du du
dx (2π) = dx (0). Now,
check that u is given by an integral of the form (.178) with A as stated.
Solution: Integrating once we find that if v = du dx − u then
Z x Z x
(.182) v(x) = −e−x es f (s)ds, u0 (x) = ex e−t v(t)dt
0 0
2 0
gives a solution of the equation − ddxu2 + u0 (x) = f (x) so combinging these
two and changing the order of integration
Z x
1 y−x
u0 (x) = − ex−y

Ã(x, y)f (y)dy, Ã(x, y) = e
0 2
(.183) (
1
(ey−x − ex−y ) x ≥ y
Z
u0 (x) = A0 (x, y)f (y)dy, A0 (x, y) = 2
(0,2π) 0 x ≤ y.

Here A0 is continuous since à vanishes at x = y where there might other-


wise be a discontinuity. This is the only solution which vanishes with its
derivative at 0. If it is to extend to be periodic we need to add a solution
of the homogeneous equation and arrange that
du du
(.184) u = u0 + u00 , u00 = cex + de−x , u(0) = u(2π), (0) = (2π).
dx dx
So, computing away we see that
(.185)
2π 2π
du0
Z Z
0 1 y−2π 1 y−2π
− e2π−y f (y), + e2π−y f (y).
 
u (2π) = e (2π) = − e
0 2 dx 0 2
Thus there is a unique solution to (.184) which must satify
(.186)
du0
c(e2π − 1) + d(e−2π − 1) = −u0 (2π), c(e2π − 1) − d(e−2π − 1) = − (2π)
dx
Z 2π Z 2π
1 1
(e2π − 1)c = e2π−y f (y), (e−2π − 1)d = − ey−2π f (y).
 
2 0 2 0
182 SOLUTIONS TO PROBLEMS

Putting this together we get the solution in the desired form:


(.187)
1 e2π−y+x 1 e−2π+y−x
Z
u(x) = A(x, y)f (y), A(x, y) = A0 (x, y) + − =⇒
(0.2π) 2 e2π − 1 2 e−2π − 1
cosh(|x − y| − π)
A(x, y) = .
eπ − e−π
(3) Check, either directly or indirectly, that A(y, x) = A(x, y) and that A is
real.
Solution: Clear from (.187).
(4) Conclude that the operator S extends by continuity to a bounded operator
on L2 (S). √
Solution. We know that kSk ≤ 2π sup |A|.
(5) Check, probably indirectly rather than directly, that
(.188) S(eikx ) = (k 2 + 1)−1 eikx , k ∈ Z.
Solution. We know that Sf is the unique solution with periodic
boundary conditions and eikx satisfies the boundary conditions and the
equation with f = (k 2 + 1)eikx .
(6) Conclude, either from the previous result or otherwise that S is a compact
self-adjoint operator on L2 (S).
Soluion: Self-adjointness
√ and compactness follows from (.188) since
ikx
we know that the e / 2π form an orthonormal basis, so the eigenvalues
of S tend to 0. (Myabe better to say it is approximable by finite rank
operators by truncating the sum).
(7) Show that if g ∈ C 0 (S)) then Sg is twice continuously differentiable. Hint:
Proceed directly by differentiating the integral.
Solution: Clearly Sf is continuous. Going back to the formula in
terms of u0 + u00 we see that both terms are twice continuously differen-
tiable.
(8) From (.188) conclude that S = F 2 where F is also a compact self-adjoint
1
operator on L2 (S) with eigenvalues (k 2 + 1)− 2 .
1
Solution: Define F (eikx ) = (k 2 + 1)− 2 eikx . Same argument as above
applies to show this is compact and self-adjoint.
(9) Show that F : L2 (S) −→ C 0 (S).
Solution. The series for Sf
1 X 2 1
(.189) Sf (x) = (2k + 1)− 2 (f, eikx )eikx

k
converges absolutely and uniformly, using Cauchy’s inequality – for in-
stance it is Cauchy in the supremum norm:
1
X
(.190) k (2k 2 + 1)− 2 (f, eikx )eikx | ≤ kf kL2
|k|>p

for p large since the sum of the squares of the eigenvalues is finite.
(10) Now, going back to the real equation (.176), we assume that V is contin-
uous, real-valued and 2π-periodic. Show that if u is a twice-differentiable
2π-periodic function satisfying (.176) for a given f ∈ C 0 (S) then
(.191) u + S((V − 1)u) = Sf and hence u = −F 2 ((V − 1)u) + F 2 f
SOLUTIONS TO PROBLEMS 183

and hence conclude that


(.192) u = F v where v ∈ L2 (S) satisfies v + (F (V − 1)F )v = F f
where V − 1 is the operator defined by multiplication by V − 1.
Solution: If u satisfies (.176) then
d2 u(x)
(.193) − + u(x) = −(V (x) − 1)u(x) + f (x)
dx2
so by the uniquenss of the solution with periodic boundary conditions,
u = −S(V − 1)u + Sf so u = F (−F (V − 1)u + F f ). Thus indeed u = F v
with v = −F (V − 1)u + F f which means that v satisfies
(.194) v + F (V − 1)F v = F f.
(11) Show the converse, that if v ∈ L2 (S) satisfies
(.195) v + (F (V − 1)F )v = F f, f ∈ C 0 (S)
then u = F v is 2π-periodic and twice-differentiable on R and satisfies
(.176).
Solution. If v ∈ L2 (0, 2π) satisfies (.195) then u = F v ∈ C 0 (S) sat-
isfies u + F 2 (V − 1)u = F 2 f and since F 2 = S maps C 0 (S) into twice
continuously differentiable functions it follows that u satisfies (.176).
(12) Apply the Spectral theorem to F (V − 1)F (including why it applies) and
show that there is a sequence λj in R \ {0} with |λj | → 0 such that for all
λ ∈ C \ {0}, the equation
(.196) λv + (F (V − 1)F )v = g, g ∈ L2 (S)
has a unique solution for every g ∈ L2 (S) if and only if λ 6= λj for any j.
Solution: We know that F (V − 1)F is self-adjoint and compact so
L2 (0.2π) has an orthonormal basis of eigenfunctions of −F (V − 1)F with
eigenvalues λj . This sequence tends to zero and (.196), for given λ ∈
C \ {0}, if and only if has a solution if and only if it is an isomorphism,
meaning λ 6= λj is not an eigenvalue of −F (V − 1)F.
(13) Show that for the λj the solutions of
(.197) λj v + (F (V − 1)F )v = 0, v ∈ L2 (S),
are all continuous 2π-periodic functions on R.
Solution: If v satisfies (.197) with λj 6= 0 then v = −F (V − 1)F/λj ∈
C 0 (S).
(14) Show that the corresponding functions u = F v where v satisfies (.197) are
all twice continuously differentiable, 2π-periodic functions on R satisfying
d2 u
(.198) − + (1 − sj + sj V (x))u(x) = 0, sj = 1/λj .
dx2
Solution: Then u = F v satisfies u = −S(V − 1)u/λj so is twice
continuously differentiable and satisfies (.198).
(15) Conversely, show that if u is a twice continuously differentiable, 2π-periodic
function satisfying
d2 u
(.199) − + (1 − s + sV (x))u(x) = 0, s ∈ C,
dx2
and u is not identically 0 then s = sj for some j.
184 SOLUTIONS TO PROBLEMS

Solution: From the uniquess of periodic solutions u = −S(V − 1)u/λj


as before.
(16) Finally, conclude that Fredholm’s alternative holds for the equation (.176)
Theorem 21. For a given real-valued, continuous 2π-periodic func-
tion V on R, either (.176) has a unique twice continuously differentiable,
2π-periodic, solution for each f which is continuous and 2π-periodic or
else there exists a finite, but positive, dimensional space of twice continu-
ously differentiable 2π-periodic solutions to the homogeneous equation
d2 w(x)
(.200) − + V (x)w(x) = 0, x ∈ R,
dx2
R
and (.176) has a solution if and only if (0,2π) f w = 0 for every 2π-periodic
solution, w, to (.200).
Solution: This corresponds to the special case λj = 1 above. If λj is not an
eigenvalue of −F (V − 1)F then
(.201) v + F (V − 1)F v = F f
has a unque solution for all f, otherwise the necessary and sufficient condition is
that (v, F f ) = 0 for all v 0 satisfying v 0 + F (V − 1)F v 0 = 0. Correspondingly either
(.176) has a unique solution for all f or the necessary and sufficient condition is
that (F v 0 , f ) = 0 for all w = F v 0 (remember that F is injetive) satisfying (.200).
Problem P10.1 Let H be a separable, infinite dimensional Hilbert space. Show
that the direct sum of two copies of H is a Hilbert space with the norm
1
(.202) H ⊕ H 3 (u1 , u2 ) 7−→ (ku1 k2H + ku2 k2H ) 2
either by constructing an isometric isomorphism
(.203) T : H −→ H ⊕ H, 1-1 and onto, kukH = kT ukH⊕H
or otherwise. In any case, construct a map as in (.203).
Solution: Let {ei }i∈N be an orthonormal basis of H, which exists by virtue of
the fact that it is an infinite-dimensional but separable Hilbert space. Define the
map
X∞ ∞
X
(.204) T : H 3 u −→ ( (u, e2i−1 )ei , (u, e2i )ei ) ∈ H ⊕ H
i=1 i=1
The convergence of the Fourier Bessel series shows that this map is well-defined
and linear. Injectivity similarly follows from the fact that T u = 0 in the image
implies that (u, ei ) = 0 for all i and hence u = 0. Surjectivity is also clear from the
fact that
X∞
(.205) S : H ⊕ H 3 (u1 , u2 ) 7−→ ((u1 , ei )e2i−1 + (u2 , ei )e2i ) ∈ H
i=1

is a 2-sided inverse and Bessel’s identity implies isometry since kS(u1 , u2 )k2 =
ku1 k2 + ku2 k2
Problem P10.2 One can repeat the preceding construction any finite number
of times. Show that it can be done ‘countably often’ in the sense that if H is a
separable, infinite dimensional, Hilbert space then
X
(.206) l2 (H) = {u : N −→ H; kuk2l2 (H) = kui k2H < ∞}
i
SOLUTIONS TO PROBLEMS 185

has a Hilbert space structure and construct an explicit isometric isomorphism from
l2 (H) to H.
Solution: A similar argument as in the previous problem works. Take an
orthormal basis ei for H. Then the elements Ei,j ∈ l2 (H), which for each i, i consist
of the sequences with 0 entries except the jth, which is ei , given an orthonromal
basis for l2 (H). Orthormality is clear, since with the inner product is
X
(.207) (u, v)l2 (H) = (uj , vj )H .
j

Completeness follows from completeness of the orthonormal basis of H since if


v = {vj } (v, Ej,i ) = 0 for all j implies vj = 0 in H. Now, to construct an isometric
isomorphism just choose an isomorphism m : N2 −→ N then
X
(.208) T u = v, vj = (u, em(i,j) )ei ∈ H.
i
I would expect you to go through the argument to check injectivity, surjectivity
and that the map is isometric.
Problem P10.3 Recall, or perhaps learn about, the winding number of a closed
curve with values in C∗ = C \ {0}. We take as given the following fact:1 If Q =
[0, 1]N and f : Q −→ C∗ is continuous then for each choice of b ∈ C satisfying
exp(2πib) = f (0), there exists a unique continuous function F : Q −→ C satisfying
(.209) exp(2πiF (q)) = f (q), ∀ q ∈ Q and F (0) = b.
Of course, you are free to change b to b + n for any n ∈ Z but then F changes to
F + n, just shifting by the same integer.
(1) Now, suppose c : [0, 1] −→ C∗ is a closed curve – meaning it is continuous
and c(1) = c(0). Let C : [0, 1] −→ C be a choice of F for N = 1 and
f = c. Show that the winding number of the closed curve c may be defined
unambiguously as
(.210) wn(c) = C(1) − C(0) ∈ Z.
Solution: Let C 0 , be another choice of F in this case. Now, g(t) =
0
C (t) − C(t) is continuous and satisfies exp(2πg(t)) = 1 for all t ∈ [0, 1]
so by the uniqueness must be constant, thus C 0 (1) − C 0 (0) = C(1) − C(0)
and the winding number is well-defined.
(2) Show that wn(c) is constant under homotopy. That is if ci : [0, 1] −→ C∗ ,
i = 1, 2, are two closed curves so ci (1) = ci (0), i = 1, 2, which are homo-
topic through closed curves in the sense that there exists f : [0, 1]2 −→ C∗
continuous and such that f (0, x) = c1 (x), f (1, x) = c2 (x) for all x ∈ [0, 1]
and f (y, 0) = f (y, 1) for all y ∈ [0, 1], then wn(c1 ) = wn(c2 ).
Solution: Choose F using the ‘fact’ corresponding to this homotopy
f. Since f is periodic in the second variable – the two curves f (y, 0),
and f (y, 1) are the same – so by the uniquess F (y, 0) − F (y, 1) must be
constant, hence wn(c2 ) = F (1, 1) − F (1, 0) = F (0, 1) − F (0, 0) = wn(c1 ).
(3) Consider the closed curve Ln : [0, 1] 3 x 7−→ e2πix Idn×n of n×n matrices.
Using the standard properties of the determinant, show that this curve
is not homotopic to the identity through closed curves in the sense that
there does not exist a continuous map G : [0, 1]2 −→ GL(n), with values in
1Of course, you are free to give a proof – it is not hard.
186 SOLUTIONS TO PROBLEMS

the invertible n × n matrices, such that G(0, x) = Ln (x), G(1, x) ≡ Idn×n


for all x ∈ [0, 1], G(y, 0) = G(y, 1) for all y ∈ [0, 1].
Solution: The determinant is a continuous (actually it is analytic)
map which vanishes precisely on non-invertible matrices. Moreover, it is
given by the product of the eigenvalues so
(.211) det(Ln ) = exp(2πixn).
This is a periodic curve with winding number n since it has the ‘lift’ xn.
Now, if there were to exist such an homotopy of periodic curves of matri-
ces, always invertible, then by the previous result the winding number of
the determinant would have to remain constant. Since the winding num-
ber for the constant curve with value the identity is 0 such an homotopy
cannot exist.
Problem P10.4 Consider the closed curve corresponding toLn above in the case
of a separable but now infinite dimensional Hilbert space:
(.212) L : [0, 1] 3 x 7−→ e2πix IdH ∈ GL(H) ⊂ B(H)
taking values in the invertible operators on H. Show that after identifying H with
H ⊕ H as above, there is a continuous map
(.213) M : [0, 1]2 −→ GL(H ⊕ H)
with values in the invertible operators and satisfying
(.214)
M (0, x) = L(x), M (1, x)(u1 , u2 ) = (e4πix u1 , u2 ), M (y, 0) = M (y, 1), ∀ x, y ∈ [0, 1].
Hint: So, think of H ⊕ H as being 2-vectors (u1 , u2 ) with entries in H. This allows
one to think of ‘rotation’ between the two factors. Indeed, show that
(.215) U (y)(u1 , u2 ) = (cos(πy/2)u1 + sin(πy/2)u2 , − sin(πy/2)u1 + cos(πy/2)u2 )
defines a continuous map [0, 1] 3 y 7−→ U (y) ∈ GL(H ⊕ H) such that U (0) = Id,
U (1)(u1 , u2 ) = (u2 , −u1 ). Now, consider the 2-parameter family of maps
(.216) U −1 (y)V2 (x)U (y)V1 (x)
where V1 (x) and V2 (x) are defined on H ⊕ H as multiplication by exp(2πix) on the
first and the second component respectively, leaving the other fixed.
Solution: Certainly U (y) is invertible since its inverse is U (−y) as follows in the
two dimensional case. Thus the map W (x, y) on [0, 1]2 in (.216) consists of invertible
and bounded operators on H ⊕ H, meaning a continuous map W : [0, 1]2 −→
GL(H ⊕ H). When x = 0 or x = 1, both V1 (x) and v2 (x) reduce to the identiy,
and hence W (0, y) = W (1, y) for all y, so W is periodic in x. Moreove at y = 0
W (x, 0) = V2 (x)V1 (x) is exactly L(x), a multiple of the identity. On the other
hand, at x = 1 we can track composite as
   2πix       4πx 
u1 e u1 u2 u2 e u1
(.217) 7−→ 7−→ 7−→ 7−→ .
u2 u2 −e2πx u1 −e4πx u1 u2
This is what is required of M in (.214).
Problem P10.5 Using a rotation similar to the one in the preceeding problem
(or otherwise) show that there is a continuous map
(.218) G : [0, 1]2 −→ GL(H ⊕ H)
SOLUTIONS TO PROBLEMS 187

such that
(.219) G(0, x)(u1 , u2 ) = (e2πix u1 , e−2πix u2 ),
G(1, x)(u1 , u2 ) = (u1 , u2 ), G(y, 0) = G(y, 1) ∀ x, y ∈ [0, 1].
Solution: We can take
   2πix 
Id 0 e 0
(.220) G(y, x) = U (−y) U (y) .
0 e−2πix 0 Id
By the same reasoning as above, this is an homotopy of closed curves of invertible
operators on H ⊕ H which satisfies (.219).
Problem P10.6 Now, think about combining the various constructions above
in the following way. Show that on l2 (H) there is an homotopy like (.218), G̃ :
[0, 1]2 −→ GL(l2 (H)), (very like in fact) such that
∞ ∞
(.221) G̃(0, x) {uk }k=1 = exp((−1)k 2πix)uk k=1 ,


G̃(1, x) = Id, G̃(y, 0) = G̃(y, 1) ∀ x, y ∈ [0, 1].


Solution: We can divide l2 (H) into its odd an even parts
(.222) D : l2 (H) 3 v 7−→ ({v2i−1 }, {v2i }) ∈ l2 (H) ⊕ l2 (H) ←→ H ⊕ H.
and then each copy of l2 (H) on the right with H (using the same isometric isomor-
phism). Then the homotopy in the previous problem is such that
(.223) G̃(x, y) = D−1 G(y, x)D
accomplishes what we want.
Problem P10.7: Eilenberg’s swindle For an infinite dimenisonal separable Hilbert
space, construct an homotopy – meaning a continuous map G : [0, 1]2 −→ GL(H)
– with G(0, x) = L(x) in (.212) and G(1, x) = Id and of course G(y, 0) = G(y, 1)
for all x, y ∈ [0, 1].
Hint: Just put things together – of course you can rescale the interval at the end
to make it all happen over [0, 1]. First ‘divide H into 2 copies of itself’ and deform
from L to M (1, x) in (.214). Now, ‘divide the second H up into l2 (H)’ and apply
an argument just like the preceding problem to turn the identity on this factor into
alternating terms multiplying by exp(±4πix) – starting with −. Now, you are on
H ⊕ l2 (H), ‘renumbering’ allows you to regard this as l2 (H) again and when you
do so your curve has become alternate multiplication by exp(±4πix) (with + first).
Finally then, apply the preceding problem again, to deform to the identity (always
of course through closed curves). Presto, Eilenberg’s swindle!
Solution: By rescaling the variables above, we now have three homotopies,
always through periodic families. On H ⊕ H between L(x) = e2πix Id and the
matrix
 4πix 
e Id 0
(.224) .
0 Id
Then on H ⊕ l2 (H) we can deform from
 4πix   4πix 
e Id 0 e Id 0
(.225) to
0 Id 0 G̃(0, x)
with G̃(0, x) in (.221). However we can then identify
(.226) H ⊕ l2 (H) = l2 (H), (u, v) 7−→ w = {wj }, w1 = u, wj+1 = vj , j ≥ 1.
188 SOLUTIONS TO PROBLEMS

This turns the matrix of operators in (.225) into G̃(0, x)−1 . Now, we can apply the
same construction to deform this curve to the identity. Notice that this really does
ultimately give an homotopy, which we can renormalize to be on [0, 1] if you insist,
of curves of operators on H – at each stage we transfer the homotopy back to H.
Bibliography

[1] W.W.L Chen, http://rutherglen.science.mq.edu.au/wchen/lnlfafolder/lnlfa.html Chen’s notes


[2] W. Rudin, Principles of mathematical analysis, 3rd ed., McGraw Hill, 1976.
[3] T.B. Ward, http://www.mth.uea.ac.uk/∼h720/teaching/functionalanalysis/materials/FAnotes.pdf
[4] I.F. Wilde, http://www.mth.kcl.ac.uk/∼iwilde/notes/fa1/.

189

You might also like